Chuyên đề đa thức và số học

Tài liệu chuyên đề đa thức và số học gồm 102 trang được biên soạn bởi các tác giả: Doãn Quang Tiến, Huỳnh Kim Linh, Tôn Ngọc Minh Quân, Nguyễn Minh Tuấn, bổ trợ cho học sinh trong quá trình ôn thi học sinh giỏi môn Toán.

| 1/102

Preview text:

DOÃN QUANG TIẾN HUỲNH KIM LINH TÔN NGỌC MINH QUÂN NGUYỄN MINH TUẤN CHINH PHỤC OLYMPIC TOÁN n n n
x y z Chuyên đề ĐA THỨC VÀ SỐ HỌC
TẠP CHÍ VÀ TƯ LIỆU TOÁN HỌC
TẠP CHÍ VÀ TƯ LIỆU TOÁN HỌC
Copyright © 2019 by Tap chi va tu lieu toan hoc.
All rights reserved. No part of this book may be reproduced or distributed in any form
or by anymeans, or stored in data base or a retrieval system, without the prior written the permission of the author. LỜI GIỚI THIỆU
ố học hay đa thức đều là các chủ đề thường xuyên xuçt hiện trong các đề thi học
S sinh giôi cçp quốc gia, các kì thi khu vực cũng như quốc tế với các bài toán khó
tới rçt khó được các nước cũng như các thæy cô phát triển rçt nhiều. Đa thức là
mâng mà chứa đựng trong nó các yếu tố về đäi số, giâi tích, hình học và câ các tính
chçt về số học. Chính vì thế ta có thể xem đa thức có thể xem như là các bài toán tổ
hợp giữa các mâng khác của Toán học cũng như đóng vai trò liên kết các mâng đó läi
với nhau thành một thể thống nhçt. Và chúng ta cũng biết rằng số học không phâi tự
nhiên rçt nhiều nhà toán học, những người làm toán gọi nó với cái tên mỹ miều là
chúa của toán học.
Thế giới các con số rçt quen thuộc với chúng ta trong cuộc sống
thường ngày, là một thế giới hết sức kì lä đæy bí èn: loài người đã phát hiện trong đó
bao nhiêu tính chçt rçt hay, bao nhiêu quy luật rçt đẹp và bçt ngờ đồng thời cũng đang
chịu "bó tay" trước nhiều sự kiện, nhiều dự đoán. Điều lí thú là nhiều mệnh đề khó
nhçt của số học được phát biểu rçt đơn giân, ai cũng hiểu được ; nhiều bài toán khó
nhưng có thể giâi rçt sáng täo với những kiến thức số học phổ thông đơn giân. Không ở
đåu như trong số học,chúng ta läi có thể læn theo được dçu vết của những bài toán cổ
xưa để đến được với những vçn đề mới đang còn chờ đợi người giâi – Trích từ cuốn
sách Số học – Bà chúa của toán học – Hoàng Chúng.
Chính vì thế sự kết hợp của 2
mâng kiến thức này sẽ mang tới cho chúng ta những bài toán đẹp nhưng vẻ đẹp thì
không bao giờ là dễ để chúng ta chinh phục câ, nó luôn èn chứa những điều khó khăn
và “nguy hiểm”. Trong chủ đề của bài viết này, chúng ta sẽ đi khám phá cũng như
chinh phục phæn nào vẻ đẹp của sự kết hợp đó.
Nhóm tác giả ĐA THỨC VÀ SỐ HỌC
TẠP CHÍ VÀ TƯ LIỆU TOÁN HỌC CHINH PHỤC OLYMPIC TOÁN Đa thức và số học CÂU CHUYỆN TOÁN HỌC
Joseph Louis Lagrange
ếu nhạc sĩ người Áo Wolgang Amadeus Mozart (1756 – 1791) đã để lại cho đời sau
N những bản nhạc tuyệt vời thì hơn hai trăm năm sau, trong những năm đầu tiên
của thế kỷ 21, với lòng tôn sùng một bậc tài danh, những người yêu âm nhạc cổ
điển chỉ còn biết lắng nghe để thưởng thức âm điệu mà thôi. Nhưng cùng thời với ông, ở
Âu châu còn có một thiên tài khác cũng lừng danh, nhưng tiếng tăm không vang ra ngoài
nhân thế vì ở trong bộ môn hạn hẹp là toán học. Tuy vậy công trình của ông để lại, không
những được người đời sau ghi chú học hỏi, mà còn được áp dụng trong nhiều bộ môn
khoa học thực dụng cho đời sống hàng ngày, và cả trong những chương trình thám hiểm
không gian và vũ trụ để tìm hiểu về nguồn gốc đời sống của con người và tương lai về ÁN O
sau. Người được nhắc đến trong bài này là Joseph Louis Lagrange (1736 – 1813), một nhà
toán học lỗi lạc nhất, mà cũng là người thật khiêm tốn, đã được nhiều bậc vương giả Âu
IC TP châu trọng vọng vào cuối thế kỷ 18 và đầu thế kỷ 19. Để phê bình về danh nhân này, Đại YM
đế Napoléon đã từng nói rằng: "Lagrange thật là một kim tự tháp cao vời trong bộ môn
toán học". Lời nói của Hoàng đế thường đi đôi với việc làm và ông đã phong OL C
cho Lagrange làm Bá tước, cử ông làm Thượng Nghị sĩ và còn vinh tặng ông Đệ Nhất Đẳng
Bắc Đẩu Bội Tinh. Nhiều bậc vương giả khác ở Âu châu như Quốc vương xứ Sardinia và PH
Hoàng đế Frederick của Đức quốc cũng đã hết mực tôn vinh Lagrange. NH I CH
Joseph Louis Lagrange (1736 1813) Chinh phục olympic toán| 1
Bồi dưỡng học sinh giỏi
Lagrange sinh ngày 25 – 1 – 1736 tại Turin (Italia), mất ngày 10 – 4 – 1813 tại Paris (Pháp).
Ông được xem là một trong những thiên tài toán học lớn nhất trong lịch sử toán học, đồng
thời cũng là một nhân vật đặc sắc trong thời đại ông – một thời đại đầy xáo động về mọi
mặt: chính trị, văn hóa, xã hội.
Ông là người Pháp, nhưng có pha dòng máu Ý. Tổ phụ của Lagrange là một đại úy kỵ binh
Pháp, đã tới phục vụ dưới trướng của Quốc vương đảo Sardinia là Charles Emmanuel
II. Sau đó vị sĩ quan kỵ binh điển trai và anh dũng này tới định cư ở tỉnh Turin và được
nhận vào làm rể của dòng họ quyền quý Conti của nước Ý. Thân phụ của Lagrange cũng
được hưởng cái may mắn trong hôn ước như thế và đã kết duyên cùng cô Marie Thérèse
Gros là ái nữ độc nhất của một bác sĩ giàu có ở tỉnh Cambiano. Cặp tài tử và giai nhân này
sống vui hạnh phúc và hai ông bà có đến mười người con nhưng tất cả đều mệnh yểu khi
còn tuổi ấu thơ và chỉ về sau mới may mắn được thêm cậu út là Joseph Louis ra đời ngày C Ọ
25 tháng Giêng năm 1736 để rồi lớn lên và trở thành một nhà bác học danh tiếng lẫy lừng. H
Thân phụ của Lagrange cũng là người có tài trí, và đã có thời làm Tổng Giám Đốc ngân
sách binh bị cho đảo quốc Sardinia. Ông xây dựng nên một tài sản khá lớn, lại cộng thêm ÁN
với của hồi môn của bà vợ nên gia đình được vào hạng giàu có lớn trong tỉnh. Nhưng ông
lại ham mê đầu tư nên theo với nền kinh tế đương thời ở Châu Âu, tài sản của gia đình bị U TOỆ
giảm sút dần dần đến khánh kiệt khi Lagrange bước vào tuổi trưởng thành. Cậu con út LI
được cưng chiều nay lại không được thừa kế chút di sản nào của cha mẹ, vì thật ra không TƯ
còn gì đáng giá để lại. Trong cuộc đời sau này của Lagrange, ông thường cho sự phá sản đó
lại là một điều may cho mình và đã nói rằng: "Nếu tôi được hưởng một gia tài lớn thì chắc VÀ Í
tôi đã không dựa vào Toán Học để xây dựng đời mình". CH P
Sự Nghiệp Toán Học ẠT
Vào đầu thế kỷ 18, nền khoa học nói chung, và toán học nói riêng, chưa phải là một môn
học chính cho sĩ tử, nên lúc mới đầu Lagrange theo về văn học cổ điển. Nhưng trong khi
nghiên cứu về văn hoá Hy Lạp, chàng thanh niên được biết đến những công trình về Hình
Học của những vĩ nhân toán học đời trước như Euclid (330 – 275 tr. CN) và Archimedes (287
– 212 tr. CN). Tuy vậy chàng cũng không chú ý lắm về những môn này. Nhưng sau
đó Lagrange được đọc một bài tham luận của nhà thiên văn học Edmund Halley (1656
1742) ca tụng môn Giải Tích Học mới được xây dựng và hoàn bị bởi nhà bác học Isaac
Newton (1642 – 1727) và cho rằng môn toán học này vượt trội hơn môn Hình Học. Bài này
gợi trí tò mò của chàng thanh niên và anh đã dồn hết tâm trí vào để trong một thời gian
ngắn học được hết những gì đã được công bố trên sách vở về những phép tính vi phân và
tích phân trong môn giải tích học. Sự hiểu biết về toán học cao cấp này đã làm
2 | Tạp chí và tư liệu toán học Đa thức và số học
cho Lagrange được bổ nhiệm làm giáo sư toán học tại Trường Pháo binh Hoàng gia ở tỉnh
Turin khi chàng mới 16 tuổi. Nơi đây, hàng ngày Lagrange giảng bài cho lớp sinh viên mà
người nào cũng lớn tuổi hơn mình. Tuy vậy chàng cũng thừa uy tín để chinh phục được
mọi người và có nhiều năng lực để tổ chức được một Hội Nghiên cứu Khoa học là khởi thủy
của một Trung tâm để sau này trở thành Viện Hàn lâm Khoa học Turin. Chỉ mấy năm sau,
vào năm 1759, khi Lagrange mới 23 tuổi, mà Hội Nghiên cứu do chàng sáng lập đã xuất bản
được Tập san đầu tiên. Nhưng ta phải nói rằng với một tâm địa tốt, luôn luôn nâng đỡ các
bạn đồng nghiệp mà nhiều bài khảo cứu toán học đăng trên những số đầu tiên của tập san
nghiên cứu, tuy ký tên những tác giả khác, mà thực ra là công trình của Lagrange vì đã
được chàng sửa chữa và viết lại hoàn toàn. Trong những trường hợp này, có một tác giả
của một bài viết thật đặc sắc – sau khi đã được Lagrange sửa lại – được mọi người chú ý và
ngợi khen, và khi chuyện tới tai quốc vương Sardinia, tác giả được vời tới và giao cho
giữ Bộ Hải quân là một chức vụ thật quan trọng vì Sardinia là một đảo quốc. Chỉ có một ÁN
điều là trong lịch sử môn toán học, người ta thấy ông này chỉ viết ra được một bài độc O
nhất là bài mà do sự nâng đỡ của Lagrange đã giúp cho ông được địa vị trong triều. Cũng IC T
trong thời gian sáng tác phong phú này mà Lagrange đã tạo dựng nên lý thuyết cho P
môn Cơ học Giải tích. YM OL
Một bài toán được biết từ thời thượng cổ là bài toán đẳng chu (isoperimetric problem) khi C Ụ
người ta tìm một hình phẳng có môt diện tích cực đại cho một chu vi cho sẵn. Lời giải tất
nhiên là hình tròn nhưng phải đợi đến thế kỷ 17 mọi người mới chú ý đến những bài toán PH
cực đại hay cực tiểu khi hai anh em toán gia Bernouilli, người Thụy Sĩ, ông anh tên
NH I là James (1654 – 1706) và người em là John (1667 – 1748) thách thức nhau giải bài toán sau đây: CH
"Từ một điểm khởi đầu O, thả trôi một cái vòng theo một đường giây nhẵn thín nằm trong mặt
phẳng thẳng đứng, để cho tuột xuống một điểm A ở dưới. Phải uốn đường giây theo hình nào để
cho thời gian tuột được ngắn nhất."
Dĩ nhiên hai anh em nhà Bernouilli không những đưa ra nhiều lời giải, nhưng lại còn đề ra
nhiều bài toán khác nữa thuộc loại này. Những bài viết của anh em nhà Bernouilli đã gây
phấn khởi cho một thiên tài toán học khác người Thụy Sĩ là Leonhard Euler (1707 – 1783) là
học trò của John Bernouilli, và Euler đã đưa ra phương pháp tổng quát để giải những bài
toán mà James Bernouilli đã đề nghị khi xưa. Ông cũng đặt tên cho phép tính này là Phép
tính biến thiên (Calculus of Variations). Nhưng ngưòi thực sự đã đưa phép giải những bài
toán để tìm ra những trường hợp tối ưu lại là Lagrange, lúc đó vẫn chỉ còn là một giáo sư ở
Turin. Tuy chàng thanh niên, mới ở tuổi 19 và ở thế hệ sau, chỉ nghiên cứu bài toán đẳng Chinh phục olympic toán| 3
Bồi dưỡng học sinh giỏi
chu sau những bậc tiền bối danh tiếng vang lừng, nhưng Lagrange đã có những nhận xét
tân kỳ để giải bài toán, và đã có can đảm viết một bức thư cho Euler, đang là Chủ tịch Ủy
ban Toán học của Viện Hàn lâm Khoa học Vương quốc Phổ ở Berlin, để đưa ra một lời giải mà
chàng cho là có tính cách tổng quát. Cũng may là Euler tuy là một thiên tài toán học thời
ấy, danh tiếng vang lừng, nhưng cũng là người rộng lượng, ông nhận ngay ra rằng
phương pháp của Lagrange đã giải toả được một vài thắc mắc của chính ông khi tìm
phương pháp giải bài toán và Euler đã nhường cho Lagrange công bố kết quả ra trước. Hơn
hai trăm năm sau, những khoa học gia không gian, khi tìm những qũy đạo tối ưu để đưa
những vệ tinh thám sát lên những hành tinh xa vời trong Thái dương hệ, đều phải viết
những phương trình có tên chung là phương trình Euler Lagrange. Không mấy người, dù
chỉ trong một khoảnh khắc, đã nghĩ đến tài trí siêu việt của Lagrange và đức tính cao
thượng của Euler, là những người đầu tiên đã khai phá ra môn toán học này. Trong những
năm đầu tiên của một cuộc đời nghiên cứu và sáng tác toán học của Lagrange, những bài C Ọ
viết đều được đăng trong những tập san đề là Miscellanea Taurinensia tất cả tổng cộng có 5 H
Tập. Những bài viết này dù là để tên những học sinh hay những người cộng sự đều là do
chàng giáo sư tuổi mới ngoài hai mươi đưa ra ý kiến và duyệt xét cùng sửa đổi lại. Tuy là ÁN
ở một thị thành hẻo lánh nơi có hội toán học mà Lagrange sáng lập mà sau này trở
thành Viện Hàn lâm Khoa học Turin, nhưng những tập san toán học phát xuất từ nơi đây, U TOỆ
mà ngay ở số đầu tiên đã nói về Phép tính biến thiên, đã được toàn thế giới khoa học ở Âu LI
châu chú ý tới và làm cho Lagrange đương nhiên trở thành một toán gia hàng đầu được TƯ
mọi người ngưỡng mộ. VÀ Í
Ngoài toán gia Euler, Lagrange còn được một trưởng bối người Pháp là D’Alembert (1717 – CH
1783) nhiệt tình ủng hộ. Những người bạn tốt này đều nghĩ rằng chỉ khi nào chàng tới một P
thủ đô văn học và tiếp xúc với những toán gia hàng đầu của thế kỷ thì tài năng ẠT
của Lagrange mới được nảy nở toàn diện. Trước đó Lagrange đã được mời tới London,
nhưng đi được nửa đường khi vừa tới Paris thì bị ốm. Nơi đây ông được tiếp đón trọng
vọng và vì sức khoẻ chưa hồi phục được nên đành phải trở về Turin một thời gian để chờ
cơ hội khác. Mấy năm sau thì dịp may đó tới khi đại toán gia Euler nhận lời mời của Viện
Hàn lâm Khoa học St Petersburg để chuyển cư tới đó. Do đề nghị của 2 nhà toán học
D’AlembertEuler, Hoàng đế Frederick của Phổ Quốc đã viết cho Lagrange một bức thư
đại để nói là Hoàng đế Frederick vĩ đại nhất châu Âu muốn được toán gia lừng danh nhất
của thế kỷ tới vương triều để hàng ngày cùng nhau bàn luận. Lagrange đã nhận lời để tới
Berlin thế vào chỗ trống của Euler và trong khoảng 20 năm khi cư ngụ ỏ Phổ Quốc ông đã
viết hơn một trăm bài khảo luận toán học để đăng trên các tập san ở Turin và ở Berlin.
Cũng trong thời gian này mà Lagrange hoàn tất tác phẩm vĩ đại nhất của đời ông về
môn Cơ học Giải tích.
4 | Tạp chí và tư liệu toán học Đa thức và số học
Khi mất ông được chôn cất trong điện Panthéon tại Paris.
Nguồn nội dung: Diễn đàn toán học Việt Nam – VMF
Nguồn ảnh: Wikipedia ÁN O IC TP YM OL C Ụ PH NH I CH Chinh phục olympic toán| 5
Bồi dưỡng học sinh giỏi Chuyên đề
ĐA THỨC VÀ SỐ HỌC
Tạp chí và tư liệu toán học
Trong chủ đề này, thay vì việc phân chia các dạng toán cụ thể kèm lời phân tích chi tiết
từng dạng thì mình sẽ mang tới cho bạn đọc một tuyển tập các bài toán hay và khó để ôn
tập và nâng cao kiến thức chuẩn bị cho kì thi học sinh giỏi cũng như các kì thi khác mà các
bạn tham gia. Nào chúng ta cùng bắt đầu nhé!
CÁC KIẾN THỨC CƠ BẢN C Ọ 1. ĐA THỨC. H ÁN
Đơn thức theo biến x là biểu thức có dạng . n
m x trong đó m là hằng số và n là số nguyên không âm. U TO
Đa thức là tổng hữu hạn của nhiều đơn thức hay đa thức là biểu thức có dạng Ệ k k1 LI
P x  a x a x
  a x a a k k ... k 0 . 1 1 0   TƯ Khi đó
ai được gọi là các hệ số của đa thức. VÀ Í Nếu a iP x i , thì ta gọi đa thức
  tức tập các đa thức hệ số nguyên. CH
n được gọi là bậc của đa thức, ký hiệu là deg P n . P ẠT
2. MỘT SỐ TÍNH CHẤT CẦN NẮM.
Tính chất 1. Với hai số nguyên a, b trong đó b  0 , nếu tồn tại số nguyên c sao cho a bc
thì ta gọi a chia hết cho b hoặc b chia hết a hoặc b là ước của a hoặc cũng hay gọi a là bội của b .
Ký hiệu. a b hoặc | b a .
Tính chất 2. Với P  x và a,b là hai số nguyên khác nhau, ta luôn có
P a  Pb a b.
Chứng minh. Giả sử P xn n1  a x a      x a x a a n n ... n 0 1 1 0  .
Sử dụng hằng đẳng thức k k
a b  a b k1 k2 k1 a
a b  ... b  với k  1 là số nguyên.
Khi đó P a  P b  a b a a   a b   b   a
a   a b   b      a n   n 1 n 2 n 1 ...
n n 2 n 3 n 2 ... ... . 1  1 
6 | Tạp chí và tư liệu toán học Đa thức và số học
Từ đó bài toán được chứng minh.
Tính chất 3. Cho đa thức P x với hệ số nguyên.
Khi đó không tồn tại ba số phân biệt a, b,c sao cho P a  b, P b  c.P c  . a
3. NHỮNG ĐỊNH LÝ QUAN TRỌNG.
Hai đa thức f x , g x được gọi là nguyên tố cùng nhau nếu ước chung lớn nhất của hai
đa thức đó là một hằng số.
Nhưng vì ước chung lớn nhất của hai đa thức chỉ khác nhau hằng số nên nếu hai đa thức
nguyên tố cùng nhau thì ta có thể xem ước chung lớn nhất nhất của chúng là 1. Nên ta ký
hiệu  f x , g x  1.
Định lý Bézout. Cho hai đa thức P x ,Q x  x. Gọi d x là ước chung lớn nhất của ÁN
hai đa thức P x ,Q x. O
 Khi đó tồn tai hai đa thức U x ,V x sao cho dx  U x.Px V x.Qx. IC TP
 Nếu Px ,Qx  1 thì tồn tại các đa thức U x,V x sao cho
U x.Px V x.Qx  1. YM
Định lý Schur. Cho P x  x là đa thức khác đa thức hằng. Khi đó có vô hạn các số OL C Ụ
nguyên tố p thỏa mãn tính chất: Ứng với số nguyên tố p tồn tại số nguyên m sao cho p P m. PH Chứng minh.
Ta xét các trường hợp sau
NH I Trường hợp 1. Hệ số tự do bằng 0. Khi đó p Pp với mọi số nguyên tố p. CH
Trường hợp 2. Hệ số tự do bằng 1. Tức là P 0  1 .
Ta giả sử tập các số nguyên tố thỏa mãn bài toán là hữu hạn. Gọi p là số nguyên tố lớn
nhất trong các số đó. Ta xét P p!  1mod p! . Ta gọi q  1 là số nguyên tố khác mà thỏa
mãn p P p! thì q p vì nếu q p thì do q p! nên từ q P p! ta suy ra q 1 là vô lí.
Nhưng q p thì lại mâu thuẫn với chuyện p là số nguyên tố lớn nhất.
Trường hợp nếu P 0  a  1 . 1
Ta xét đa thức Qx  P ax thì Q x  x và Q0  1 . Theo trên tồn tại vô hạn các số a
nguyên tố p sao cho ứng với mỗi số p thì luôn tồn tại số nguyên m để cho p Q m .
Nhưng nếu p Q x  p P ax . Định lí được chứng minh.
Định lý Dirichlet về số nguyên tố. Cho a,b là các số tự nhiên với a  0,a, b  1. Khi đó tập
hợp an b,n   chứa vô hạn số nguyên tố. Chinh phục olympic toán| 7
Bồi dưỡng học sinh giỏi
Định lý về dãy tuần hoàn. Cho f , g là hai đa thức hệ số nguyên và nguyên tố cùng nhau. Đặt a f n g n n a n gcd   ,   ,
1, 2, 3.... Khi đó dãy  n  tuần hoàn.
Chứng minh – Nguyễn Hữu Điển.
Do f , g là hai đa thức hệ số nguyên và nguyên tố cùng nhau nên tồn tại hai đa thức F,G
và số nguyên dương a sao cho f .F g.G a . Khi đó do chia hết cho cả f n , g n nên ta có
f na, .
n . Ta sẽ chứng minh an tuần hoàn theo chu kì a.
Ta chứng minh rằng a a n na .
Thật vậy ta có f n  f n amod a mà a a, a f n  a f n a n n n  .
Tương tự ta có a g n a a a n
. Như vậy ta có n na.
Lập luận tương tự ta có a a a a n
na . Vậy n na .
Chú ý. Hai đa thức nguyên tố cùng nhau khi ước chung lớn nhất của chúng là một đa thức hằng. C
Bổ đề Hensel. Cho đa thức f x hệ số nguyên và số nguyên tố p . Nếu phương trình đồng H 1 1 1
f x  0mod p có đúng r nghiệm phân biệt x , x ,, x thuộc đoạn 1; psao cho ÁN 1 2 r 1
f 'x   0mod p,i r  0 mod k f x p i
1, thì phương trình đồng dư   
có đúng r nghiệm U TO k
nguyên phân biệt thuộc đoạn 1, p    . LI Chứng minh.
Với k  1 hiển nhiên đúng.
Giả sử khẳng định đúng với k  1 . Điều đó có nghĩa là trên đoạn 1, k
p  , phương trình VÀ   Í    0mod k f x p  k k k k
có đúng r nghiệm x , x ,, x f 'x p i r i  0mod , 1, 1 2 r và . CH P Ta cần chứng minh    1 0 mod   k f x
p  có đúng r nghiệm thuộc k 1 1,   p    . ẠT Gọi x f x  0 mod p 1
0 là một nghiệm của phương trình       . Xét số   k x x
p t,t  0; p  1 1 0 
 với t là nghiệm duy nhất của phương trình
f x0   f ' x t p k   0 mod 0    p Ta sẽ chứng minh x 1 0 mod   k f x p 2 1 là nghiệm của      . nf ' x f ' x 2 f x 0 0  0  n
Ta có f x f x x x x x  x x 1   0 
   1 0   1 0  1 0  1! 2! n! n  f ' x f ' x f x f x f x p t p t  n k k k p t 1   0       2 0 0  0    1! 2! n! if x0  Suy ra     '   1 m d o    k k f x f x f x p t p  1 0 0  vì i!
8 | Tạp chí và tư liệu toán học Đa thức và số học f xf x   k   0   p
f 'x t p p k   k1 mod   0 k1 mod 1 0   p
Vậy phương trình 2 có ít nhất r nghiệm.
Thật vậy, giả sử x là nghiệm của 2 , gọi x 1
0 là nghiệm của   . Ta có    0 k1 mod
     mod k   mod k    k f x p f x f x p x x p x x p t 0    0   0 f x0 
Theo chứng minh trên thì t là nghiệm của phương trình  f ' x t p . k   0 mod 0    p Vậy phương trình    1 0 mod   k f x
p  có đúng r nghiệm.
Từ cách chứng minh trên ta rút ra được nhận xét sau.
Nhận xét. f k x
p tf x f 'x k p t k 1 mod     p 0 0 0 
Công thức nội suy Lagrange. ÁN
Cho đa thức P x có bậc nhỏ hơn n  1 và n  1 số thực phân biệt x , i  1,n i 1. O n1  x xj
Khi đó P x được xác định duy nhất như sau: P x   P xi  1  n . IC T ij x  1 1 x i j P ji YM OL C Ụ PH NH I CH Chinh phục olympic toán| 9
Bồi dưỡng học sinh giỏi ĐỀ BÀI
Câu 1. Tìm các đa thức P x có hệ số nguyên, không âm, bậc không lớn hơn 6 thỏa P 7  102013.
Câu 2. Cho a, b,c  thỏa mãn các đa thức f x 2
ax bx c
g x a b 2 
x c ax a b có nghiệm chung. Chứng minh rằng a b  2c 3 .
Câu 3. Tồn tại hay không đa thức f x 2
x ax b với a,b nguyên thỏa mãn 2
a  4b  0 và
nhận giá trị chính phương tại 2010 điểm phân biệt.
Câu 4. Chứng minh rằng không tồn tại đa thức P với hệ số nguyên, khác đa thức hằng có
bậc không quá 4 thỏa mãn: tồn tại 5 số nguyên x , x ,..., x 1 2 5 khác nhau sao cho
Px P x P x P x P x  1.  1 
 2   3  4   5 n x x Z x n  C Câu 5. Chứng minh
1 bất khả quy trên  , 2 Ọ
Câu 6. Cho đa thức P là đa thức hệ số và tồn tại số nguyên dương m sao cho H
P 1 ; P 2 ;...; P m không chia hết cho m . Chứng minh rằng P a  0 với mọi a . ÁN
Câu 7. Cho P xn n1  a x a x
 a x a
a , a ,, a n n1 1 0 là đa thức với 0 1
n là các số nguyên
dương. Đặt P x P x P x P P x k M kk1  1     và     với 1 . Tồn tại hay không 0 sao cho U TOỆ
với m M ta có | m P m P m   LI  
Câu 8. Cho đa thức P x là đa thức hệ số nguyên với bậc n  2 . Chứng minh rằng đa TƯ
thức P P x  x có nhiều nhất n nghiệm. VÀ Í
Câu 9. Tìm tất cả các đa thức P hệ số nguyên sao cho  |2n P n
 1 với mọi số nguyên CH dương n . P
x , x ,, x x x   xCâu 10. Gọi 0 1
n là các số nguyên thỏa mãn 0 1
n . Chứng minh rằng một T n!
trong các số P x , P x , P x 0   1
n không nhỏ hơn với Px là đa thức bậc n. 2n
Câu 11. Giả sử các đa thức P x ,Q x , R x và S x thỏa mãn: P 5
x   xQ 5 x  2  x R 5 x    4 3 2
x x x x  1Sx
Chứng minh rằng khi đó đa thức P x chia hết cho x  1 .
Câu 12. Tìm tất cả các đa thức P có hệ số nguyên sao cho với mọi số nguyên tố p và mọi
cặp số tự nhiên u, v thỏa mãn P
| uv  1 thì ta có P PuPv  1 .
Câu 13. Tìm tất cả các đa thức P hệ số nguyên sao cho với mọi số tự nhiên a, b,c ta luôn
a b c P a  P b  P c .
Câu 14. Cho m, n là các số nguyên dương. Chứng minh rằng n m khi và chỉ khi
P xQx , trong đó Px ;Qx là các đa thức hệ số nguyên được xác định:
10 | Tạp chí và tư liệu toán học Đa thức và số học Pxn1 1 n2 n1
x C x  C n ... n Qxm1 1 m2 m1  xC x  C m ... m
Câu 15. Tìm tất cả các đa thức P hệ số nguyên thỏa mãn tính chất: Với m, n là hai số
nguyên tố cùng nhau thì hai số P m ; P n cũng là hai số nguyên tố cùng nhau.
Câu 16. Cho P x ,Q x  x là hai đa thức monic bất khả quy thỏa mãn với n đủ lớn thì
P n ,Qn có cùng tập ước nguyên tố. Chứng minh rằng P Q .
Câu 17. Gọi đa thức P x  x khác đa thức hằng và gọi n, k là hai số nguyên dương.
Chứng minh rằng tồn tại số nguyên dương a sao cho mỗi f a , f a  1 ,..., f a n 1  có
ít nhất k ước nguyên tố phân biệt.
Câu 18. Cho đa thức P x ,Q x là hai đa thức có hệ số nguyên nguyên tố cùng nhau. Đặt a P n Q n a n   ,  
. Chứng minh dãy  n tuần hoàn. ÁN
Câu 19. Với hai đa thức có hệ số nguyên p x ,q x ta viết p x  q xmod 2 nếu O
px  q x có tất cả các hệ số đều chia hết cho 2. Cho dãy đa thức p x n   thỏa mãn IC T
p x p x  1 p x p x xp x nnn 1 1   2   P và 2   1  
   , với mọi n là số tự nhiên. Chứng minh rằng p x n    1mod2 . YM 2
Câu 20. Giả sử p x p x p n
 1 là hai số tự nhiên lớn hơn 1 và n   . OL 2 n C
Dãy số vô hạn n  1, 2, 3... được xác định như sau Ụ p          p xp p xp xp x p xp n n n n n n 1 4 3 2  2 1  2   n 2 n1 PH
Chứng minh rằng trong dãy số nói trên chứa vô hạn số đôi một nguyên tố cùng nhau. NH f n I
Câu 21. Với mỗi số tự nhiên n , ta kí hiệu   là tổng các chữ số của nó biểu diễn trong
hệ thập phân. Ta xây dựng dãy số như sau CH
u n f n ;u n f f n ;...;u n f f f n k ... ... 1     2             k
Chứng minh rằng với mọi số tự nhiên n , tồn tại số tự nhiên d sao cho u n u n k   d k   d  , 1 .
Câu 22. Cho abc là một số nguyên tố.
Chứng minh rằng phương trình 2
ax bx c  0 1 không có nghiệm hữu tỉ.
Câu 23. Cho đa thức P x   2 x   2 x   2 2
3 x  6 . Chứng minh rằng với mọi số nguyên tố
p đều tìm được số nguyên dương n sao cho P np .
Câu 24. Chứng minh rằng với mọi số nguyên p ta có thể tìm được số nguyên dương
f x sao cho n không phải là số chính phương.
Chinh phục olympic toán| 11
Bồi dưỡng học sinh giỏi
Câu 25. Cho đa thức P x 2017 1000  xx
 1 . Tồn tại hay không các số tự nhiên a , a ,... 1 2 , a P a P a a ai . 2018 sao cho tích
   j i j với mọi i j .
Câu 26. Với mọi số tự nhiên m, n , chứng minh rằng n! chia hết cho m khi và chỉ khi tồn n
tại đa thức hệ số nguyên f xk  a xk
thỏa mãn a , a ,..., a m m f j n , 1, 0 1    với mọi j k0 nguyên dương.
Câu 27. Cho đa thức f x 5 4 3 2
 2009x x x x  2006x  1. Chứng minh rằng với n là số
nguyên tùy ý thì các số f n , f f n ,..., f ... f n... đôi một nguyên tố cùng nhau. 1 n 1   
Câu 28. Gọi P x là đa thức bậc n thỏa mãn với k  0,1,, n thì P k     k
Định đa thức P n  1 . C
u  1990,u  1989,u  2000 Ọ
Câu 29. Cho dãy un được xác định như sau 1 2 3  . u        u u u n n 19 n 9 n n 1991, 1,2... 3 2 1 H
a) Với mọi n gọi r u r
n là số dư của phép chia
n cho 1992. Chứng minh rằng dãy  n là một ÁN dãy tuần hoàn. U TO
b) Chứng minh rằng tồn tại vô số số x của dãy un sao cho Ệ 1992 1994 1975 1945 1990 2       LI 5x 5x 4x 8x 2x 11x 48 chia hết cho 1992. TƯ
Câu 30. Chứng minh rằng tồn tại hằng số dương c sao cho với mọi số nguyên dương n
và các số thực a , a ,, a
P x x a x a x a 1 2 n , nếu    1   2   n  thì Í
max Px  n
c max Px CH x   0,2 x   0,1 P
Câu 31. Tìm tất cả các đa thức P x  x và  m  sao cho  2n m
P n là số chính ẠT
phương với mọi số nguyên dương n .
Câu 32. Cho P x  x , p là số nguyên tố và x a mod p . Chứng minh rằng
Px  Pa  x aP'a 2 mod p
Câu 33. Với p là số nguyên tố, đặt h x là đa thức có hệ số nguyên sao cho
h  h   h 2
0 , 1 , , p  1 là một hệ thặng dư đầy đủ modulo 2
p . Chứng minh rằng
h  h   h 3
0 , 1 , , p  1 là một hệ thặng dư đầy đủ modulo 3 p .
Câu 34. Với số nguyên n  3 , đặt f x , g x là đa thức với hệ số thực sao cho các điểm
f 1,g1, f 2, g2,, f n, gn trong tập 2 là các đỉnh của đa giác n cạnh
theo thứ tự ngược chiều kim đồng hồ. Chứng minh rằng ít nhất một trong các đa thức f , g
có bậc không nhỏ hơn n  1 .
12 | Tạp chí và tư liệu toán học Đa thức và số học p  1
Câu 35. Cho p là một số nguyên tố lẻ. Chứng minh rằng có ít nhất giá trị 2 p1
n0,1,2, p  
1 sao cho  k! k
n không chia hết cho p . k0
Câu 36. Có tồn tại hay không một dãy số thực và khác 0 là a ; a ;, a n 1 2 n thỏa với mỗi 
thì đa thức a a x  n a x 0 1 n
có đúng n nghiệm trên .
Câu 37. Cho P x  x thỏa mãn P x là số chính phương với mọi x nguyên thì P x 2
Q x với Qx x .
Câu 38. Tìm tất cả các đa thức số hệ số nguyên thỏa mãn a b là số chính phương thì
P a  P b cũng là số chính phương, trong đó a,b là các số tự nhiên.
Câu 39. Giả sử m, p là các số nguyên tố khác nhau. Chứng minh rằng nếu có một số tự  
nhiên x nào đó mà p là ước của P x   m 1 m 2 xx
 ... 1 thì ta có p  1 modm . ÁN O
Câu 40. Cho đa thức P x có bậc n và có n nghiệm phân biệt x , x ,, x 1 2 n . Chứng minh rằng: IC TP P' x P' x P' x 1   2   n a)    0 P'x P' x P' x 1   2   n YM 1 1 1 OL b) P     0 ' x P' x P' x 1   2   n C Ụ
Câu 41. Cho f là một đa thức có hệ số hữu tỉ và bậc không nhỏ hơn 2, và dãy an  chỉ PH
gồm các số hữu tỉ thỏa mãn f a   a n 1 
n với mọi số nguyên dương n. Chứng minh rằng NH a I
dãy  n  tuần hoàn. P' 1   n CH
Câu 42. Cho P x là đa thức bậc n với hệ số thực sao cho P 1 khác 0 và  P  1   2
Chứng minh rằng P x luôn có ít nhất một nghiệm x x  1 0 sao cho 0 .
Câu 43. Giả sử tồn tại đa thức hệ phức P,Q, R thỏa mãn a b c P Q
R a,b,c   . 1 1 1
Chứng minh rằng    1 a b c
Câu 44. Cho đa thức P x và Q x với số thực k bất kì thỏa mãn P  z
Pz  k k và
Q  z |Qz  k P Q P Q P x Q x k . Chứng minh rằng 0 0 và 1 1 suy ra được     .
Câu 45. Cho đa thức P x  x,deg P  2 . Chứng minh rằng tồn tại  m
để P m! là hợp số.
Chinh phục olympic toán| 13
Bồi dưỡng học sinh giỏi
Câu 46. Cho f x là đa thức với hệ số hữu tỉ bậc lớn hơn hoặc bằng 2. Xét dãy an các số
hữu tỉ thỏa mãn điều kiện f a   k   a n n n , 1 1 
. Chứng minh rằng tồn tại 1 để a    a n n k n  1 .
Câu 47. Tìm tất cả đa thức
 x sao cho   2p p
p,p là số nguyên tố
Câu 48. Xét đa thức T x 3 2
x  17x  1239x  2001. Đặt
T x T x ,T x T T x ,,T x T T x n  1     2    1   n1  
n   với mọi 1,2,3...
Chứng minh rằng tồn tại số nguyên n  1 sao cho T x  x n
chia hết cho 2003 với mọi số nguyên x .
Câu 49. Tìm tất cả các cặp số nguyên a, b sao cho tồn tại đa thức P x  x sao cho tích
 2x axbPx là đa thức được viết dưới dạng: n n1 x c x   c x c
c ,c ,,cn1 1 0 với 0 1 n1 bằng 1 hoặc 1 C Ọ
Câu 50. Cho hai đa thức hệ số nguyên n n H Px 1  a x a x
 a x a n n1 1 0 n n ÁN Qx 1  b x b x
 b x b n n1 1 0
Biết rằng a b ab n
n là một số nguyên tố và n1
n1 . Gọi m là là một nghiệm hữu tỷ chung U TOỆ
của hai đa thức. Chứng minh rằng m là một số nguyên. LI
Câu 51. Hỏi có tất cả bao nhiêu đa thức P x
n   bậc n chẵn thỏa mãn các điều kiện TƯ
 Các hệ số của P x M  0; 1  ;1 Pn 0 0
n   thuộc tập   và   .  2   VÀ
Tồn tại đa thức Q x có các hệ số thuộc M sao cho P x x 1.Q x n  . Í 
Câu 52. Cho dãy số nguyên a m n a n  thỏa mãn |
a với mọi số tự nhiên m,n phân CH n1 m n P
biệt. Giả sử tồn tại đa thức P x sao cho a P n n n
 , . Chứng minh rằng tồn tại đa thức ẠT
Qx sao cho a Qn ,n n .
Câu 53. Cho n là số nguyên dương và a , a ,..., a 1 2
n là các số thực dương.
Ta đặt g x  x a
x a ... x an . 1   2    Gọi a nn
f x x a g x x
b x  ... b x b n n .
0 là một số thực bất kì và đặt       1 0 1 1
Chứng minh rằng b , b ,..., b
a a a  ...  an. 1 2
n1 đều là số âm khi và chỉ khi 0 1 2
Câu 54. Cho F là tập các đa thức  có hệ số nguyên và phương trình  x  1 có nghiệm
nguyên. Cho trước một số nguyên dương k, tìm giá trị nhỏ nhất của m  1 theo k thỏa
mãn tồn tại   F sao cho  x  m có đúng k nghiệm nguyên phân biệt.
14 | Tạp chí và tư liệu toán học Đa thức và số học
Câu 55. Cho hai đa thức P x ,Q x  x nguyên tố cùng nhau và khác đa thức hằng.
Chứng minh rằng không có quá ba số thực  thỏa mãn P x  Q x là bình phương của một đa thức.
Câu 56. Chứng minh rằng nếu đa thức f x  x có bậc n và nhận giá trị nguyên tại
n  1 giá trị nguyên liên tiếp từ a a n, a thì f x , x   .
Câu 57. Tìm tất cả các đa thức P x  x sao cho với mọi a,b a không là nghiệm
của P x thì P aP a b  P b .
Câu 58. Xét đa thức P xn n1  x a x
  a x a n nP x n ... 1 1 0 với 2,
. Giả sử   có n
nghiệm là x , x ,..., x max x
x , x ,..., xn. n . 1 2 Kí hiệu
i  là số lớn nhất trong các số 1 2 Chứng 1
minh rằng P x   2 n1 n
 2n  ,x  maxx   i  với 0 . ix  1 xi ÁN
Câu 59. Giả sử R là nghiệm của phương trình n n1 x a x  ... a    x a n n 0 1 1 và đặt O n n A  a B ja A B A R j ,
  j. Khi đó thì ta có . IC T j1 j1 P
Câu 60. Cho đa thức P x là đa thức monic bậc n  1 có n nghiệm thực là x , x ,..., x 1 2 n YM
phân biệt và khác 0 . Chứng minh rằng: OL 1 1 1  1  n1 C   ...  Ụ x Px x Px x Px x x x n n ... 1  1 2  2    1 2 n PH n x
Câu 61. Cho n  2 và đa thức P x xác định bởi P x   k . Chứng minh rằng phương k0 k ! NH I
trình P x  0 không có nghiệm hữu tỉ. CH
Câu 62. Cho p là một số nguyên tố. Tìm tất cả các đa thức f x với hệ số nguyên sao cho
với mọi số nguyên dương n , f x là ước của n p  1.
Câu 63. Cho số nguyên dương n và số nguyên tố p lớn hơn n  1 . Chứng minh rằng đa 2 p x x x
thức P x  1    không có nghiệm nguyên. n  1 2n  1 pn  1
Câu 64. Cho đa thức P x 3 2
x  11x  87x mm  . Chứng minh rằng với mọi m tồn tại
số nguyên n sao cho P n 191 .
Câu 65. Cho m là số nguyên dương, tìm số nghiệm của phương trình 2
x xmodm .
Câu 66. Cho p là số nguyên tố  p  3 . Xét đa thức
f x  p   p2 x
 p   p3 2 1 2 x
 3x  2x  1 .
Chinh phục olympic toán| 15
Bồi dưỡng học sinh giỏi
Biết rằng hệ A  a , a ,, a 1 2
p là một hệ thặng dư đầy đủ modulo p. Chứng minh rằng khi
đó hệ B   f a , f a ,, f a 1   2 
p cũng là một hệ thặng dư đầy đủ modulo p.
Câu 67. Xét đa thức P x 3 2
x  14x  2x  1. Chứng minh rằng tồn tại số nguyên dương n
sao cho với mọi số nguyên x ta có 101 P PP x    x .
Câu 68. Cho tập S  p , p ,, p P x 1 2
k  là tập hợp k số nguyên tố phân biệt và   là đa thức
với hệ số nguyên sao cho với mọi số nguyên dương n đều tồn tại pi trong S sao cho p P n p P n n i
 . Chứng minh rằng tồn tại i sao cho   * ,  i .
Câu 69. Cho đa thức P x 3 2
x  153x  111x  38 .
a) Chứng minh rằng trong đoạn 2000 0;3  
 tồn tại ít nhất một số nguyên dương a sao cho Pa 2000 3 . C b) Hỏi trong đoạn 2000 0;3  
 có tất cả bao nhiêu số nguyên dương a sao cho P a chia hết Ọ H cho 2000 3 .
Câu 70. Tìm tất cả các đa thức f với hệ số nguyên sao cho f nf m  n m . ÁN
Câu 71. Cho a, b,c, d, e, f là các số nguyên dương. Giả sử rằng S a b c d e f là ước U TO
của các số abc def ab bc ca de ef fd . Chứng minh rằng S là hợp số. Ệ LI
Câu 72. Tìm tất cả các đa thức P với hệ số nguyên thỏa mãn Pnn *
|2557  213.2014,n TƯ
Câu 73. Cho P là đa thức hệ số nguyên, có bậc n  1 và k là số nguyên dương bất kỳ. Xét VÀ Í đa thức    k Q x
P x với P được tác động k lần. Chứng minh rằng có nhiều nhất n số CH
nguyên t sao cho Q t  t . P Ạ
Câu 74. Cho A là tập vô hạn các số nguyên dương. Tìm tất cả các số nguyên dương n T
thỏa mãn với mọi a là phần tử của A thì 2 n 1! 2! n!
1  a a  ...  a 1  a a  ...  a
Câu 75. Cho P,Q là hai đa thức hệ số nguyên không âm, khác đa thức hằng. Xét dãy số Pnx  2016  Q n n n
 ,  1. Chứng minh rằng tồn tại vô hạn số nguyên tố p thỏa mãn: ứng
với mỗi p , tồn tại số nguyên dương m sao cho p xm .
Câu 76. Tìm tất cả các đa thức P hệ số nguyên thỏa mãn   2p P p
p , với mọi số nguyên tố p .
Câu 77. Cho P x ,Q x là các đa thức hệ số nguyên khác đa thức hằng. Giả sử rằng đa
thức P x.Q x  2009 có ít nhất 25 nghiệm nguyên phân biệt. Chứng minh rằng bậc của
mỗi đa thức P x ,Q x đều không nhỏ hơn 3.
16 | Tạp chí và tư liệu toán học Đa thức và số học
Câu 78. Gọi d n là ước nguyên tố nhỏ nhất của số nguyên n, với n   1  ,0,  1 và ta kí hiệu d  1
   d0 ,d1  0.
Tìm tất cả các đa thức P x với hệ số nguyên thỏa mãn P n d n  n d P n.
Câu 79. Tìm tất cả các số nguyên dương k thỏa mãn tồn tại đa thức f x với các hệ số
đều nguyên, có bậc lớn hơn 1 sao cho với mọi số nguyên tố p và mọi số tự nhiên a, b
p ab k thì p f af b  k .
Câu 80. Cho P là đa thức hệ số nguyên thỏa mãn P 0  0 và P 0 , P 1 ,...  1 . Chứng
minh rằng có vô hạn số n sao cho P n  P 0 , P n  1  P 1 ,    n .
Câu 81. Cho p là số nguyên tố và P x là các đa thức bậc d hệ số nguyên thỏa mãn
P0  0, P1  1
 Với mọi số nguyên dương n thì số dư trong phép chia Pn cho p là 0 hoặc 1 . ÁN O
Chứng minh rằng d p  1 .
Câu 82. Chứng minh rằng không tồn tại đa thức P hệ số thực deg P n  1 sao cho P m
IC TP là số nguyên tố với mọi số nguyên dương m.   n n1      YM Câu 83. Cho n
,n 3 và đa thức f xx a ax a x a x n 1 1 0   thỏa mãn 0    OL chẵn, a a k n k
nk chẵn với mọi 1, 1 . C  g h
Giả sử thêm rằng tồn tại hai đa thức g x , h x x thỏa mãn deg deg , mọi hệ số PH
của h x đều lẻ và f x  g x.h x , x  .
Chứng minh rằng f x có ít nhất một nghiệm nguyên.
NH I Câu 84. Cho đa thức f x monic, hệ số nguyên, bất khả quy và f 0 không phải là số CH
chính phương. Chứng minh rằng     2 g x
f x  cũng là đa thức bất khả quy.
Câu 85. Cho đa thức hệ số nguyên f xn n1  a x a    x a x a n n 1 1 0 thỏa mãn điều kiện
a a a  a a f x 0 1 2 n và 0 là số nguyên tố thì   bất khả quy.
Câu 86. Tìm tất cả các đa thức P x ,Q x hệ số nguyên thỏa mãn với dãy số xn  xác định
bởi x  2014, x      P x x Q x n n n , n n , 0 2 1  2  2 2  2 1 
thì mỗi số nguyên dương m là ước
của một số hạng khác 0 nào đó của x . n
Câu 87. Chứng minh rằng không tồn tại đa thức Pxn n1  a x a     n   x a x a x n n ... 1 1 0   bậc 1
sao cho P 0 , P 1 ,... đều là số nguyên tố.
Câu 88. Cho đa thức P xn n1  x a     a a   ax a x a P x x n ... , 1 1 0  
  với 0 chẵn và n k k
chẵn, với mọi k  1, n  1. Giả sử P x  Q xR x , với Q x , R x  là các đa thức hệ số
Chinh phục olympic toán| 17
Bồi dưỡng học sinh giỏi
nguyên khác hằng, deg Qx  deg Rx và tất cả các hệ số của R x đều lẻ. Chứng
minh rằng, đa thức P x có nghiệm nguyên. n
Câu 89. Chứng minh rằng với mọi số nguyên dương n thì đa thức P x  x x2 2  1 là
đa thức bất khả quy trên .
Câu 90. Giả sử n là số tự nhiên lớn hơn hoặc bằng 2 và P xn n1  x a     x a x n ... 1 1 1 là
đa thức hệ số nguyên dương. Giả sử a a k n k
nk với mọi 1,
1. Chứng minh rằng tồn tại y P  x
vô hạn cặp số nguyên dương x, y sao cho:  * . x P y 
Câu 91. Chứng minh rằng, với mỗi số nguyên dương n tồn tại đa thức P x  x bậc n
sao cho P 0 , P 1 ,..., P n phân biệt và tất cả các số đó đều có dạng 2.2019k 3, k    . C Ọ
Câu 92. Chứng minh rằng tồn tại tập vô hạn các điểm H
...,P ,P ,P ,P ,P ,P ,P ,... 3 2 1 0 1 2 3  ÁN
trong mặt phẳng thỏa mãn tính chất: Với ba số nguyên a, b,c phân biệt thì các điểm P P P
a b c
a , b , c thẳng hàng khi và chỉ khi 2014. U TO
Câu 93. Cho x x  ...  x n, n  3 số thực thỏa mãn: Ệ 1 2 n LI
x x x x x x  ...  x x 2 1 3 2 4 3 n n1 TƯ
Giả sử đa thức P x có n nghiệm thực là các giá trị x , x ,..., xn. 1 2
Chứng minh rằng giá trị
lớn nhất của P x đạt được tại một điểm x x x n , n . 0  1  VÀ Í
Câu 94. Cho P xn n1  a x a     x a x a n n ... 1 1
0 là đa thức hệ số nguyên thỏa mãn điều kiện CH     P
PrPs 0 trong đó r,s là các số nguyên thỏa mãn điều kiện 0 r . s Chứng minh ẠT
rằng tồn tại k 0,1, 2,..., 
n sao cho a  s k .
Câu 95. Cho P x ,Q x là các đa thức hệ số nguyên. Đặt a n n n ! . Chứng minh rằng nếu Pa Pnn    n  thì  n
 và Qn  0 . Q a Qn , n  ,
Câu 96. Cho P x là đa thức bậc n  5 với hệ số nguyên và Pxn n1  a x a     x a x a n n ... 1 1 0
Giả sử P x có n nghiệm nguyên phân biệt là 0,  ,..., n. 2
Tìm tất cả các số nguyên k sao
cho thỏa mãn P P k  0.
Câu 97. Tìm số nguyên dương n nhỏ nhất sao cho tồn tại đa thức P x bậc n có hệ số
nguyên thỏa mãn: P 0  0, P 1  1 và với mọi *   thì
P2P1P là bội của p với p là số nguyên tố
18 | Tạp chí và tư liệu toán học Đa thức và số học
Câu 98. Chứng minh rằng với mọi m
tồn tại đa thức P x m
 có hệ số hữu tỉ thỏa mãn với mọi * n  thì 2m1 2m1 2m1 1  2  ...  nP n n m   1
Câu 99. Cho P x ,Q x , R x là ba đa thức hệ số thực thỏa mãn:
P Qx  PRx  c, x
  với c const
Chứng minh rằng: P x là hằng số hoặc Qx  R x là hằng số.
Câu 100. Chứng minh rằng tồn tại các đa thức hệ số nguyên S ,S , 1 2 tương tứng với các
biến x , x ,, y , y , n  1 2 1 2
thỏa mãn với mọi số nguyên 1 n n n   d d d
dS  dx y d d d  * | d n | d n  
Với hàm tổng chạy qua các ước nguyên dương d của n .
Chú ý. Lưu ý rằng ta chỉ xét đến các đa thức trong trường x . Ví dụ, xét hàm S x y 1 1 1 ÁN
S x y x y n  2 2 2 1 1 trong trường hợp 2 ta được O 2
S  2S   2 2
x y  2  x y * 1 2 1 1 
 2 2  là hàm   thỏa mãn yêu cầu bài toán. IC T P YM OL C Ụ PH NH I CH
Chinh phục olympic toán| 19
Bồi dưỡng học sinh giỏi HƯỚNG DẪN GIẢI
Câu 1. Tìm các đa thức P x có hệ số nguyên, không âm, bậc không lớn hơn 6 thỏa P 7  102013.
Đề chọn đội tuyển DakLak 2014 Lời giải Ta có P xn n1  a x a x
 a x a n n1 1 1 0  P7 n n1  a a a
a P7  a a a n 7    n 7 7 1 1 0 n n1 0  7 Lại có 5 3 2 0
102013  6.7  3.7  2.7  6.7  2.7  6032627  P x 5 3 2
 6x  3x  2x  6x  2 .
Nhận xét. Ý tưởng các dạng bài toán này là ta đưa về việc xử lý dữ kiện đề bài dưới dạng
chuyển đổi linh hoạt các hệ cơ số để đơn giản hóa cách làm cho bài toán. C Ọ H
Câu 2. Cho a, b,c  thỏa mãn các đa thức f x 2
ax bx c và 2 ÁN
g x  a bx c ax a b có nghiệm chung. Chứng minh rằng a b  2c 3 . Lời giải U TO 2 Ệ
Ta có f x  g x  a b cx x  1 . Giả sử x0 là nghiệm chung của 2 phương trình LI
f x  0 và g x  0 . Khi đó
 Nếu a b c  0 thì do a b  2c a b c mod 3 nên a b  2c 3
 Nếu a b c  0 , thì do x là nghiệm chung của f x và gx nên x là nghiệm Í 0 0 của phương trình  2
x x  1  0 . Theo định lý về phép chia với số dư, ta có CH P
f x  a 2
x x  1  r x * ẠT
trong đó r  x,deg r  2 . Trong * , thay x x0 ta được
f x   a   2 1 5 0
x x  1  r x x  ,r x  0 0 0 0   0 0  0  2
Từ đó, do r  x ,deg r  1, r x  0 x r x  x  0  và 0 nên   0 suy ra
f x  a 2
x x  1
Và do đó b a,c  a suy ra a b  2c  0 3 .
Câu 3. Tồn tại hay không đa thức f x 2
x ax b với a,b nguyên thỏa mãn 2 a  4b  0
và nhận giá trị chính phương tại 2010 điểm phân biệt. Lời giải
Tồn tại đa thức bậc hai có tính chất như vậy. Thật vậy:
20 | Tạp chí và tư liệu toán học Đa thức và số học Xét f x 2
x ax b , ta có f x  x ax b   x a2 2 2 4 4 4 4 2  4b a
Giả sử tồn tại x , x ,..., x
y , y ,..., y , x x , 1
  i j i j 2010 1 2 2010; 1 2 2010 
 là các số nguyên thỏa
4 f x  2x a b a i 4  1   2 2 mãn  i  1, 2010  4  f x y i  2  4 i Suy ra 2
4b a  2y  2x a2y  2x a i i i i i  1, 2...2010  a 4
Chọn a, b thỏa mãn 
(P là các số nguyên tố phân biệt) 2 4b a  
16P .P ...P i 1 2 2010  2y x a P i 2   i 4  i Xét hệ phương trình  4P P ...Pi  1 2 2010
2y  2x a  , 1.2...2010  i i PiP P ...P 1 2 2010  y P    i i P ÁN Giải hệ, thu được  i , thỏa mãn. O P P ...  P a 1 2 2010 x   P   1  i Pi 2
IC TP Rõ ràng x x  1i j i j  YM
Vậy tồn tại đa thức thỏa mãn yêu cầu. OL C
Câu 4. Chứng minh rằng không tồn tại đa thức P với hệ số nguyên, khác đa thức hằng Ụ
có bậc không quá 4 thỏa mãn: tồn tại 5 số nguyên x , x ,..., x 1 2 5 khác nhau sao cho PH
Px P x P x P x P x  1.  1 
 2   3  4   5 NH I Lời giải
Không mất tính tổng quát, ta chỉ xét ba trường hợp sau: CH
Trường hợp 1. Với Px P x P x P x P x  1 1   2   3   4   5  .
Khi đó đa thức P x  1 có bậc không quá 4 mà có đến 5 nghiệm nguyên khác
nhau. Điều đó dẫn đến: P x  1
 (loại vì đa thức P cần tìm khác đa thức hằng).
Trường hợp 2. Với Px P x P x  1;P x P x  1 1   2   3   4   5  . Khi đó:
2  Px P x P x P x P x P x 4   1  4  2   4  3  2  P
x P x P x P x P x P x 5   1  5  2   5  3
Ta có 2 là bội của các số x x ; x x ; x x ; x x ; x x ; x x 4 1 4 2 4 3 5 1 5 2 5 3 .
Điều n ày là không thể.
Trường hợp 3. Với Px P x P x P x  1;P x  1 1   2   3   4   5  .
Khi đó đa thức P x  1 có bốn nghiệm là x ; x ; x ; x 1 2 3 4 phân biệt.
Do đó P x  1  K.x x x x x x x x 1   2   3  
4  với K là một hằng số nguyên.
Chinh phục olympic toán| 21
Bồi dưỡng học sinh giỏi Điều này đẫn đến 2
  Px  1  K. x x x x x x x x 5 
 5 1 5 2  5 3  5 4 .
Điều này lại không thể. Như vậy không tồn tại đa thức thỏa yêu cầu bài toán.
Câu 5. Chứng minh n
x x  1 bất khả quy trên Zx,n  2 Lời giải
Bổ đề. Với mọi nghiệm phức z của đa thức    n P x
x x  1 chúng ta có bất đẳng thức sau.  1  1 2 Re z      1 2  z z
Chứng minh. Ta đặt  . it
z r e , bất đẳng thức tương đương   r t 2 1 2 cos
r  1  0 và do nó 2n 2
là nghiệm của P x nên 2n 2 2n 2
r z z  1  1  2r cost r  1  2r cost r r
Bất đẳng thức trở thành  2n 2 r r  2
r  1  0 (đúng) C Ọ
Giả sử P x  f xg x trong đó deg f  1 và f , g Zx , gọi các nghiệm phức của P x H k  1  k 1 z k i     ÁN là z , z , .  z 2 1 2
n , sử dụng bổ đề, ta có 2 i1 
zi i1 zi U TO
Trong đó ta đã giả sử f x  
k xxi. Ệ i1 LI Lại theo Viete ta có  k z f i 0  1. TƯ i1 1 VÀ
Theo bất đẳng thức AM GM ta có  kk  0 . Í 2 i1 zi CH  1  P Từ đó kéo theo  k Rez i   0 . Ạ i1  zi  T  1 
Mặc khác f monic và hệ số nguyên nên  k Rez i   1 . i1  zi    1 
Lý luận tương tự với g , cộng lại thu được Re   nz i   2.   i  1  zi   1  Theo Viete, ta lại có  nz i   1 i1  zi
Từ đây suy ra mâu thuẫn, vậy ta có P bất khả quy trên x .
Câu 6. Cho đa thức P là đa thức hệ số và tồn tại số nguyên dương m sao cho
P 1 ; P 2 ;...; P m không chia hết cho m . Chứng minh rằng P a  0 với mọi a  . Beijing 1967 Lời giải
22 | Tạp chí và tư liệu toán học Đa thức và số học
Giả sử tồn tại a  sao cho P a  0 .
Ta viết a dưới dạng a qm n với q, n là các số nguyên và 0  n m .
Trường hợp 1. Với n  0 . Khi đó qm Pm  Pa  Pm qm mm .
Điều này dẫn đến P m chia hết cho m (mâu thuẫn).
Trường hợp 2. Với 1  n m . Hiển nhiên a1;2;...; 
m nên q  0 .
Khi đó P n  P a qm  P aqm .
Điều này dẫn đến P nm (mâu thuẫn giả thiết).
Vậy bài toán được chứng minh.
Câu 7. Cho P xn n1  a x a x
 a x a
a , a ,, a n n1 1 0 là đa thức với 0 1 n là các số
nguyên dương. Đặt P x P x P x P P x k kk1  1     và     với 1 . Tồn tại hay không
M  0 sao cho với m M ta có | m P m P m     ÁN
S.T.E.M.S Cat A/B P4 O
Lời giải – AoPS IC T
Bổ đề. Nếu Q x là đa thức với hệ số thực thì khi đó: P
x y modm  Qx  Qymodm YM
Ta sẽ chứng minh M không tồn tại. Giả sử điều ngược lại rằng M tồn tại. OL m  0 mod m Pm  P m P m P m  0 mod  C Do 
, áp dụng bổ đề trên ta được     . Ụ
Vì vậy m M, | m PP m 0   . PH
Gọi q là số nguyên tố lớn hơn max{M, a } 0 . NH I Đặt t | q P 0 t t P q
q là số nguyên dương nhỏ nhất sao cho t   , như vậy tồn tại và q   vì q q CH
các điều kiện ở trên.
Yêu cầu 1. Nếu | q P 0 t r r  thì q|
Chứng minh. Ta dùng chung ý tưởng trong việc chứng minh ord x| n n  
Chứng minh phản chứng. Đặt r t x y y t q
với 0   q , khi đó: 0  P P P P q r 0   rt
q t  0  q
rt 0mod  q
Lặp lại các thao tác trên, ta thu được P q t
y  0  0 mod  , mâu thuẫn với điều tối thiểu của q
Yêu cầu 2. t |P 1 q
Chứng minh. Từ các điều kiện đã cho và từ yêu cầu 1, t |P q q t q   suy ra được q . Mặt khác | q a q a  0 0 – vô lý vì 0 .
Vậy gcdt ,q  qk  1 modt q
1 dẫn đến tồn tại số tự nhiên k sao cho  q
Từ những điều kiện trên ta có | q q | k P 0 P qk   
Chinh phục olympic toán| 23
Bồi dưỡng học sinh giỏi
Từ yêu cầu 1 suy ra được rằng t |P qk
P 1  P qk modt q
  , nhưng theo bổ đề ta có     q  Vậy t |P 1 q . Quay trở lại bài toán
Xét các số P 0 , P 0 ,P 0 1   2   P 1    
. Ta chọn số nguyên tố p đủ lớn sao cho p không chia
hết cho tất cả các số trên và p  maxM, P 1
Khi đó, theo cách chọn p , t P p 1 .
Nhưng theo yêu cầu 2 thì t |P 1 P 1  0 q
, điều này dẫn tới  
, vô lý vì tất cả các hệ số đều dương.
Vậy M không tồn tại.
Câu 8. Cho đa thức P x là đa thức hệ số nguyên với bậc n  2 . Chứng minh rằng đa
P P x   x thức  
có nhiều nhất n nghiệm. C Ọ
Romani TST – Gh. Eckstein H Lời giải
Ta chứng minh bằng phản chứng. ÁN
Giả sử tồn tại các số nguyên a a  ...  a
P P a a i  1 2 n1 sao cho  
i với mọi i . U TO
Theo tính chất trên ta có: Ệ LI
a a P a P a P P aP P a
a a   i j n j i |
j  i|   j   i j i; 1 1 TƯ
Điều đó kéo theo P a P a a a j   i j
i . Từ đó ta được: n n n VÀ Í P  a              P a P a   P a a a a a P a P a i 1   i
n 1  1 n 1 1  i 1 i
i 1  i i1 i1 i1 CH
Như vậy tất cả các giá trị P aA BP a i 1 
i  có cùng dấu, nên tồn tại , sao cho: P Ạ
Pa a A i   n
P a a B i   n i i ; 1, 2,..., 1 hoặc  i i ; 1,2,..., 1 . T
Tuy nhiên nhiên điều trên là không thể vì deg P x  x  n không thể nhận cùng một giá
trị nào đó với n  1 giá trị khác nhau của x . Bài toán được chứng minh.
Nhận xét. Với P x là đa thức có bậc là n  1 .
Chứng minh rằng phương trình P P...Px  x có nhiều nhất n nghiệm. k IMO – 2006
Câu 9. Tìm tất cả các đa thức P hệ số nguyên sao cho  |2n P n
 1 với mọi số nguyên dương n . Polish Olympiad Lời giải
24 | Tạp chí và tư liệu toán học Đa thức và số học
Dễ dàng kiểm tra được rằng nếu P là đa thức hằng thì P  1; 1.
Ta giả sử P khác đa thức hằng. Không mất tính tổng quát, ta có thể giả sử hệ số cao nhất
của đa thức P x là số nguyên dương.
Xét n sao cho P n  1 và xét p là số nguyên tố trong phân tích của P n . Khi đó   2n p P n
 1 và theo tính chất bên trên, ta được:          2n        p p n p n P n p P n p P n p  1
Điều đó ta có p 2 .n2p  2p  2p  1  2p n p p
 2  1  2  1  p 2  1 
Điều trên là trái với định lý Fermat.
Câu 10. Gọi x , x ,, x x x   x 0 1
n là các số nguyên thỏa mãn 0 1
n . Chứng minh rằng n!
một trong các số P x , P x , P x P x 0   1
n không nhỏ hơn với   là đa thức bậc n. ÁN 2n O
Đề xuất 1977 IMO – Vietnam Lời giải
IC TP Định f , f ,, f
x , x ,, x 0 1 n bằng 0 1 n . YM f x
Theo công thức nội suy Lagrange ta có P x  
n Pxii   i0 f x i i  OL C
Vì cả hai vế là đa thức bậc n và bằng nhau tại các giá trị x , x ,, x . Ụ 0 1 n Px  PH
So sánh các hệ số của n
x ta được 1  n i i0 f x i i
NH I Do x ,x ,,x 0 1
n là các số nguyên, giảm nghiêm ngặt, ta có in CH nf x x x x
x i n i 1 ! !  i i  1    j i   j i   j0 ji1 n! i
Đặt giá trị lớn nhất của P x , P x , P x P x 0   1
n là  k  . Px Px  n 2n n n P x i kk
Theo bất đẳng thức tam giác ta được 1       i0|| f x n i n i i | ! i0   ! n Vậy P x  !  k . 2n
Câu 11. Giả sử các đa thức P x ,Q x , R x và S x thỏa mãn: P 5
x   xQ 5 x  2  x R 5 x    4 3 2
x x x x  1Sx
Chứng minh rằng khi đó đa thức P x chia hết cho x  1 . USAMO – 1976 Lời giải
Chinh phục olympic toán| 25
Bồi dưỡng học sinh giỏi
Giả sử S xn n1  s x s     x s x s n n ... 1 1 0 . Khi đó:
x 1P 5
x   xx  1Q 5 x  2
x x  1 R 5 x    5
x  1Sx
 x  1 P 5
x   xx  1Q 5 x  2
x x  1 R 5 x    5
x  1 S x S x   1   2   n với S x 5 5
s s x ... ms x ;m
S x S x S x 1 0 5 5m  và 2  
  1  . Khi đó ta được: 5    xP 5
x   xx  1Q 5 x  2
x x  1R 5 x   5
x  1S x   5
x  1S x  P 5 x 2 1 
Vì vế phải của đẳng thức trên là đa thức mũ bội 5 trong khi vế trái thì không.
Điều này xảy ra khi và chỉ khi cả hai vế là hai đa thức có tất cả các hệ số là 0 .
Điều đó dẫn đến P  5 x    5
x  1S x P 1  0
P x x  1 .G x G x 1     . Do đó       với   là một đa thức nào đó.
Câu 12. Tìm tất cả các đa thức P có hệ số nguyên sao cho với mọi số nguyên tố p và C Ọ
mọi cặp số tự nhiên u, v thỏa mãn P uv  1 thì ta có P P uP v  1 . H Iran TST 2009 ÁN Lời giải
Ta xét các trường hợp sau. U TOỆ
Trường hợp 1. Với P là đa thức hằng. Dễ dàng kiểm tra được Px  1; Px  1  . LI   
Trường hợp 2. Gọi n  deg P  1 . Ta đặt G xn 1
x P  thì Gx cũng là đa thức hệ TƯ  x  số nguyên. VÀ Í
Từ giả thiết bài toán, ta có được p PuP  1
u   1 với mọi số nguyên dương u sao CH
cho u, p  1 và trong đó 1
u là chỉ số nguyên dương v sao cho uv  1mod p. P Ạ n 1  n T
Từ đó ta được p u PuPu   u  hay nói cách khác      n p P u G u u với mọi số
nguyên dương u sao cho u, p  1 .
Ta cố định u và lấy p đủ lớn, thì tính chất trên chỉ đúng khi và chỉ khi     n
P u G u u . Hơn nữa GxPx là các đa thức nên điều đó kéo theo     n
G x H x x . Mà deg P n nên G là đa thức hằng. Tức là   n
P x a x , bằng cách
thử lại ta được a  1; a  1  .
Vậy ta có bốn hàm đa thức thỏa mãn    1;    1  ;   n  ;   n P x P x P x
x P x  x .
Câu 13. Tìm tất cả các đa thức P hệ số nguyên sao cho với mọi số tự nhiên a, b,c ta luôn
a b c P a  P b  P c . Lời giải
26 | Tạp chí và tư liệu toán học Đa thức và số học
Giả sử P xn n1  a x a    a   x a x a n n ... 1 1 0 với i .
Với ba số tự nhiên a, b,c  1 tùy ý, ta có 2P 0  P c  3a modc 0   Tức c 3a a  0
0 , mà c là số tự nhiên từ ý nên điều trên đúng khi và chỉ khi 0 . Từ đó bằng
cách cho c  0 ta được tính chất a  |
b Pa  Pb đúng với mọi a,b  .
Dễ dàng chứng minh được tất cả các hệ số của đơn thức bậc chẵn đều bằng 0 . Do đó:
a b c|Pc Pa b  
Kết hợp với giả thiết của bài toán, ta được P a  P b  P a b  0mod a b c
P là đa thức và a, b,c là các số tự nhiên tùy ý nên P a  P b  P a b .
Tức P cộng tính, một lần nữa vì P là đa thức nên ta được P x  k.x . Thử lại ta được kết luận cho bài toán. ÁN
Câu 14. Cho m, n là các số nguyên dương. Chứng minh rằng n m khi và chỉ khi O
P xQx , trong đó Px ;Qx là các đa thức hệ số nguyên được xác định: IC T Pxn1 1 n2 n1  x C x  C n ... P n Qxm1 1 m2 m1  xC x  C m ... m YM Lời giải OL
Theo các định nghĩa của hai đa thức ta được: C Ụ
     1n 1;     1m xP x x xQ x x  1 PH n m n m m
Do đó P xQx  x  1  1 x  1  1     
x1 1,x1 1x1 1 NH I m,n m
 x  1   1  x  1  1  m,n  m n m CH
Bài toán được chứng minh.
Câu 15. Tìm tất cả các đa thức P hệ số nguyên thỏa mãn tính chất: Với m, n là hai số
nguyên tố cùng nhau thì hai số P m ; P n cũng là hai số nguyên tố cùng nhau. Iran TST Lời giải
Dễ thấy P không thể là đa thức hằng.
Giả sử với p là số nguyên tố đủ lớn sao cho p không là ước của P p .
Khi đó p, p P p  1 do đó P p và P p P p là hai số nguyên tố cùng nhau.
Tuy nhiên ta thấy P p  P
 p  p p |  P
  p Pp  Pp  Pp|P
pPp
Như vậy, để đúng với giả sử bên trên ta cần P p  1 hoặc P p  0 .
Điều này là không thể vì P là đa thức khác đa thức hằng và p là số nguyên tố đủ lớn.
Tóm lại, với p là số nguyên tố đủ lớn thì |
p P p , theo tính chất bên trên, ta lại có:
Chinh phục olympic toán| 27
Bồi dưỡng học sinh giỏi
p p  0|P p  P0  | p P 0
Mặt khác p là số nguyên tố đủ lớn tùy ý nên P 0  0 .
Từ đó ta được P x  x.G x với G x là đa thức hệ số nguyên thỏa mãn tính chất giống
như P x nhưng có bậc nhỏ hơn.
Thực hiện tương tự như vậy kết hợp với phương pháp quy nạp theo bậc của đa thức ta được:   n
P x  x với * n  .
Câu 16. Cho P x ,Q x  x là hai đa thức monic bất khả quy thỏa mãn với n đủ lớn
thì P n ,Q n có cùng tập ước nguyên tố. Chứng minh rằng P Q . Lời giải
Ta chứng minh bằng phản chứng bằng cách giả sử P Q . Khi đó P,Q là hai đa thức C
nguyên tố cùng nhau do chúng đều bất khả quy và là đa thức monic.
Theo định lí Bézout thì tồn tại hai đa thức f , g có hệ số nguyên và số nguyên dương a sao H
cho P x. f x  Q x.g x  a . Lại theo định lí Schur thì tồn tại vô hạn số nguyên tố p sao ÁN
cho ứng với mỗi số p đang xét thì ta có tồn tại số nguyên n sao cho p P n . Nhưng vì
P n ,Qn có cùng tập ước nguyên tố nên ta có p a . Như vậy số a có chứa vô hạn các U TOỆ
ước nguyên tố của nó là điều vô lý. Vậy Q P . LI TƯ
Câu 17. Gọi đa thức P x  x khác đa thức hằng và gọi n, k là hai số nguyên dương. VÀ
Chứng minh rằng tồn tại số nguyên dương a sao cho mỗi f a , f a  1 ,..., f a n  1 Í
có ít nhất k ước nguyên tố phân biệt. CH P
Bulgaria Olympiad ẠT Lời giải
Bài này gợi ta nghĩ đến định lí thặng dư Trung hoa.
Theo định lí Schur tồn tại vô hạn số nguyên tố p sao cho ứng với mỗi p sẽ tồn tại số
nguyên m sao cho p P m . Ta gọi A là tập số nguyên tố đó thì A   .
Gọi p 1  i n,1  j k ij
 là các số nguyên tố thuộc tập A.
Ta cố định mỗi i . Theo định lí Schur thì tồn tại số x P x p ij 0 mod ij sao cho    ij  .
Theo định lí thặng dư Trung Hoa thì cứ mỗi i cố định sẽ tồn tại số a sao cho
a i  1  x p j k
ij mod ij  với
1; . Khi đó với mỗi i cố định thì ta đều có
f a i  1  f x p j k ij  0mod ij  với 1, .
Nên f a i  1 chia hết cho tất cả các số nguyên tố p j  1, k i n ij  với mọi số 1; .
28 | Tạp chí và tư liệu toán học Đa thức và số học
Câu 18. Cho đa thức P x ,Q x là hai đa thức có hệ số nguyên nguyên tố cùng nhau.
Đặt a P n Q n a n
  ,  . Chứng minh dãy  n tuần hoàn. Lời giải
Theo đính lý Bézuot tồn tại hai đa thức R x ,Sx và hằng số a nguyên thỏa mãn
Px.Rx  Qx.Sx  a, x
  . Suy ra Pn.Rn Qn.Sn  a, n   .
Từ giả thiết a P n Q n a a na n
  ,  , ta suy ra , n
. Do tập các ước là hữu hạn nên các số n
lặp lại. Ta cần chứng minh là a   a n a n .
Ta có P a a P a a
a P n a a a P n a n , n
n mod . Mà   n n  .
Hoàn toàn tương tự ta có a Q n a a a n   n na .
Mặt khác ta lại có P a a P a a a    P n a a a a P n n a , n
n mod . Mà   n a n a   .
Lập luận tương tự ta có a a   aQ n a a n a
  . Do đó n a n. Từ đó ta có n a n . ÁN O
Câu 19. Với hai đa thức có hệ số nguyên p x ,q x ta viết p x  q xmod 2 nếu IC T
px  q x có tất cả các hệ số đều chia hết cho 2. Cho dãy đa thức p x n   thỏa mãn P
p x p x  1 p x p x xp x nnn 1 1   2   và 2   1  
   , với mọi n là số tự nhiên. YM Chứng minh rằng p x n    1mod2 . OL 2 C Lời giải
Ta quy ước viết p x  p ,n  1, 2, 3 n n PH
Trước hết ta xét ppxppxpxpx pxp nnnnnn 1 4 3 2  2 1  2   n2 n1 NH 2 2 I
 x  1 p x p xp x p x p p x p nnn 2 1 nn nn mod 2 * 2  2    2 2    2 p  CH
Phần tiếp theo ta chứng minh p n n mod 2 2   với mọi n.
Thật vậy với n  1 ta có p p  1 2 1 nên khẳng định đúng.
Giả sử khẳng định đúng với n k  1 .
Khi đó kết hợp với * ta có 2 pp x p kk k mod 2 2 2 2 2 2    p x p
p x p p xpp xpp k k mod 2  k k
2 k k mod2  k k 2 2 2 2 2 2 2 2 k mod 2 1 1 1 1   n Như vậy 2 4 2 p p      p p p n n n n 1 mod 2 1 1 2 1  . 2 2.2 2 2
Câu 20. Giả sử p x p x p n
 1 là hai số tự nhiên lớn hơn 1 và n   . 2 n
Dãy số vô hạn n  1, 2, 3... được xác định như sau p          p xp p xp xp x p xp n n n n n n 1 4 3 2  2 1  2   n 2 n1
Chứng minh rằng trong dãy số nói trên chứa vô hạn số đôi một nguyên tố cùng nhau. Lời giải
Chinh phục olympic toán| 29
Bồi dưỡng học sinh giỏi
Để ý là a  1 nên a, a  1  d  1.
Giả sử từ dãy x  1 p    x p xp n 2  n 2
n  ta trích ra dãy con hữu hạn 2x  1 2 2 px p px p nn nn mod2 2 2  
Mà hai số bất kì trong dãy này thì nguyên tố cùng nhau, do dãy ban đầu vô hạn nên luôn trích được như vậy. Đặt 2 p p n  2n
n . Xét n số sau
1 . Khi đó luôn tồn tại hai số đồng dư modulo q.
Tức là tồn tại hai số n k  1 sao cho 2 p      p x p p x p 2k 2 2k 2k2 . Do đó 2 2 2 k k1 . Do 2 2 2
p x p   p xppp xp k k 2 1 k k1 nên ta suy ra  k k 2 2
k1 , nên ta cũng suy ra  t
ax ,q  13
Từ 1 ,3 ta có st  1mod  st x q xlq  1, l   . Xét số u
a lq   a   alq i  1 1 14 . k1 C k Ọ Vì u
alq   al u u u j   1, 2,..., k i , .  i 1, i 1 .
nên  k 1 j   i 1 k 1 j H j1
Hệ thức 5 chứng tỏ ta có thể bổ sung vào dãy u u u i , i , ...,
các bộ số mới mà bộ số này ÁN 1 2 ik
vẫn thỏa mãn hai số bất kì nguyên tố cùng nhau. Chứng tỏ có vô hạn số như vậy. U TO Ệ
Câu 21. Với mỗi số tự nhiên n , ta kí hiệu f n là tổng các chữ số của nó biểu diễn trong LI
hệ thập phân. Ta xây dựng dãy số như sau TƯ
u n f n ;u n f f n ;...;u n f f f n k ... ... 1     2             VÀ Í k
Chứng minh rằng với mọi số tự nhiên n , tồn tại số tự nhiên d sao cho CH    P u n u n k d k   d  , 1 . ẠT Lời giải
a) Giả sử rằng n a a ...a a  0 1 2
n là số được biểu diễn trong hệ thập phân với 1 . Khi đó ta có biểu diễn p1 p2 n a .10  a .10  ... a   .10 a 1 2 p 1 p .
Ta luôn có bất đẳng thức n a a  ...  a f n 1 2 n   .
Dấu đẳng thức xảy ra khi và chỉ khi số có 1 chữ số.
Ta thấy rằng dãy un là dãy giảm thực sự cho đến khi ta thu được số có 1 chữ số thì dãy
trở thành dãy dừng bởi vì f n  f f n  ...  f f ... f n . Gọi d là số có một chữ số ấy.
Khi đó ta có u n u n k   d k   d  , 1 .
30 | Tạp chí và tư liệu toán học Đa thức và số học
b) Kí hiệu S x f x k k
 :   là tập hợp các số x mà trong biểu diễn thập phân thì tập S   S S k
vì chẳng hạn số 11...1 thuộc k .Với mỗi số tự nhiên k thì tập k là hữu hạn nên k
trong mỗi tập Sk đều tồn tại số bé nhất.
Câu 22. Cho abc là một số nguyên tố.
Chứng minh rằng phương trình 2
ax bx c  0 1 không có nghiệm hữu tỉ. Lời giải
Do abc là nguyên tố nên a  0,c  0 . Ta chứng minh bằng phản chứng.
Giả sử phương trình có nghiệm hữu tỉ nên 2 2
  b  4ac d ,d  . Do đó 2 2
b d  4ac  0  b d . 2 2
Mặt khác ta có a abc aa
b c   a b   2
b ac   a b 2 4 . 4 100 10 20 4 20  d
 20a b d20a b d . ÁN
abc là số nguyên tố nên suy ra abc là ước của 20a b d hoặc 20a b d . O
Tuy nhiên ta có 20a b d  100a b d  100a  10b d abc .
IC TP Tương tự 20abd100abd100a10bdabc . YM
Như vậy ta suy ra điều vô lí.
Vậy phương trình không thể có nghiệm hữu tỉ. OL C Ụ
Câu 23. Cho đa thức P x   2 x   2 x   2 2
3 x  6 . Chứng minh rằng với mọi số nguyên PH
tố p đều tìm được số nguyên dương n sao cho P np . NH I Lời giải
Rõ ràng với p  2 thì ta tìm được n  2 thỏa mãn bài toán. CH
Với p  3 ta tìm được n  3 cũng thỏa mãn.
Bây giờ ta xét các số nguyên tố p  3 . p1
Trước hết ta có nhận xét là nếu a, p  1 2
n  amod p , . n thì 2 a  1
 mod p. Chứng minh.
Với mỗi k 1, 2,..., p 1 thì tồn tại duy nhất một số k '1, 2,..., p   1 sao cho
k.k'  amod p . p  1 Vì 2
n amod p , n
 nên k k' . Ta để ý là p  1 là số chẵn nên ta sẽ có được cặp 2
tương ứng giữa k với k ' . p1
Từ đó ta suy ra  p       k k  2 1 ! 1.1' 2.2' ... . '  a
modp . Ở đây ta hiểu 1' là số tương
ứng với 1 để 1.1'  amod p .
Chinh phục olympic toán| 31
Bồi dưỡng học sinh giỏi p1
Mặt khác theo định lí Wilson ta có  p  1!  1
 mod p . Vậy nên ta có 2 a  1  mod p.
Nhận xét được chứng minh.
Trở lại với bài toán. Giả sử P n không chia hết cho p với mọi n .
P n   2 n   2 n   2 2 3 n  6 nên 2
n không chia hết cho 2; 3;6 . p1 p1 p1
Theo nhận xét ở trên ta suy ra 2    p 2    p 2 2 1 mod ;3 1 mod ;6  1  mod p . p1 p1 p1 Nhưng vì 2    p 2    p 2 2 1 mod ;3 1 mod  6
 1mod p là điều mâu thuẫn.
Chứng tỏ rằng giả thiết phản chứng là sai.
Câu 24. Chứng minh rằng với mọi số nguyên p ta có thể tìm được số nguyên dương
f x sao cho n không phải là số chính phương. C Lời giải f n n p p
f x b b A H Đặt   . Nếu
1 là số chính phương thì 1 hoặc   , . * n     ÁN
Giả sử tồn tại bộ số nguyên n N ,p
1 p 1 b sao cho với mọi số nguyên dương f x thì *
n N đều là số chính phương. Khi đó q đều là các số chính phương. Ta có U TOỆ
f n ; f n q  f n n q n  q f nq 1 LI
Mặt khác do tính chất của số chính phương nên ta suy ra f n qq . TƯ
Kết hợp với 1 ta suy ra f 2  f 4  0mod 4 . q
Suy ra p q,p  1 q 2 . Í
Bây giờ ta lại xét q
p p modq . CH P
Lập luận như trên ta suy ra q  1   q p
p p  p  1 q 3. ẠT
Kết hợp 2 ,3 ta suy ra * n  là điều vô lí.
Chứng tỏ giả thiết phản chứng là sai.
Câu 25. Cho đa thức P x 2017 1000  xx
 1 . Tồn tại hay không các số tự nhiên a , a ,... 1 2 , a P a P a a ai . 2018 sao cho tích
   j i j với mọi i j . Lời giải
Giả sử tồn tại các số a , a ,..., aP a P a a a i . 1 2
2018 sao cho với mọi i
j ta có    j i j .
Dễ thấy rằng a P aP a ai ,
i  1. Từ đó suy ra  i j và với mọi i j . Ta suy ra  2017 1000 aa  1 a a a i j i , j 1 i i
j , từ đó suy ra   với mọi .
Không mất tính tổng quát ta có thể giả sử rằng a a  ...  a 1 2 2018 .
32 | Tạp chí và tư liệu toán học Đa thức và số học Từ P a a P a a .a ...aa i j ta suy ra rằng   2017 1 2 3 2018 1 .
Nhưng từ P x ta suy ra P a  2017  a 1 1 là điều mâu thuẫn.
Câu 26. Với mọi số tự nhiên m, n , chứng minh rằng n! chia hết cho m khi và chỉ khi n
tồn tại đa thức hệ số nguyên f xk  a x
a , a ,..., a m m f j n , 1, k thỏa mãn  0 1    với k0
mọi j nguyên dương. Lời giải
Ta chứng minh tính hai chiều
Giả sử ta có m n! . Ta xét đa thức sau f x  x  1x  2...x n thì f là đa thức với hệ
số nguyên. Ta cũng có f j  n! n C m n! f j m nj . Do
  nên ta có   . Mặt khác do hệ số bậc
cao nhất của f là 1 nên ta có điều kiện a , a ,..., a m n , 1 0 1 
được thỏa mãn. Như vậy tồn tại ÁN O
đa thức f x  x  1x  2...x n . n IC T
Ngược lại giả sử ta có tồn tại đa thức f xk
 a x hệ số nguyên thỏa mãn điều kiện là P k k0 YM
a ,a ,...,a m m f j n , 1, 0 1   . OL
Ta xét dạng của f f x  b b x b x x  1  ...  b x x x x n n 1 2 ... 1 0 1 2        C n
b  b x x 1 ... x k 1 0 k     . PH k1
Với mỗi đa thức f hoàn toàn xác định thì các số b k n
k xác định duy nhất với mọi số 0, .
NH I Tiếp theo ta có nhận xét là b ,b ,...,b a a a n , ,..., 0 1   0 1 n  . CH
Thật vậy. Ta giả p là một ước của b ,b ,...,b f j 0 1
n  , thì khi đó p là ước của   với mọi số
nguyên j . Do đó khi và chỉ khi p là một ước của a , a ,..., a 0 1 n  . Do đó ta có
b ,b ,...,b a a a n , ,..., 0 1   0 1 n  .
Do m f j nên f j  0mod m với mọi j  0,1, 2,..., n . Nên đặc biệt ta có
b f 0  0 modm
b b f 1  0 modm b  0 modm 0     ; 0 1     1   .
Tương tự f 2  b  2b  2b  0 mod m  2b  0 mod m 0 1 2   2   .
Bằng cách tương tự ta có khẳng định là j!.b  0 mod m j  .
Do b , b ,..., b m a a a m
c ,c ,...,c D n , n , , ,..., n , 1 0 1   0 1 
nên tồn tại các số nguyên 0 1 sao cho
c b c b  ...  c b Dm   n c b c b   c b n Dm n n n 1 ! ... n n ! ! 1 0 0 1 1  0 0 1 1    .
Nhưng do j!.b  0 mod m 1 n m j
 nên từ   ta suy ra ! (ở bên vế phải).
Chinh phục olympic toán| 33
Bồi dưỡng học sinh giỏi
Câu 27. Cho đa thức f x 5 4 3 2
 2009x x x x  2006x  1. Chứng minh rằng với n
số nguyên tùy ý thì các số f n , f f n ,..., f ... f n... đôi một nguyên tố cùng nhau. Lời giải
Nhận xét. Nếu a b mod q thì h a  h bmod q .
Đặt f x f f f x k k  
 ...  ... với f có mặt k lần với 1. Gọi pP x f n p k 0 mod n
k là một ước nguyên tố tùy ý của   thì ta có    k  Ta có P x f     n f f n f p k k 1 1 mod 2  1      n   ;    
Bằng quy nạp ta chứng minh được f n pP x f n nm   1mod  với mọi
  , tức là k   và f n m k
f n f n d k , m  . m
 nguyên tố cùng nhau với mọi số
. Vì nếu không như thế thì    
Gọi p là một ước nguyên tố của d thì p là ước chung của cả f n f n
k   , m   là mâu thuẫn
với chứng minh nêu trên. Ta hiểu là là khi mà k thay đổi thì số nguyên tố p được điều chỉnh C 2015 Ọ
thay đổi theo chỉ số k . Do k là số tùy nên ta có P x  x x   1 i
đôi một nguyên tố cùng i1 H nhau. ÁN 1 n 1    U TO
Câu 28. Gọi P x là đa thức bậc n thỏa mãn với k  0, 1,, n thì P k    Ệ  k  LI
Định đa thức P n  1 . TƯ
Đề xuất 1981 IMO – Romanian Lời giải Í 1 n 1   
k!n  1  k! CH
Với k  0,1,, n , đặt w k c k    k và  k n  P  1! ẠT nk n  1 !
Định các giá trị cho f , f ,, f f k k n k f n   k 1  !  ! k 1 0 1 n , ta được       và     (n  1  k) n f n k 1 n nk
Theo công thức nội suy Lagrange ta có P n  1    ck   kf k k    1 0 k0
Bằng 0 nếu n lẻ và bằng 1 nếu n chẵn.
Nhận xét. Trong một số bài toán về đa thức, ta đôi lúc sẽ bắt gặp một số công thức như nội
suy Lagrange, khai triển Taylor,... việc hiểu và vận dụng linh hoạt các công thức trên sẽ
giúp bài toán trở nên gọn gàng và ít rườm rà hơn.
34 | Tạp chí và tư liệu toán học Đa thức và số học
u  1990,u  1989,u  2000
Câu 29. Cho dãy un được xác định như sau 1 2 3  . u        u u u n n 19 n 9 n n 1991, 1,2... 3 2 1
a) Với mọi n gọi r u r
n là số dư của phép chia
n cho 1992. Chứng minh rằng dãy  n là một dãy tuần hoàn.
b) Chứng minh rằng tồn tại vô số số x của dãy un sao cho 1992 1994 1975 1945 1990 2 5x  5x  4x  8x  2x  11x  48 chia hết cho 1992. Lời giải
a) Đây là dãy truy hồi tuyến tính cấp ba. Gọi r u
n là số dư khi chia
n cho 1992. Ta thấy chỉ
có tối đa 1992 số dư khác nhau. Số bộ ba các số dư có kể thứ tự khác nhau có thể là 3 A . 1992
Xét các bộ ba số dư của phép chia un cho 1992 là ( vì ta cần truy hồi tới ba số hạng phía
trước nên ta xét các bộ ba số r ,r ,r ; r ,r ,r ; , , , ; r r r
r r r i , i , i
; i , i , i ; , , , 1 2 3   2 3 4   1 2   1 2 3  số các bộ ÁN O
số lập theo cách trên là vô hạn. Tuy nhiên chỉ có hữu hạn số dư ( tối đa là 1992 số) trong phép chia u A
n cho 1992 nên chỉ có hữu hạn các bộ ba khác nhau. Tối đa là 3 1992 bộ. Do đó
IC TP luôn tồn tại các số nguyên dương ,ms sao cho r r YM m ms   r r     r
r r   r r r m ms 1 m , m , m m s , m s , 1 2   1
ms2  , tức là 1 1   . OL r    r C m 2 ms2 Ụ
Như vậy có nghĩa là tồn tại k m,m  1,m  2 sao cho ta có r r    k k k s  2  , 1 PH
Ta sẽ chứng minh bằng quy nạp rằng r r    k k k s  2  , 1 . NH
Ta đã có r r với k m, k m  1, k m  2. I k ks
Giả sử ta có 2 đúng với m k p trong đó p m  2 . Ta xét k p  1 . CH Ta có u          u u u u   u u   u p s p 19 p s p 9 1 1 
  p s 1 p 1  p s 2 p2  19r        r r   r r   r p s p  9 p s p p s p 0 mod1992 1 1   2 2  . Như vậy ta có r    r p s 1 p1 .
Vậy ta có r r    k m k k s ,
Cuối cùng ta phải chứng minh khẳng định r r    k k k s , 1.
Nếu m  1 thì ta có r r 1 1s .
Nếu m  1 , ta phải chứng minh r r     k m k k s , 1 1 .
Từ cách xác định dãy ta truy ngược có u      u u u n 19 n 9 n 1991 3 2 1 n . Cho nên ta có u          u u   u u   u u u m s m m s m 19 m s m 9 1 1  2 2   1 1   m s m  r         r r   r r r m s m 19 m s m 9 m s m 0 2 2   1 1    .
Cứ thế sau m  1 bước lùi chỉ số ta thu được r r     k m k k s , 1 1.
Chinh phục olympic toán| 35
Bồi dưỡng học sinh giỏi
Kết hợp lại ta có r r    k k k s  2  , 1 . Như vậy dãy r s r  2   r  3 
n là dãy tuần hoàn với chu kì 1 ( tại vì 1 2 )
b) Đặt P x 1992 1994 1975 1945 1990 2  5x  5x  4x  8x  2x  11x  48. Ta thấy dãy u n
n là dãy tăng khi 2 .
Do đó ta cũng suy ra với chu kì s  1 ở trên thì ta có ( chu kỳ s  1 vì r r 2 1 ).
u u   u us s ... ks ... 2 k1s
Đây là dãy tăng thực sự nên có vô hạn phần tử.
Ta chỉ cần chứng minh P u k   ks  1992, 1.
Ta có từ cách xác định dãy ta suy ra p xq x .
Do đó p x  q x ( do p x  q x ) mà p x
p x p x  1 n   , lại do 1   2   nên suy ra p    x p x xp x n 2   n 1   n   . C
Vậy ta có điều phải chứng minh! Ọ H
Câu 30. Chứng minh rằng tồn tại hằng số dương c sao cho với mọi số nguyên dương n ÁN
và các số thực a , a ,, a
P x x a x a x a 1 2 n , nếu    1   2   n  thì P x n U TO
max   c max Pxx   0,2 x   0,1 Ệ LI
Lời giải – Lee Kai Seng i
Đặt S là giá trị lớn nhất của |P x|x 0,1 . Với i  0,1, 2, ,
n , đặt b in
f x  x b x b x b x b i 0   i1   i1   n  Í f x i CH
Bằng công thức nội suy Lagrange, với mọi số thực x ta có P x  
n Pbi   P i0 f b i i  ẠT
Với các giá trị w [0, 2], w b  2  b k  0, 1, 2, n k k , nên f wi n n n n nf   
 2 2  12  2   1 2 ! 2   n i i 2      n n i0  n n n!n
i! n i !
Lại có P b   S f b ii i    . n n n n f w n n i i  2  2  
Theo bất đẳng thức tam giác ta có P w   Pb S i        i0 f b i n i i i0   
n  2n 2n i
n  2n 2n  2n Suy ra 2n 4  
      2    2 n
i0  i  
n i0  i  n   n
Vậy max P w 4
 2 nS  16n max Px . w   0,2 x   0,  1
36 | Tạp chí và tư liệu toán học Đa thức và số học
Câu 31. Tìm tất cả các đa thức P x  x và  m  sao cho  2n m
P n là số chính
phương với mọi số nguyên dương n .
Nguyễn Song Minh Lời giải
Lời giải sau đây của thầy Nguyễn Song Minh – Admin Mathscope.
Giả sử P x và m là là đa thức và số nguyên dương thỏa mãn, ta có 2 nhận xét sau:
Nhận xét 1. Nếu p là ước nguyên tố lẻ của  2n m
P n thì |  p P n
Chứng minh. Ta có v m  2n P n p
   2 , theo bổ đề Fermat bé thì:  2n m Pn
npp1  m  2
Pn pp  1 mod p
npp1
Vì thế ta lại có v m P n p p p   2
   1  2, theo bổ đề tiếp tuyếnđịnh lý Euler ta
npp1 có  
       n   n m P n p p m P n
pp  P'n 2 0 2 1 2 2 1 mod p  . ÁN p P' n O Từ đây có   .
Nhận xét 2. Nếu p là ước nguyên tố lẻ của  2K m P n với  K  thì |  p P n .
IC TP Chứng minh. Theo định lý thặng dư Trung Hoa, sẽ tồn tại số nguyên dương N sao cho | 2N p m P N  YM
N K  mod p  1 và N n mod p , từ đây có  . OL
Do đó theo nhận xét 1 thì |
p P'N  nhưng P'x x nên từ N nmod p ta có: C Ụ p P'n PH
Quay lại bài toán, với mỗi  n
đủ lớn, sẽ có vố số số nguyên tố p có dạng 8k  3 đồng  2  NH
p  maxm, P n     1  I thời
  . Lúc này ta để ý rằng
nên 2 là căn nguyên thủy modulo p p  CH
khi đó sẽ tồn tại K sao cho  2K p m
Pn (ta chọn K  ind mP 2 
 với P là nghịch đảo
của P n theo mod p , theo nhận xét 2 thì p P 'n .
Do đó có vô số số nguyên tố p như thế, và giá trị của chúng có thể lớn tùy ý nên P'n  0
Từ đây ta thấy rằng phương trình P'x  0 có vô số nghiệm, nên P x  C (với C là hằng
số), từ đây ta dễ dàng chứng minh được P x  0 còn m là số chính phương.
Nhận xét. Bài toán trên yêu cầu ta phải vận dụng linh hoạt các tính chất số học của đa
thức, ngoài ra có một số bổ đề nhỏ ta cần phải quan tâm như bổ đề tiếp tuyến được phát
biểu và chứng minh ngắn gọn như sau.
Câu 32. Cho P x  x , p là số nguyên tố và x a mod p . Chứng minh rằng
Px  Pa  x aP'a 2 mod p
Bổ đề tiếp tuyến Lời giải
Chinh phục olympic toán| 37
Bồi dưỡng học sinh giỏi
Với mọi đa thức P x và a b nguyên thì a b P a  P b kP x0 
Nếu P x  x và  k  thì  x  0
, do tích của k số nguyên liên tiếp chia k!
hết cho k !, mà đạo hàm cấp k thì chứa các tích đó.
Khai triển Taylor của đa thức tại x x0 :   n   P x P x P x
P x Px x x x x   x n x 0 
   0   02 0 0  0   0 1! 2! n!
Từ những điều trên, xét x a pt với t  thì dễ dàng có:
Px  Px   P'x pt P' x pt2   Px   P'x pt  2 mod p 0 0 0 0 0  2 p
Nhận xét. Bổ đề trên là cơ sở cho bổ đề Hensel (Hensel lifting lemma) dùng để chứng minh
tồn tại hoặc đếm số nghiệm của phương trình đồng dư với modulo là lũy thừa của số C nguyên tố. Ọ H
Câu 33. Với p là số nguyên tố, đặt h x là đa thức có hệ số nguyên sao cho ÁN
h  h   h 2
0 , 1 , , p  1 là một hệ thặng dư đầy đủ modulo 2
p . Chứng minh rằng 3 U TO
h0 ,h1 ,,hp  1 là một hệ thặng dư đầy đủ modulo 3 p . Ệ LI Putnam 2008 B4 Lời giải
Dưới đây xin giới thiệu với bạn đọc lời giải chính thức của cuộc thi. VÀ
degh i Í h x
Ta xác định đa thức với khai triển Taylor h x y   i y i0 i! CH i P h x Ạ Trong khai triển này,
là đa thức theo biến x với hệ số nguyên. T i!
Với x  0,, p  1 , ta suy ra được h x p  h x  ph 'x 2 mod p
Điều này có thể suy ra được trực tiếp bằng cách sử dụng khai triển Nhị thức Newton
Vì thế ta giả sử h x và h x p phân biệt với modulo 2
p , ta kết luận được
h'x  0mod p. Do h ' là đa thức với hệ số nguyên, ta có h'x  h'x mpmod p với
mọi số nguyên m , vì thế h 'x  0mod p với mọi số nguyên x . Với 2
x  0,, p  1 và y  0,, p  1, ta viết được đa thức h dưới dạng: h 2
x yp   hx 2
p yh'x 3 mod p
Do đó h xh  2
x p   hx  p   2 , , ,
1 p  chạy qua tất cả các lớp giá trị dư của modulo 3 p ,
đồng dạng với h x theo modulo 2 p .
38 | Tạp chí và tư liệu toán học Đa thức và số học
Vì đa thức h x đã chạy qua các lớp giá trị dưtheo modulo 2
p , chứng minh này đã dẫn tới
h  h   h 3
0 , 1 , , p 1 phân biệt với modulo 3 p . Cách 2.
Trước hết ta chứng minh P 'x không chia hết cho p với mọi x nguyên.
Thật vậy giả sử tồn tại x P 'x  0 mod p , khi đó ta chọn y sao 0  y p
y x mod p . Theo cách chứng minh định lí Hensel ta có
P y  P x
p  Px p  Py  p P y
p  P y 2 ' ' mod . ' mod mod p . Mà ta lại có 2
0  y y p p  1 . Điều này mâu thuẩn với hệ P  P   P 2 0 , 1 , , p  1 là
hệ thặng dư đầy đủ modulo p.
Phần tiếp theo ta chứng minh hệ P  P   P 3 0 , 1 , ,
p  1 là một hệ thặng dư đầy đủ modulo 3 p . ÁN O
Giả sử tồn tại hai số 3
0  a b p P a  Pb 3 mod p . IC T Ta đặt 2 2
a a p a ;b b p b với  2
0  a ,b , a ,b p ;0  a,b p 0 0 1 1 . P 0 1 0 1
Ta cần chứng minh khi đó a b . YM
Từ cách đặt và giả sử ta có các điều kiện ta suy ra OL
Pa  Pa  2
mod p ,Pb  Pb  2
mod p ;Pa  Pb 2 mod p 0 0  C Ụ
Ta suy ra P a   P b  2 mod p a b 0 0 . Từ đó suy ra 0 0. PH
Lại theo phần chứng minh trong định lí Hensel ta có NH
0  Pa  Pb  Pa  2
p a P'a   Pb  2
p b P'b  2
  p P'a a b     3 mod p 0 1 0 0 1 0 0 1 1  I CH
a b  0 mod p P ' x p 1 1 
 , do ta đã chứng minh   .
Từ đó ta có a b P a  P b 3
mod p   a b . Ta có điều cần chứng minh. Nhận xét.
 Một cách tổng quát hơn, bằng cách chứng minh tương tự như trên ta có thể chỉ ra
rằng với số nguyên d, e  1 bất kỳ, đa thức h hoán vị cho các lớp giá trị dư modulo d
p khi và chỉ khi nó hoán vị cho các lớp giá trị dư modulo e
p . Lập luận trên được
thể hiện quen thuộc trong kết quả chứng minh của bổ đề Hensel.
 Khái niệm về lớp giá trị dư – Residue class – AoPS: Trong các bài toán số học về
modulo, phần còn lại của một số nguyên a theo modulo n là giá trị duy nhất của
0  r n  1 , vậy a kn r . Lớp giá trị dư là tập các số nguyên đồng dạng theo
modulo n với một vài giá trị nguyên dương n . Trong modulo n , có chính xác n
lớp giá trị dư phân biệt, tương ứng với n giá trị còn lại 0,1, 2, 3, n 2, n 1 . Mỗi
lớp giá trị dư chứa tất cả các số nguyên dạng kn r với r là phần dư tương ứng.
Chinh phục olympic toán| 39
Bồi dưỡng học sinh giỏi
Câu 34. Với số nguyên n  3 , đặt f x , g x là đa thức với hệ số thực sao cho các điểm
f 1,g1, f 2,g2,, f n,gn trong tập 2 là các đỉnh của đa giác n cạnh
theo thứ tự ngược chiều kim đồng hồ. Chứng minh rằng ít nhất một trong các đa thức f , g n  có bậc không nhỏ hơn 1 . Putnam 2008 A5 Lời giải
Ta biểu diễn các điểm trên dưới dạng đa thức hệ số phức P z  f z  ig z . Không khó
để chứng minh rằng deg P n  1 , khi đó một trong các hàm f , g phải có bậc không nhỏ hơn n  1 .
Thay P z bằng aP z  b với các số a, b  thích hợp, ta biến đổi các đỉnh của đa giác n  2i  cạnh trên thành dạng 2  , ,,n   n exp n n n với 
 . Dễ kiểm tra rằng không tồn tại đa  n  C n i   Ọ
thức P z có bậc không vượt quá
2 sao cho Pi  i n n với 1, , . H
Ta chứng minh rằng với mọi số phức t 0, 
1 , với mọi số nguyên m  1 , mọi đa thức ÁN
Qz sao cho    i Q i
t với i  1,, m có bậc không bé hơn m  1 .
Nếu deg Q d và có hệ số cao nhất là c, khi đó R z  Q z  1  tQ z có deg R d và có U TOỆ
hệ số cao nhất là 1  tc . Tuy nhiên, theo giả thiết trên, R z có các nghiệm phân biệt LI
1, 2,m  1 nên d m  1 . TƯ
Nhận xét. Ta có thể giải bài toán trên bằng cách sử dụng ma trận Vandermonde hoặc công
thức nội suy Lagrange để tính toán hệ số cao nhất của Q . VÀ Í CH p  1 P
Câu 35. Cho p là một số nguyên tố lẻ. Chứng minh rằng có ít nhất giá trị 2 ẠT p1
n0,1,2, p  
1 sao cho  k! k
n không chia hết cho p . k0 Putnam 2011 Lời giải p1
Đặt f x  k! k x F x F
p   là tập hợp các đa thức bậc p có hệ số thuộc trường p . k0
Ta phân hoạch F Z Z'
Z  aF ; f a  
0 ;Z'  aF ; f a  p p 0 p , trong đó  
Ta thấy f 0  1  0  Z'  Z  1. Đặt Z p r, Z'  r  1
Xét đa thức s x  x a  deg sx  r .  a Z'
Dưa vào cách ta định nghĩa thì đa thức s xf x triệt tiêu ( luôn bằng 0 ) tại mọi điểm của trường Fp .
40 | Tạp chí và tư liệu toán học Đa thức và số học
Do đó       p s x f x
x xhx trong đó deg h r  1 và thuộc F x
p[ ] , lưu ý đến bậc của hai
vế + định lý Fermat nhỏ. Trong những tính toán tiếp theo có sử dụng đến đạo hàm hình thức. Ta có d xp1 p2 p1 xf x d k1  x
k!x xk 1! kx  k! kx f x1 dx dx k0 k0 k1 2
x f x  xf x  f x 2 '
 1  x f 'x  x  1 f x  1  0
Lấy đạo hàm hình thức       p s x f x
x xhx ta được:
'       '    p s x f x s x f x
x xh'x  hx
Chú ý. Loại bỏ các số hạng triệt tiêu trên trường Fp
Từ các kết luận trên ta thấy 2
      p x s x f x x x 2 x h x 2
x h x sx 2 ' ' x f 'x ÁN   p x x 2 x h x 2 '
x hx  sxx  1 f x  1 O
  p   2 '  2 
  1 p x x x h x x h x x
x xhx  sx IC TP   p x x 2
x h x  x  hx 2 ' 1
x hx  sx YM
Ta thấy vế trái triệt tiêu trên   0 , p Z
x x triệt tiêu trên F 2 x h x s x p nên đa thức     OL
triệt tiêu trên z   0 . C Ụ  1 Ta thấy  1  deg  2 
     1   1  p r x h x s x Z p r r . PH 2
Câu 36. Có tồn tại hay không một dãy số thực và khác 0 là a ; a ;, a 1 2 n thỏa với mỗi NH I
n  thì đa thức a a x  n a x 0 1 n
có đúng n nghiệm trên . CH
Lời giải – Dark Templar – VMF. Diễn đàn toán học Việt Nam
Ta chứng minh dãy trên tồn tại bằng cách xây dựng nó bằng quy nạp.
Chọn a  1, a  1 n  0 1
sao cho thỏa mãn tính chất trên với 1 .
Đặt P x   n k a x
r r    r n k
với n nghiệm phân biệt là 1 2 n . k0 Chọn x r
x r ;rk n P x k k k ; 1; 1 0 1 . Cho  1  
 là giá trị mà n   đạt giá trị lớn nhất. Cho x r
P x  0;k n a n k 0; n n , có  
  và ta có thể chọn 0 sao cho n1 a xP x k n k
n ( k  ;  0; .
Nếu P x   a  a n n 0, đặt n1 n1
Nếu P x   aa a xP xP x P x n a x n n 0, đặt n1 , có k
n k  chứng tỏ rằng n  k n k  1 1 n1
luôn khác không và có cùng dấu với P x lim P x P x n k  và
1   có dấu khác n1  n  .  n x Do đó P x x ; xk n x k k ; 0; 1 n1   có 1 nghiệm thuộc  1  
 và 1 nghiệm lớn hơn n .
Chinh phục olympic toán| 41
Bồi dưỡng học sinh giỏi
Câu 37. Cho P x  x thỏa mãn P x là số chính phương với mọi x nguyên thì P x 2
Q x với Qx x . Lời giải 2
Ta chứng minh bằng phản chứng. Giả sử P x  Q x  p x p x p x 1   2  
k   với p x i   là
các đa thức bất khả quy bậc lớn hơn 1 Do p x p x
p ' x .p x p x
i   bất khả quy nên 1   và 1   2  
k   nguyên tố cùng nhau nên theo
định lý Bézout thì tồn tại hai đa thức R x ,S x với hệ số nguyên sao cho Rx 
p x S x p x p x p x n 1  
  1   2   k   (n )
Chọn p n nguyên tố sao cho tồn tại a để p ap a p a p a
1   chia hết cho p t hì 1   2   k  
không chia hết cho p , luôn tồn tại p theo Schur. Nếu p a v P a p  
1   không chia hết cho 2
p ta suy ra vô lý vì khi đó   lẻ. C 2 Ọ Nếu p a
p a p p a p p ' a ( mod p p a p 1   chia hết cho 2 p ta xét 1   1   1     thì 1   không H chia hết cho 2 p . ÁN
Chứng minh tương tự như trên ta suy ra mâu thuẫn Vậy P x 2  Q x U TOỆ Nhận xét. LI
 Bài toán có thể tổng quát cho trường hợp Px là lũy thừa bậc n của một số TƯ
nguyên với cách chứng minh hoàn toàn tương tự.
 Bằng bài toán nhỏ trên ta có thể giải bài toán dưới đây với lời giải ngắn gọn hơn. VÀ Í CH
Câu 38. ìm tất cả các đa thức số hệ số nguyên thỏa m n a b là số chính phương thì P
P a  P b cũng là số chính phương, trong đó a,b là các số tự nhiên. ẠT
Lời giải – Nguyễn Tiến Khải, Phạm Khoa Bằng – THPT chuyên KHTN
Bổ đề. Nếu P x  x là số chính phương với mọi x thì nó là bình phương của một đa thức khác
Quay trở lại bài toán nhận xét rằng a b chính phương thì a b 2
x chính phương.
Cố định a, b, ta xét đa thức Q x  P  2
ax   P 2 (
(bx  là số chính phương với vô số x nên là
bình phương của đa thức G x nào đó.
Đặt G x  m g x  n P x p x m và     n ,
Đồng nhất hệ số bậc cao nhất ta suy ra p n n a b g n  2 (  
nghĩa là với mọi a b chính
phương thì ta có điều trên
42 | Tạp chí và tư liệu toán học Đa thức và số học
Cặp 1, 0 có 1  0 chính phương suy ra với mọi a b số chính phương thì n n a b là số
chính phương với n  1
Với n  0 cho ta P x 2  2k . Với 2, 2  ta suy ra n n 1 2.2 2  
suy ra n lẻ. Chọn a, b  3,1 ta có n 2
3  1  u hay 3n  u  1u  1
Ta có 3, 2  1 và gcd u  1, u  1|2 , mà 3 nguyên tố nên u  1  1 hay u  2  n  1
Đặt P x  kx ta cần có k chính phương
Vậy các đa thức cần tìm: P x 2  k x Px 2 ;  2k .
Câu 39. Giả sử m, p là các số nguyên tố khác nhau. Chứng minh rằng nếu có một số tự m1 m2
nhiên x nào đó mà p là ước của P x  xx
 ... 1 thì ta có p  1 modm . ÁN Lời giải O
Giả sử p mk r 0  r m  1 , ta cần chỉ ra là r  1 thì khi đó p  1  mk . IC T m1 m2 m m P
Giả sử tồn tại số tự nhiên x sao cho xx
 ... 1 p p x 1  x  1mod p1  YM
Do đó x không chia hết cho p nên x, p 1. r1 x  1 mod p OL Ta sẽ chứng minh  . C     r1 x  1 mod p 2 Ụ Nếu p m
r p là số nguyên tố nên 
  đúng do định lí Fermat. p r PH
 Nếu p m . Ta có k
. Trong 1 ta nâng lũy bậc k hai vế , ta được m NH pr x
 1mod p * I p1 x  1 mod p CH
Nhưng theo định lí Fermat thì  . pr  
Do đó trừ vế theo vế ta suy ra xr 1 x     pr 1 1 0 mod  x
 0mod p do ta đ có * .
Cuối cùng ta chỉ ra r  1 . Ngược lại nếu r  1 . Khi đó m, r  1  1 vì m là số nguyên tố. m r1 m,r1
Do đó x  1, x  1    x
 1  x  1. Nhưng từ 1 ,2 ta có m p x  1 và r 1 p x   1 , do
đó p x  1  x  1mod p , điều này do ước chung lớn nhất thì chia hết cho mọi ước. Dẫn
đến P x  mmod p , điều này trái với giả thiết là p P x . Vậy r  1 nên p  1 mod m .
Chinh phục olympic toán| 43
Bồi dưỡng học sinh giỏi
Câu 40. Cho đa thức P x có bậc n và có n nghiệm phân biệt x , x ,, x 1 2 n . Chứng minh rằng: P' x P' x P' x 1   2   n a)    0 P'x P' x P' x 1   2   n 1 1 1 b) P     0 ' x P' x P' x 1   2   nLời giải
a) Ta viết đa thức P dưới dạng P x  x x
x x x x 1   2  
n  , giả sử rằng
x x   x 1 2 n .  1 1 1  1
Ta có P'x  P x  
  Px n 1 x x x x x x x x n i   1 2  1 i P x i  Do  n i  0,
1, nên theo định lý Rolle tồn tại C c c c
x c x c    c  Ọ , , , x n ; 1 2 1 1 1 2 2 n1 n H Sao cho P 'c i n i   0,  1,  1 2 ÁN 1  1 1 1  1
Lại có P''x  P'x  
  P'x n 3 x c x c x c x c n i   1 2 1  1 i U TOỆ Từ 1 ,2 suy ra: LI    nP' 1 1 1 1 c P c      P c   0 1   1  1 TƯ c x c x c x c x n i    1 1 1 2 1  1 1 i   1 1 1  n 1  VÀ
P'c P c      P c   0 2   2   2  Í  c x c x c x c x n i   2 1 2 2 2  1 2 i  CH  P   1 1 1  n 1 Ạ P'cP c P c n n      n      0 1   1  1 T  cx cx cx c x n n n n i    1 1 1 2 1  1 n1 i Do P c i n i   0,  1,  1 nên ta có:  1 1 1 n 1      0
c x c x c x c x n i  1 1 1 2 1 1 1  i  1 1 1 n 1       0 c x c x c x c x n i  2 1 2 2 2 1 2 i ...................    1 1 1 n 1      0
c x c x cx c x n n n n i   1 1 1 2 1 1 n1 i n P' x n 1 n 1 n i 1 Suy ra       ...   0 iP x c x c x c x i ii ii i  1 '  1 1 1 2 1 n1 i
44 | Tạp chí và tư liệu toán học Đa thức và số học
b) Xét phân tích P x  x x
x x x x x x nn  1   2     i  . i1 n Đặt P x x x P x P x i     n   j 
     i  .
j1, ji i1
Ngoài ra P x  0, j i; P x  0, i 1,    
   n P x   P x ,i n i j i i i i i 1, . P x
Xét đa thức Q x    n i  1 n  
có bậc không vượt quá 1 .
i1 P xi
Ta có i  1, n Q xi   0 .
Suy ra đa thức này có n nghiệm thực, tức Q x  0 và hệ số cao nhất của Q cũng bằng 0. 1 1 1 Do đó . P     0 ' x P' x P' x 1   2   n ÁN O
Câu 41. Cho f là một đa thức có hệ số hữu tỉ và bậc không nhỏ hơn 2, và d y an  chỉ
gồm các số hữu tỉ thỏa mãn f a   a n 1 
n với mọi số nguyên dương n. Chứng minh rằng
IC TP dãy an tuần hoàn. YM Lời giải f x OL
Để ý vì P x có bậc lớn hơn 2 nên lim  .  C x
 Ta sẽ chỉ ra dãy an  là bị chặn. PH
f có bậc không nhỏ hơn 2 nên f có tính chất là khi x đủ lớn thì f x   .  NH I f x
Điều đó có nghĩa là tồn tại số M  0 mà nếu x M và đồng thời lim   thì ta có CH x
f x  x M. Ta chọn được số M như vậy mà M a a  . 1 vì do 1 Ta sẽ chỉ ra là * a M n   n , .
Thật vậy. Giả sử phản chứng tồn tại số n để a M n .
Khi đó f a   a a     a M. n n n 1 n
Tiếp tục như vậy ta đi đến a M 1 là mâu thuẫn với
cách chọn ban đầu. Như vậy dãy an  bị chặn.
 Tiếp theo ta cần chỉ ra dãy an  chỉ nhận hữu hạn giá trị.
Để làm việc đó ta cần chỉ ra là tồn tại một số nguyên dương N để N.an là số nguyên với mọi .
n Như thế thì đi đến dãy Na a
n  chỉ có hữu hạn phần tử. Và đi đến dãy  n  cũng chỉ có hữu hạn phần tử.
Thật vậy. Gọi k là số thỏa mãn g x  k. f x là một đa thức có hệ số nguyên.
Đặt g x  k. f xs s1  b x b    bx b x b s s ... 1 1
0 thì s là các số nguyên.
Chinh phục olympic toán| 45
Bồi dưỡng học sinh giỏi
Gọi N là số nguyên dương để cho N.a  . 1
Khi đó ta thấy nếu Na Nan * . n thì 1   Chứng minh *.  x  Ta thấy Na f    a
n1 là nghiệm của đa thức
n . Ta tạo ra đa thức monic bằng cách xét đa  N
k N   x   thức h x . s   f  
a thì đa thức này nhận Na làm nghiệm nên nghiệm này s n b N     n1
phải là nghiệm nguyên. Vì vậy NaNa n . 1
Từ đó theo nguyên lý quy nạp ta có  n
nguyên với mọi số số nguyên dương .
n Lại vì a Na
n  là dãy bị chặn nên  n  cũng bị
chặn. Vì thế dãy Nan  chỉ có hữu hạn phần tử.
 Cuối cùng ta cần chỉ ra sự tồn tại của chu kỳ.
Giả sử dãy trên có tất cả m giá trị phân biệt. C Ọ
Xét các bộ gồm m  1 số của dãy là a a a i , i ,.., im. 1 H
Vì bộ này có tính thứ tự nên số các bộ khác nhau tối đa là m 1 m  . ÁN
Điều này có nghĩa là sẽ có một loại bộ xuất hiện vô hạn lần trong dãy.
Trong mỗi bộ m  1 số này luôn có hai số bằng nhau theo nguyên lý Dirichlet . U TO
Ta giả sử hai số đó là a a j
jk . Như thế luôn tồn tại k nguyên dương sao cho với j đủ lớn Ệ LI sao cho a a a a a aj jk . Từ j
jk ta suy ra i
ik với mọi i
j do cấu trúc của hàm số cho TƯ
ban đầu. Mặt khác vì ta luôn chọn được j đủ lớn để có được điều trên nên ta suy ra a a    i i i k , . VÀ Í CH P' 1   n P    P
Câu 42. Cho P x là đa thức bậc n với hệ số thực sao cho  1 khác 0 và P 1   2 ẠT
Chứng minh rằng P x luôn có ít nhất một nghiệm x x  1 0 sao cho 0 . Lời giải
Giả sử x , i n a i
1, là các nghiệm phức của đa thức P , khi đó ta xét với 0 :   n n P x P P x   n ax xi  '  1 ' 1   1        iP x i x x P x i i  1   1  1 1 1 i P' 1   n n n 1 n  1
1  1 n x i 1  Suy ra P            1
  2 2 i  x x x i ii i  1 1 1  2 1  2 1  i 1  x x x x x i 1   i 1  i 1 2   i 1   i 1
Theo tính chất của số phức   Re   ,i  1,n 2 2 x x x i 1  x i 1   i 1   i 1
46 | Tạp chí và tư liệu toán học Đa thức và số học P' 1   n P' 1   2 n 1  x i 1  2 Mặt khác P    R    n
   x   x  1   2 P  1   0 i 1 i 1 2 2 2   i1 x i 1  
Vậy ta có điều phải chứng minh.
Câu 43. Giả sử tồn tại đa thức hệ phức P,Q, R thỏa mãn a b c P Q
R a,b,c   . 1 1 1
Chứng minh rằng    1 a b c
The IMO Compendium Lời giải
Bổ đề. Nếu A, B,C là các cặp đa thức “nguyên tố cùng nhau” (coprime) theo dạng
A B C , khi đó bậc của mỗi đa thức nhỏ hơn số lượng các đa thức khác 0 của đa thức ABC . k l m ÁN a b c
Chứng minh. a đặt Ax  x p B x x q C x x r i i ,
     ii ,      ii . O i1 i1 i1 1  1  A x C xB x C x  1 IC T
Viết lại điều kiện A B C dưới dạng         , đạo hàm hai vế biểu P
thức theo biến x ta được: YM     k m l m 1   a c b c A x C x  i
i BxCx 1     
 i   i  OL
ix p x r x q x r i ii ii i   1 1   1 1 i  C Ụ Ax Ta thấy rằng
có thể viết được dưới dạng thương của hai đa thức có bậc không vượt Bx PH
quá k l m  1 , từ đây ta suy được rằng A B “nguyên tố cùng nhau”
NH I Áp dụng bổ đề trên với các đa thức a, b, c
P Q R có các bậc của chúng là CH
adeg P,bdeg Q,c deg R lần lượt nhỏ hơn deg P  deg Q  deg R , từ đó suy ra được rằng: 1 deg  P
a deg P  degQ  deg R
Cộng lần lượt vế theo vế các bất đẳng thức, ta có điều phải chứng minh. Nhận xét.
 Trong bài toán trên ta bắt gặp một khái niệm mới về “cặp đa thức nguyên tố cùng
nhau” (coprime polynomials) được phát biểu tương tự như đối với cặp số nguyên
tố cùng nhau, là khi các đa thức p t ,q t  thỏa mãn gcd pt ,q t  1
 Bài toán ta vừa thực hiện là mở rộng cho định lý lớn Fermat cho đa thức.
Câu 44. Cho đa thức P x và Q x với số thực k bất kì thỏa mãn P  z
Pz  k k
Q  z  |Qz  k P Q P Q P x Q x k  . Chứng minh rằng 0 0 và 1 1 suy ra được     Lời giải
Chinh phục olympic toán| 47
Bồi dưỡng học sinh giỏi
Không mất tính tổng quát, giả sử n  deg P  deg Q .
Đặt P z , z ,, z P z zz k , k , , 0  1 2 k  và 1  1 2 km .
Hai đa thức P Q trùng nhau tại k m điểm z , z ,, z 1 2 km .
Ta cần chứng minh k m n    
Ta có P x  x z  1 x z x z x z k k    kk m k  1  km 1 1 1   Với  ,,  1
km là các số tự nhiên.  
Xét P 'x , ta biết đa thức này chia hết cho x z  1 i i   k m i với 1,2, , . km  1
Suy ra x z P x i i | '  i1 km  
Vì vậy 2n k m  deg x z P n
k m n i  1 i  deg '   1, hay 1 . i1
Vậy ta có điều phải chứng minh. C Ọ H
Câu 45. Cho đa thức P x  x,deg P  2 . Chứng minh rằng tồn tại  m
để P m! ÁN là hợp số.
IMO Shortlist 2005 U TOỆ Lời giải LI
Xét số nguyên tố p và số tự nhiên chẵn k p . heo định lý Wilson ta có TƯ
p k!k 1   1
 k  1p  1!  1mod pn nn i i ni
Do đó k  1!P p k!  a k p k k a k p i
 1!   ! 1!     i   1!     mod  Í i0 i0 CH
Suy ra k  1!P p k!  S k  1!mod p với Sx  a a x  n a x n n1 0 P Ạ
Từ đó suy ra p P p k!  Sk  1! p T
Chú ý rằng S k  1! là đa thức phụ thuộc vào k . k 1!
Xét k  2a s n 1 ta được
là số nguyên dương chia hết cho mọi số nguyên tố nhỏ an hơn k .
Ta có S k  1!  a b ,b s n k k 1mod  .
Suy ra bk chỉ chia hết cho các số nguyên tố lớn hơn k .
Cho k càng lớn thì S k  1! càng lớn. Với k đủ lớn thì b  1 k .
Do đó tồn tại ước nguyên tố p của b
p Pp k ! k mà   .
Ta cần chọn k sao cho đủ lớn để P p k!  p .
Xét số nguyên tố q đủ lớn mà k  q  1!
48 | Tạp chí và tư liệu toán học Đa thức và số học
Nhận xét rằng k i là hợp số với i  1,, q  1
Khi đó p là số nguyên tố lớn hơn k P p k! : p nên p k q  1 .
Suy ra p k q r, r  0
Với số nguyên tố q đủ lớn, do deg P  2 nên
Pp k!  Pq r!  q r!  q  1! q r p
Vậy ta có p P p k! và p P p k! nên ta có điều phải chứng minh.
Câu 46. Cho f x là đa thức với hệ số hữu tỉ bậc lớn hơn hoặc bằng 2. Xét dãy an các
số hữu tỉ thỏa m n điều kiện f a    a n k n n , 1 1 
. Chứng minh rằng tồn tại 1 để a    a n n k n  1 Lời giải ÁN O
Phần đầu ta chứng minh dãy an là giới nội (bị chặn), nghĩa là tồn tại số M sao cho a M n   n , . IC TP f x
Thật vậy, do deg f  2 nên lim   . x YM x
Do đó tồn tại số M sao cho khi x M f x  x , chẳng hạn có thể lấy M a . OL 1 C a M n   Ụ
Với M chọn như thế ta chỉ ra n , . PH
Giả sử tồn tại số n để cho a M a     f a a M n . Khi đó n 1  nn . ương tự a      f a a M n 2  n 1 ,…, a M . NH n 1 1 I
Bằng cách tương tự ta có a a  ...  a M M a 1 2 n , trái với cách chọn 1 như trên. CH Vậy a M n   a n ,
nên dãy  n là dãy bị chặn.
Phần tiếp theo ta chứng minh rằng tồn tại k  1 để a    a n n k n  1 . d b x   b d ...
f x là đa thức với hệ số hữu tỉ nên ta luôn có thể viết f x 0  , trong đó c b ,...,b c d , 0 là các số nguyên. r Lại do dãy a a r s
n gồm toàn các số hữu tỉ nên ta giả sử 1
với , là các số nguyên. s
Ta cần chọn ra số tự nhiên N để cho ta có N.an là số nguyên với mọi n . a đặt N  . s b N a
d , ta chứng minh bằng quy nạp
. n là số nguyên. Thật vậy r
Ta có N.a sbr b d . . 1 d nguyên. s
Giả sử là N.a n N.a
n cũng là số nguyên với 1 . Ta xét số n1 .
Chinh phục olympic toán| 49
Bồi dưỡng học sinh giỏi dx b    b d ... 0  x   N d cN   x   Ta có f  
. Nên ta xét đa thức P x  f     a thì đa thức N    c n b N    d
P x là hệ số nguyên với hệ số bậc cao nhất là 1. d cN
Mà ta có P N.a      f a    a n n n , 1  f a   a n n n 0 1   1 ). b  ( do  1  d Nên N.a P x
n1 là nghiệm hữu tỉ của đa thức
  nên nó cũng là nghiệm nguyên. Vậy N.a N a
n1 là số nguyên, do đó
. n là số nguyên với mọi n . Dãy a N.a
n là dãy bị chặn và do
n1 là các số nguyên nên mỗi phần tử đề là một bội số 1 nào đó của . N
Do đó d y chỉ nhận hữu hạn giá trị khác nhau với vô hạn số hạng của nó. C
Chẳng hạn ta giả sử dãy chỉ nhận m giá trị khác nhau. Ọ
Ta xét m  1 số hạng của dãy là a , a ,..., a i , k 1 2
m1 . Khi đó tồn tại hai số nguyên dương 1 1 sao H
cho 1  i i k m  1 a a k m  1 1 1 1 và
. ương tự tồn tại các số cho mỗi đoạn   số 1 i 1 i  1 k j ÁN
hạng tiếp theo. Do các k k k
j chỉ nhận hữu hạn giá trị nên tồn tại số
j với vô hạn giá trị U TO khác nhau j . Ệ   LI
Ta sẽ chứng minh k chính là chu kì của dãy a a a n
n , tức là n k n , .
Ngược lại, giả sử có chỉ số n nào đó mà a   a . TƯ 0 0 n k 0 n Do dãy a n n n
n chỉ nhận hữu hạn phần tử nên ta có thể chọn
j đủ lớn  j 0  để có thể xảy VÀ Í ra a      a a   f a f a a n
. Khi đó theo giả thiết ta có n k 1
n k  n  . Tiếp tục quá trình j k nj n 1 j j j j CH P
trên sau nhiều lần ta sẽ lùi chỉ số n n a   a j về tới 0 và ta có là vô lí. 0 n k 0 n ẠT Vậy a    a n n k n , .
Bài toán được chứng minh. p
Câu 47. Tìm tất cả đa thức
 x sao cho p 2  p,p là số nguyên tố
IMOTC 2016 – Senior Batch Lời giải Ta có
2 2, 5 27 và 3 5  2 nên 2  5  1 hoặc 2  5  1 
Không mất tính tổng quát, giả sử
2  1 , ta thấy 0  1 bởi nếu tồn tại 1 ước nguyên tố lẻ cũa 0 là q thì | qq, hay |2q q (vô lý)
Xét p nguyên tố bất kỳ, gọi q là 1 ước nguyên tố bất kỳ của p thì |2p qp
Gọi m  ord q m m,q  1 2   , giả sử 2 có 
 nên theo định lý thặng dư Trung Hoa thì:
50 | Tạp chí và tư liệu toán học Đa thức và số học
 x pmodq  x p   vmodm
với v là 1 số thỏa mãn v không chia hết cho m và  p v;m  1 , hoàn toàn chọn được, do
m  2 nên m  2 ), có 1 nghiệm xmodqm
Dễ thấy do 0  1 nên  p;q  1 và  p v;m  1 nên x;qm  1
Theo định lý Dirichlet, tồn tại 1 số nguyên tố t để t x mod qm . t p tp Ta có |
q t p nên q t  q  q t  q 2 t q 2 2  1 p t  |2tp q  1 , vô
lý do t p không phải là bội của m .
Suy ra m  2 hay q  3 , vậy  p có dạng 3t với mọi p nguyên tố 1
Cố định p  3 và t , theo định lí Dirichlet thì tồn tại vô hạn số nguyên tố q thỏa mãn  1 mod 3   t q p . ÁN O Có t1
3 |q p nên t1
3 | q  p .
Kết hợp với 1 ta có    3t q
có vô số nghiệm nên đa thức là đa thức hằng.
IC TP Thử lại thấy x1 hoặc x 1 thỏa mãn. YM OL
Câu 48. Xét đa thức T x 3 2
x  17x  1239x  2001. Đặt C Ụ
T x T x ,T x T T x ,,T x T T x n  1     2    1   n1  
n   với mọi 1,2,3... PH
Chứng minh rằng tồn tại số nguyên n  1 sao cho T x  x n
chia hết cho 2003 với mọi số nguyên x . NH I
Lê Quang Nẫm – TPHCM CH Lời giải
rước hết ta có các nhận xét sau:
Nhận xét 1. x y   3 3
mod 2003  x y mod 2003 .
Để thuận tiện trong quá trình viết, ta kí hiệu là x y thay cho x y mod 2003 . Hiển nhiên ta có 3 3
x y x y (do 2003 là số nguyên tố) Ta cần chứng minh 3 3
x y x y .
Nếu x  0 thì y  0 do 3 3
x y mà 2003 là số nguyên tố.
Nếu x không chia hết cho 2003 thì suy ra 3
x không chia hết cho 2003, suy ra 3 y không
chia hết cho 2003. Suy ra y không chia hết cho 2003.
heo định lí Fermat nhỏ thì 2003 2003 2002 2002 xx, yy xy  1 . Do 3 3 2001 2001 2001 2002 2002 2001
x y xyxyxyy.yx y .
Nhận xét 2.T x  T y  x y mod 2003
Chinh phục olympic toán| 51
Bồi dưỡng học sinh giỏi
Ta phân tích T x  x  3   2x x 3 662 2003 657  2001  662 3
Do đó T x  T y  x  662  y  6623  x  662  y  662  x y .
Trở lại với bài toán. a đặt A  0; 1; 2;...; 
2002 . Khi đó với mỗi x A ,ta xét d y các đa
thức T x ,T x ,...,T xa b  1   2  
2004   . Theo nguyên lí Dirichlet sẽ tồn tại hai số 1 2004 mà T x T x T x a  
b   , vì khi chia cho 2003 thì có tối đa 2003 số dư. Ở đây các
k   này giờ là các
số)  T x T T    x T x x a  
a b a   b a   , theo nhận xét 2.
Như vậy với mỗi x A thì tồn tại một * n T x x sao cho n
, vì theo trên ta có thể xét một x
dãy 2004 đa thức khác, tức là một vòng mới mà có chỉ số tăng lên. Gọi n  1 là một bội chung
của n , n , n ,..., n 0 1 2
2002 . Ta sẽ chứng minh n thỏa mãn bài toán.
 Nếu x A thì ta có T x x T
x T x x T x T T x 2 . x n x n x nn   2n   n   (vì    x x x ương tự ta có T
x T x x k   kn   n   * , . C x x Ọ Do n n n
T x x T x x nn  2003
x tức là n là bội của x nên ta suy ra     . H
 Nếu x A thì tồn tại y A sao cho x y . Do đó T x T y y x n   n   . ÁN
Vậy T x  x n  2003. U TOỆ
Câu 49. Tìm tất cả các cặp số nguyên a, b sao cho tồn tại đa thức P x  x sao cho LI tích  2
x ax bPx là đa thức được viết dưới dạng: TƯ n n1 x c x   c x c
c ,c ,,cn1 1 0 với 0 1 n1 bằng 1 hoặc 1 VÀ Í Polish MO 2006 Lời giải CH n n P
Bổ đề. Cho đa thức P x 1  c x c x   c x P x n n1
0 , nếu 0 là nghiệm của   thì khi đó: ẠT  c x 1  max i 0
i0,n1 cn
Chứng minh. Rõ ràng chúng ta chỉ cần phải xem xét trường hợp x  1 0 : n1 n
c c x   c x n n1
c x c c x   c x 0 1 0 n1 0  x n 0 0 1 0 n1 0 0 cn n 1 1 1 c x   n c c c c i i x   n i i x  max  n i i x i 0  max 
x  1  max i 0 0 0 0 in
i0,n1 c x ini0 c c 0, 1 c 0, 1 c n 1 n i0 n n i0 0 n
Quay trở lại bài toán ta có b P 0  c b  1  ,1 0  .
Hơn thế nữa, nếu x n nx c x   c x c 0 là nghiệm của 1 n1 1 0 thì khi ấy ta có  c x 1  max i  2 0
i0,n1 cn
52 | Tạp chí và tư liệu toán học Đa thức và số học Như vậy, với x x ax b
0 là nghiệm của phương trình 2   , ta có:  2 2  a a 4b a a 4        b x   , x  2 0  và 0  2 2  
Kết hợp với dữ kiện b  1  , 
1 , ta có thể tìm được giá trị của a với a2, 1  ,0,1,2.
Câu 50. Cho hai đa thức hệ số nguyên Pxn n1  a x a x
 a x a n n1 1 0 Qxn n1  b x b x
 b x b n n1 1 0
Biết rằng a b ab n
n là một số nguyên tố và n1
n1 . Gọi m là là một nghiệm hữu tỷ
chung của hai đa thức. Chứng minh rằng m là một số nguyên. Lời giải r m
r,s ,gcd r,s  1 ÁN Đặt   . s O  r  Ta có P  0   
r a ,s a . ương tự r b ,s b . IC T 0  s n 0 n P Từ đó ta có | s a b a b s s a b n n , mà  n
n là một số nguyên tố nên 1 hoặc   n n YM r
Nếu s  1 thì m
r  , ta có điều phải chứng minh! OL s C Ụ  r   r
Nếu s a b PQn n thì ta có     0  s   s  PH n n2  r r r a b a b a b a b n n                   nn   0 2 2   1 1  0 0 NH I  s   s s  a b r a b r a b r s a b s n
n   n    n    n   n nn  2   1 0 2 2 1 1  0 0  CH  sn r  an b r
s  a b r n sa n b s   | n s r nn  2 2   1 0 2 2 1 1  0 0 
Lại có s nguyên tố nên s r, mâu thuẫn với gcd r, s  1 .
Vậy ta có điều phải chứng minh!
Câu 51. Hỏi có tất cả bao nhiêu đa thức P x
n   bậc n chẵn thỏa m n các điều kiện
 Các hệ số của P x M  0; 1  ;1 Pn 0 0
n   thuộc tập   và   .  2
Tồn tại đa thức Q x có các hệ số thuộc M sao cho P x  x  1.Q x n  .
Nguyễn Viết Long – THPT Chuyên Lam Sơn – Thanh Hóa Lời giải
Đặt deg P x n m nn Q x b x
b x   ... b   x b n   2 . Xét đa thức   2 3 0 1 n 3 n2 .
Từ giả thiết ta có P x   2 x  1.Q x n   nên suy ra
Chinh phục olympic toán| 53
Bồi dưỡng học sinh giỏi
P x b x b x   b b x   b b x    b     b x b x b n   n n 1   n 2 
n 3 ...  n n  2 0 1 2 0 3 1 2 4 n 3 n2 .
Từ đó suy ra số các đa thức P x
b ,b ,...,b
n   bằng số dãy số  0 1
n2  trong đó các số hạng của dãy b b    k n b     b k k 1;0;1 k 1;0;1
n thỏa m n điều kiện là   với mọi 0 2 và 2  
với mọi 0  k n  4 đồng thời bn 0 2 .
 Để ý rằng nếu b   b   k 0; 1 k 1 thì 2  .
 Để ý rằng nếu b b   k 1;0;1 k 0 thì 2  .
 Để ý rằng nếu b bk 0;1 k 1 thì 2   . Gọi x
b ;b ;...; b
b M i k i ,0
k là số các dãy chỉ số chẵn  0 2 2 k  thỏa mãn 2 và b       b M i k i i ,0 2 2 2 Gọi y
b ;b ;...;b b     M i k i ,0
k là số các dãy chỉ số lẻ  1 3
2k1  thỏa mãn 2 1 và b       b M i k i i ,0 2 3 2 C
Ta có x  2; x  4 và x   b   b   x x b k 0  b k 0; k 0 k k 2  vì nếu thì 2 2  2 k  và nếu thì có Ọ 0 1 1 k 1 2 2 H ba cách lấy bb xM
b ;b ;...;b k 0 2k 2 , số dãy  0 2 2 k  mà 2 bằng k1 . k k ÁN 1 2 1 2
Bằng cách sử dụng phương trình đặc trưng ta tìm được x k . 2 U TOỆ
ương tự ta có y  3, y  7, y      y y k k 2 k k , 1 0 1 1 1 . LI
Do đó số các dãy số b ,b ,...,b 0 1 n2  bằng TƯ    x          x   y   m 1  m 1 1 m m m m m 1 2 1 2 1 2 1 2 1 2 2  2      VÀ Í  mmnn
1  2 2 1  1 2 2 1  2 1  m
1 2 1 1 2 1 1     n 1 1 2 . CH 2 2 P Ạ T 
Câu 52. Cho dãy số nguyên a m n a n  thỏa mãn |
a với mọi số tự nhiên m,n n1 m n
phân biệt. Giả sử tồn tại đa thức P x sao cho a P n ,n n
. Chứng minh rằng tồn tại
đa thức Q x sao cho a Q n ,n n . Lời giải
Đặt degP d . Tồn tại đa thức Q duy nhất có bậc cao nhất là d sao cho Q k  ak với
k  1,2,, d  1 . Ta chứng minh Qn  an với mọi n .
Đặt n d  1 . Đa thức Q có thể không phải là đa thức hệ số nguyên nên ta không thể trực tiếp suy ra n  |
m Qn Qm, nhưng chắc chắn rằng Q là đa thức có hệ số hữu tỉ, khi đó
tồn tại số tự nhiên M sao cho R x  MQ x là đa thức hệ số nguyên. Từ điều kiện đề bài
ta có M a Qn  M a a   R n  R k n k k   d n n k
  chia hết cho  với 1,2, , 1 . Vậy
54 | Tạp chí và tư liệu toán học Đa thức và số học
với mỗi số n ta có a Q n L n n n d M a Q n Cn n lcm
 1,  2,,   1    n   d n   hoặc    
với C là hằng số không phụ thuộc vào n .
Giả sử rằng a Q n n
  ở một vài giá trị n. Lưu ý rằng L n  1 n d  1
n không nhỏ hơn tích   
 được chia bởi tích P của số
gcdn i,n j trên tất cả các cặp i, j phân biệt lấy từ tập {1,2,, d  1}.
Do gcd n i,n j  i j , ta có d d 1 P 1 2  
d . Từ đó ta được:
n 1n  2 n d 1  PL d CPn n
Điều này sai với n đủ lớn vì vế trái có bậc là d  1 . Vậy a Q n n N n
  cho mỗi giá trị n đủ lớn, ta gọi tạm là  .
Nếu n N , từ điều kiện đề bài M a Q n M a a R n R k n    n k       chia hết cho
mn với mọi giá trị m N , vậy chúng phải bằng 0. ÁN
Vì vậy a Q n n   với mọi n. O IC T
Câu 53. Cho n là số nguyên dương và a , a ,..., a 1 2
n là các số thực dương. P
a đặt g x  x a
x a ... x an . 1   2    YM Gọi a nn
f x x a g x x
b x  ... b x b n n .
0 là một số thực bất kì và đặt       1 0 1 1 OL
b ,b ,...,b
a a a  ...  an. C Chứng minh rằng 1 2
n1 đều là số âm khi và chỉ khi 0 1 2 Ụ
VMF – Diễn đàn Toán học PH Lời giải Chiều suy ra. NH I n
Thì theo định lý Viete ta có b a  ak , 1 0
từ đó thì ta có điều phải chứng minh. CH k1
Chiều ngược lại. n
Với x  0 thì ta có a a b bn 0. 0 k n1 nên suy ra 1 k1
Ta giả sử rằng a  a  ...  a a 1 2 n 0
Mà ta có g x  x a g ' x n  1 n x  ... b   x b n n . 0      1
Lại theo định lý Lagrange thì ta thấy tồn tại các số dương y i n i , 0, 1 sao cho
a  y  a  y  ...  a y a
f ' y  0, i   0,n  1 1 1 2 2 n 0 0 và  i  1 n Do đó ta có
f 'x  x y x y 0   i  . n  1 i1 n1 n n  
Đồng nhất hệ số của n 1
x  thì ta thu được y  y
a   a i i  0 0 0 .  i1 n 1  i1 
Từ ý tưởng trên ta sẽ dùng phương pháp quy nạp để chỉ ra các hệ số bi đều là số âm.
Chinh phục olympic toán| 55
Bồi dưỡng học sinh giỏi
Câu 54. Cho F là tập các đa thức  có hệ số nguyên và phương trình  x  1 có
nghiệm nguyên. Cho trước một số nguyên dương k, tìm giá trị nhỏ nhất của m  1 theo
k thỏa mãn tồn tại   F sao cho x  m có đúng k nghiệm nguyên phân biệt.
VMF – Diễn đàn Toán học Lời giải
Do  x  m k nghiệm nguyên phân biệt nên ta có:
x  x x x x ... x x Q x m 1   2   k   
Với các số nguyên khác nhau x , x ,..., x Q x x 1 2 k và    
Gọi t là nghiệm của phương trình  x  1 khi đó ta có:
t x t x ... t x Q t m k 1  1   2     
 1  m t x t x ... t x Q t 1 2 k  
 1  m  1.2.3...k.1  m  1  k!  m k! 1 C Ọ
Vậy từ đây ta suy ra gái trị nhỏ nhất của m k ! 1. H
Và nó xảy ra khi và chỉ khi  x  x  1x  2...x k  1x k  k! 1. ÁN
Câu 55. Cho hai đa thức P x ,Q x  x nguyên tố cùng nhau và khác đa thức hằng. U TOỆ
Chứng minh rằng không có quá ba số thực  thỏa mãn P x  Q x là bình phương LI của một đa thức. TƯ
USA December Team Selection Test 2016 Lời giải VÀ Í
Kí hiệu  Ax , Bx là ước đa thức monic có bậc lớn nhất của Ax , Bx. CH
rước hết, ta phát biểu một bổ đề sau đây: P Ạ
Bổ đề. Nếu ab cd  max deg aP x  cQ x,deg bP x  dQ x   max P x ,Q x . T Chứng minh
Đặt n  maxPx ,Qx thì ta có vế trái rõ ràng không lớn hơn . n Xét hệ số của n
x trong hai đa thức aP x  cQx ,bP x  dQ x  sẽ có một đa thức có hệ số
khác không và dễ dàng suy ra vế trái bằng n nên ta có điều phải chứng minh.
Quay lại với việc chứng minh bài toán
Giả sử tồn tại ba số thực phân biệt  ,  ,  , P x   Q x R x i   i  2 1 2 3
Theo bổ đề trên thì tồn tại đa thức R x2
maxP x ,Q x R x i có bậc bằng
    giả sử là  2 1    a đặt 1 2  
, Ax  R x R x ,B x 1   2        1 3
R x R x ,C x R x R x ,D x R x R x 1   2     1   3     1   3  
56 | Tạp chí và tư liệu toán học Đa thức và số học
Khi đó thì ta có Ax  Bx  C x  Dx  2R x , A x B x 1      
 C xDx ,  1,Ax,Bx  C x,Dx  1
AxBx có bậc bằng maxP x ,Qx .
Bây giờ ta sẽ đặt P
x  Ax,Cx,P
x  A x D x A x C x A x D x   ,           , P
x  Bx,Cx,P
x  B x D x B x C x B x D x   ,          
hì ta có các đa thức trên nguyên tố cùng nhau. Giả sử x A x
0 là nghiệm bội n của   thì
nó cũng là nghiệm bội n của duy nhất một trong hai đa thức Px,P x A x C x A x D x           hay
cũng là nghiệm bội n của đa thức Px.P x A x C x A x D x           .
ương tự thì ta cũng có điều ngược lại.
Vậy Ax  aPx.P x A x C x A x D x          
với a là hằng số nào đó.
ương tự thì ta cũng được: ÁN O
Bx  bPx.P
x,Cx  cPx.P
x,Dx  dPx.P x B x D x B x C x A x C x B x C x A x D x B x D x                          
Mà ta có Ax C x  Dx  Bx. IC TP Giả sử x Px.P x A x C x B x D x  
0 là nghiệm bội n của        
thì nó cũng là nghiệm bội n của một YM trong hai đa thức Px,P x A x C x B x D x          
nên là nghiệm bội ít nhất n của đa thức OL
Ax C x . C Ụ Giả sử x A x C x
0 là nghiệm bội n của  
  thì nó là nghiệm của nhiều nhất một trong hai PH đa thức Px,PxP x x x B x D x           , A x C x B x D x giả sử    
không có nghiệm 0 và 0 nghiệm bội m
NH I có thể bằng không của Px     . A x C x CH
Giả sử m n thì ta đạo hàm Ax C x  Dx  Bx từ 0 đến m  1 lần.
Thay x x0 thì ta có iA x i  C x m1  0, 0
  i m, Ax m1  C
x  0,Bx  Dx  0.
Đạo hàm AxBx  C xDx m  1 lần, thay x x0 vào thì ta có m1 A
xBx m1  C
xDx, dễ thấy điều vô lý.
Vậy ta phải có m n nên từ đây ta có
Ax C x  Dx  Bx  P x P x
A x C x   . B x D x            aP x bP  P dP x bP  P A x D x   , B x C x B x D x A x D x                  
BxCx
AxCx Thay Px,P x A x C x B x D x           thì ta có
Chinh phục olympic toán| 57
Bồi dưỡng học sinh giỏi     ab A x B x P x P x aP x cP x dP x bP x
A x D x   . C x D x   2
A x D x   B xC x   A x D x   B xC x                            1 2R x adP x bcP x 1  
A x D x   B xC x           
Áp dụng bổ đề trên thì ta được maxdegP x P xR x
A x D x   , deg  B x C x   2 deg  1          
 deg Qx  deg AxBx  deg P x P x P x P x A x D x  
deg  B x C x   maxdeg A x D x  ,deg B x C x                  
Từ đây ta suy ra deg P
x  degPx          0. A x D x B x C x
Từ đây thì ta có R x ,Q x P x 1  
  là các đa thức hằng nên   cũng là đa thức hằng, điều này
là vô lý do giả thiết bài toán nói đa thức P x khác đa thức hằng. C
Vậy từ đây suy ra tồn tại nhiều nhất hai số thỏa m n điều kiện đề bài, nên từ đó ta suy ra Ọ điều phải chứng minh. H ÁN
Câu 56. Chứng minh rằng nếu đa thức f x  x có bậc n và nhận giá trị nguyên tại
n  1 giá trị nguyên liên tiếp từ a a n, a thì f x , x   . U TOỆ Lời giải LI
rước hết, ta cần có công thức nội suy Abel – Newton được phát biểu như sau: TƯ
Cho f x  x có bậc n n số thực a , a ,..., a n  1 2
n khi đó tồn tại duy nhất 1 số thực VÀ
b ,b ,...,b 0 1 n sao cho: Í
f x  b x a
x a ... x a b x a x a x a      b x a b n ... n ... 0  1   2    1  1   2   1  n 1  1  n CH P
Công thức này có thể chứng minh rất là đơn giản bằng phương pháp quy nạp Toán học. ẠT
Quay trở lại bài toán
Ta áp dụng công thức nội suy Abel – Newton cho n số thực a a i i   n i , 1, .
f x  b x a  1 x a  2 ... x a n b x a  1 x a  2 0      1   
...x a n  1  ... b
x a   b n 1 1   n
f a  1  b   n
f a  2  b
b   b   n .1! 1 n n1
 f a 3  bbb   bn .2 ! n .1! n n .2 ! Từ đây suy ra 2 1 2  . ........ 
f a n  b . n 1 ! 1   
f a  b .n!  0 nx a
Ta sẽ viết lại đa thức f x như sau f x  k!bnk   . k0  k
58 | Tạp chí và tư liệu toán học Đa thức và số học
x a x a  1x a  2...x a k rong đó thì   
 ,x  và b   k n k ! , chứng minh  k k! trên.
Vậy từ đó ta được f x  , x   .
Câu 57. Tìm tất cả các đa thức P x  x sao cho với mọi a,b a không là nghiệm
của P x thì P aP a b  P b . MYTS 2016 Lời giải
Ta sẽ làm việc trên các số nguyên lớn hơn nghiệm thực lớn nhất của P x , nghĩa là ta
không cần quan tâm đến việc P   0.
Đế ý rằng P x  c const là một đa thức thỏa mãn yêu cầu đề bài, xét deg P x  1. ÁN O
Ta nhận xét thấy.
Nếu P x là một đa thức thỏa mãn yêu cầu bài toán thì P x cũng thỏa mãn yêu cầu đề bài
IC TP Không mất tính tổng quát, ta có thể giả sử hệ số cao nhất của Px là dương. YM Ta có lim (
P x)   nghĩa là từ một lúc nào đó, đa thức của ta chỉ nhận giá trị dương, lớn x OL P 1 . C hơn hẳn   Ụ
Px  1  P1
P x   P x P   1   1 heo đề bài thì ta có    . PH PxPx
Q x P x  1  P x NH Đặt       I
Trường hợp 1. deg P x  2 khi đó thì deg Qx  deg P x  1  1. CH
Qx  P1
Qx  P1 Khi đó thì
. Mặt khác, ta lại có lim  0. Px  x Px
Hay từ một chỉ số x0 nào đó trở đi thì
Qx  P1
Qx  P1 Px  1  Px
 0,x x Q x P 1  0,x x 0     0
Nghĩa là một đa thức có deg Q x  P 1  deg P x  1  1 và có vô số nghiệm, vô lý.
Trường hợp 2. Ta có deg P x  1 thì ta đặt P x  ax b, a  0. ax Khi đó
  b  0 hay Px  ax, a  0 thử lại thấy thỏa mãn yêu cầu đề bài. ax b
Vậy tất cả các hàm số thỏa mãn yêu cầu đề bài là P x  c  const và P x  ax, a  0.
Chinh phục olympic toán| 59
Bồi dưỡng học sinh giỏi
Câu 58. Xét đa thức P xn n1  x a x
  a x a n nP x n ... 1 1 0 với 2,
. Giả sử   có n
nghiệm là x , x ,..., x max x
x , x ,..., xn. n . 1 2 Kí hiệu
i  là số lớn nhất trong các số 1 2 Chứng 1
minh rằng P x   2 n1 n
 2n  ,x  maxx   i  với 0 . ix  1 xi
VMF – Diễn đàn Toán học Lời giải n
Do P x bậc n và có n nghiệm là x , x ,..., x
P x   x x 1 2 n nên    i i1
Với x  max x , i   1, n x x i   n i thì i 0, 1,
Từ đó ta áp dụng bất đẳng thức Cauchy – Schwarz thì ta được: nn n n x x P x   1 
P x   1     i  .nn   n n
ix x x x x x i ii i  1 1 1 i C Ọ
Do vậy ta cần chứng minh rằng H
x x   x x in    n n n 2 n1  in n   2n   
 2nn nn1  ÁN ix x x x i i  1 1 i n
Ta sẽ chứng minh x x   n x x i n in1
 2    i  ,  1, . U TOỆ
Tổng quát, với mọi số thực không âm x, y n  2, n  . LI  n n n x y 2 nk nk k k nk k n n 1 1   n2 2
 C x y C x y x nx y x y n     nk0 k0 2
nn  1 n n2 2 n1 n1 VÀ  2
x .x y nx y  2nx y Í 2 CH nn  1 n n1 P Do 2
n,n  2 nên từ đây ta suy ra được x x   n x x i n i
  2    i ,  1, . 2 ẠT
Hay bất đẳng thức ban đầu được chứng minh.
Câu 59. Giả sử R là nghiệm của phương trình n n1 x a x  ... a    x a n n 0 1 1 và đặt n n A  a B ja A B A R j ,
  j. Khi đó thì ta có . j1 j1
VMF – Diễn đàn Toán học Lời giải a n j Đặt c c  c j 1. j suy ra j 0 và A j1
Do hàm số y   ln x là hàm lõm trong khoảng từ 0,  nên theo bất đẳng thức Jensen thì
60 | Tạp chí và tư liệu toán học Đa thức và số học 
A  n   n A   n a   j  ln c c f R
j     j    ln   j
j    ln   j    ln    ln1  0 
R  j  1 j   1 R j   1 R n n n    
Suy ra c ln  j R c A
ln A c   R  jc j ln   j
j ln  0 và   j j1  j1   j1  1 n 1 n aj Hay a A   ja R c A j ln   j ln  , do j , 0. A j1 A j1 A
Vậy nên ta được: ln  A   ln  B A B A
R A R
Hay từ đây ta được điều phải chứng minh.
Câu 60. Cho đa thức P x là đa thức monic bậc n  1 có n nghiệm thực là x , x ,..., x 1 2 n
phân biệt và khác 0 . Chứng minh rằng: 1 1 1  1  n1 ÁN   ...  O x Px x Px x Px x x x n n ... 1  1 2  2    1 2 n IC T
VMF – Diễn đàn Toán học P Lời giải YM
Từ giả thiết bài toán thì ta suy ra P x  x x
x x ... x x 1   2   n  OL Ta bổ sung x  0 j  0 và với mọi 0 thì ta có: C Ụ Pxn n
   x x x Px x x x   x x i jjn j
j in j i PH
j1 i1;ij
i1;ij
i0;ij Q  x j   n j n  1, 1, NH  I
a xét đa thức Q x  1  x x   n in i1 Q
 0  1  P 0  1   1    xi CH  i1
Ta dễ thấy deg Qx  n nên ta áp dụng công thức nội suy Lagrange cho Q x tại n  1 x x n
n   i
i0;ij điểm x Q x  Q x i thì ta được    jn . j0  x x j i
i0;ij x x x x x x nn n ni
  i n    i n ii j i i j  0;  0;  Q P x i  1 0      Qn   nx Px x P x jj 0 1 1  1n j j1   j xj  x ii i1 i1
So sánh hệ số của n
x ở hai vế thì ta được:
Chinh phục olympic toán| 61
Bồi dưỡng học sinh giỏi n n1  n     x 1  1n Q0  1 1   1 1 i n n      i   1n 1 1 1       1   n n n
j1 x Pxx Px jjj 1 jxj xx i i i i1 i1 i1
Và từ đây ta có điều phải chứng minh. n x
Câu 61. Cho n  2 và đa thức P x xác định bởi P x   k . Chứng minh rằng k0 k !
phương trình P x  0 không có nghiệm hữu tỉ.
VMF – Diễn đàn Toán học Lời giải Nếu  
là nghiệm của P x thì  cũng là nghiệm của phương trình: n n1 n! k C x nx
 ... x  ... n!  0 Ọ k! n nn! H 1
   n  ... k
  ... n!  0 1 k! ÁN
P x là đa thức hệ số nguyên có hệ số cao nhất là 1 và có nghiệm   thì  là số nguyên. U TOỆ
Gọi p là một ước nguyên tố bất kì của n LI
k   k   k
Với k  1, n ta đặt r v k r k p  !   k      ...  . TƯ 2  s
p   p   p
rong đó, s là số tự nhiên thỏa mãn s s 1 p k p    VÀ Í k k k 1   r    p ...  k.
k r r r k CH k 2 s p p p 1  s n k n p P Ạ
n! r k1 T
r r r k p n k n 1  n k! n
Lại có, do p n nên từ   k k 1 !
1  p   p   nr p
k ,k  1,n k! Do 1 nên 1 n r p
n!, mâu thuẫn do r v n n n r p n p ( !) nên ! không chia hết cho 1 .
Vậy điều giả sử là sai hay đa thức P x không có nghiệm hữu tỉ.
Câu 62. Cho p là một số nguyên tố. Tìm tất cả các đa thức f x với hệ số nguyên sao
cho với mọi số nguyên dương n , f x là ước của n p  1.
THTT Tháng 12 – 2014 Lời giải
62 | Tạp chí và tư liệu toán học Đa thức và số học
Ký hiệu A là tập các ước số nguyên của p  1 . Khi đó tất cả các đa thức hằng
f x  b,b A đều thỏa mãn bài toán, vì *   , n n
p  1 p  1 b .
Ta sẽ chứng minh đó cũng là tất cả các đa thức thỏa mãn bài toán.
Thật vậy. Phản chứng giả sử tồn tại đa thức f x mà deg f x  1 thỏa mãn bài toán. Với * n
và giả sử q là ước số nguyên tố bất kỳ của f n .
Ta có f n q  f n n q n  q . Mà f nq nên từ đó ta có f n qq . Như vậy ta có n p
1 f nq, n  q p
 1 f n qq .
Nên suy ra nq  1   n  1  n p p p q p  1 q .
Từ các sự kiện trên ta suy ra p q vì
 ,   n 1  n q f n f n p
q p  1 và q p  1 q .
Theo định lý Fermat ta lại có q
p pmodq, suy ra q  1   q p
p p  p  1 q .
Bây giờ ta xét các số tự nhiên * n
n  1 p  1 . Ta biết số các số n thỏa mãn là vô hạn, ÁN O
trong khi tập A ta xét là hữu hạn. Suy ra tồn tại số n f n  A . n p   p  IC T Mặt khác 1  1B với P n1 n2 B pp
 p  1  nmod p  1  1mod p  1  p  1,B  1 . YM
Do f n  A suy ra có ước số nguyên tố q của f n mà B q p  1  q . OL C
Mâu thuẫn với gỉa thiết quy nạp. Ụ PH
Câu 63. Cho số nguyên dương n và số nguyên tố p lớn hơn n  1 . Chứng minh rằng đa 2 p x x x NH
thức P x  1    không có nghiệm nguyên. I n  1 2n  1 pn  1 CH Lời giải
n 12n1   pn  1
Ta viết lại P xp p1 2
a x a x  a x a xa a p p1 2 1 0 trong đó k kn  1
là các số nguyên ( chú ý là ta quy đồng hết nên trên tử chứa cả nhân tử dưới mẫu).
Do p là số nguyên tố lớn hơn n  1 nên  p,n  1 .
Vì thế tập A  1.n  1; 2n  1;; pn  
1 là hệ thặng dư đầy đủ modulo p.
Do đó tồn tại duy nhất k 1, 2,, 
p sao cho kn  1  0mod p. Ta lại có 2
k  1,0  kn  1  p nên 2
kn  1 p . Như vậy với số k đó thì a p k ( vì nhân tử
kn  1 đ bị rút gọn), đồng thời các hệ số còn lại đều chia hết cho p kế cả các hệ số a , a 0 1 nhưng không chia hết cho 2 p .
Bây giờ ta sử dụng lập luận đó để chứng minh phương trình trên không có nghiệm nguyên.
Ngược lại giả sử rằng phương trình có nghiệ nguyên là c thì khi đó ta có
Chinh phục olympic toán| 63
Bồi dưỡng học sinh giỏi Pcp p1 2
 0  a c a c  a c a c a p p 0 1 2 1 0
Theo lập luận trên thì ta có a p,i 0,1, 2,, 
p ,i k, k i 0;1 . Do đó dẫn đến k a c p k
( do p là số nguyên tố). Suy ra k i 2
c p c p a c p i  2,3,p i với mọi  . a cũng có 2 a c p a ,c 1
vì cả 1 đều chia hết cho p. P c 2 2  0 p a p 0 là điều vô lí.
Câu 64. Cho đa thức P x 3 2
x  11x  87x mm  . Chứng minh rằng với mọi m tồn
tại số nguyên n sao cho P n 191 . Lời giải Ta có P n 3 2
n  11n  87n m . C
Đối với bài này ta sẽ chứng tỏ với n A  1, 2,,191 là hệ đầy đủ modulo 191 thì ta có hệ Ọ * H
A  P1 , P2 ,..., P191 cũng là hệ đầy đủ modulo 191. ÁN
Như thế thì khi đó tồn tại n A để P n  0 mod 191 . Để chứng minh được *
A hệ đầy
đủ modulo 191 thì ta cần chỉ ra n , n A n n mod191 1 2 và 1 2   thì U TOỆ
Pn P n mod191 1   2  . LI Thật vậy. TƯ
Giả sử ta có P n P n
mod191  27P n  27P n mod191 27,191  1 1   2    1  2   , vì   . 3 3 3 3 VÀ
 3n  11  18.191n  11  27m  3n  11  18.191n  11  27m mod191 1  1  2  2   Í
 3n  113  3n  113  n n mod191 1 2 1 2   CH P
Để hoàn thành được bước cuối, ta cần có bổ đề sau Ạ 3 3 T
Bổ đề. Giả sử p là SNT và p  2 mod 3 thì x y mod p  x y mod p .
Chứng minh bổ đề  Nếu x   p 3 3 0 mod
x  0  y  0  y  0mod p . Vậy x ymod p .
 Nếu x  0mod p thì dễ thấy 3 x   p 3 0 mod
y  0mod p  y  0mod p .
Do nên theo định lí Fermat ta có p1 x   p 3k1  x   pp1 3k1 1 mod 1 mod , yy  1mod p . k k Từ đó suy ra 3k1 3k1 xy
p  x  3
x   y  3 mod
x  mod p  x ymod p do x, p  1
Bổ đề được chứng minh nên ta cũng kết thúc bài toán.
64 | Tạp chí và tư liệu toán học Đa thức và số học
Câu 65. Cho m là số nguyên dương, tìm số nghiệm của phương trình 2
x xmodm . Lời giải
Ta giả sử phân tích m ra số nguyên tố là 1 2 
m p p k p p 1 2 k trong đó
i là các số nguyên tố. Khi đó phương trình 2 x x m 2 mod x x mod     i p i k i  ,   1; . Ta có 2 x x  i p  2 mod x x 0mod i p
x x 1 0mod         i p i i i  .
Vì x, x  1  1 nên phương trình tương đương x 0 mod   i p x 1 mod   i p i  hoặc  i  .
heo định lí thặng dư rung hoa thì với mỗi bộ a , a ,, a 1 2
k thì hệ phương trình x a mod     i p i i  
luôn có nghiệm duy nhất theo modulo 1 2  m p k p p 1 2 k . i   1; k Do mỗi phương trình 2 x x 0mod    i p p
i  luôn có hai nghiệm theo modulo i i nên
phương trình đ cho có tất cả 2k nghiệm. ÁN O
Câu 66. Cho p là số nguyên tố  p  3 . Xét đa thức IC TP
f x  p   p2 x
 p   p3 2 1 2 x
 3x  2x  1 . YM
Biết rằng hệ A  a , a ,, a 1 2
p là một hệ thặng dư đầy đủ modulo p. Chứng minh rằng OL
B  f a , f a ,, f a 1 2 p  C khi đó hệ    
  cũng là một hệ thặng dư đầy đủ modulo p.Lời giải PH p p  1
Xét x  1  f 1  p  1  p  2    2  1  p . 2 NH I 1
Nếu x  1 thì ta có   
p 1 p p p p f x   1 2 x
x  p  2 2 3 x
x  x  1 . CH x  1
Do đó x   f x  p   p1 p2 x
x   p   p2 p3 x
x    2 1 1 2
2 x x  x  1
Suy ra x   f x   p   p1 x   p2 p3 1 1 xx  1 .
Áp dụng công thức tổng của cấp số nhân và quy đồng ta suy ra
x  2 f x  p  x   p1 x   p1 1 1 1 x
 1  1 xmod p 1 .
Ta cần chỉ ra bằng phản chứng
Giả sử B không phải là hệ thặng dư đầy đủ.
Khi đó tồn tại a , a A1  i j p
f a f a m p i j mod i j  sao cho       . ma a p i 12  1   i mod  Từ 1 ta suy ra  m p . m   a a p j 1 0; 1 2     1  j mod 
Nếu m  0 thì từ hệ ta suy ra a a  1 p i j
mod  là điều vô lí.
Chinh phục olympic toán| 65
Bồi dưỡng học sinh giỏi
Nếu m  0  f a f a
f 1 p a  1, a  1 i  
j  0. Nhưng   i j .
Do a , a A
1  a ,1  a i j 1 i j nên 
 . Nên nhân phương trình trên và phương trình dưới của
hệ lần lượt với 1  a và 1  a ta thu được j i
ma     a   ma  2 2 1 1
1 1  a mod p  1  a  1 a mod p  a a i j j i i j i j ( vô lí).
Câu 67. Xét đa thức P x 3 2
x  14x  2x  1. Chứng minh rằng tồn tại số nguyên dương
n sao cho với mọi số nguyên x ta có 101 P PP x    x .
Thặng dư bình phương – Nguyễn Duy Liên Lời giải
Ta sẽ chứng minh rằng x y mod 101  P x  P ymod 101 .
Ta có P x  P y  x y 2 2
x xy y  14x  14y  2 C Ọ 2 2
Do đó 4 Px  Py 
 x y 2x y  14  3y  29  H   ÁN
x y mod101
Vì 4;101  1 nên P x  P ymod 4   .
 2x y  142  3y  292   0mod101   U TOỆ 2 2
Xét 2x y  14  3y  29   0mod 101 . LI    3  TƯ
Nếu y  29,101  1 thì khi đó   
 1 nên 101  1mod6 , vô lí.  101  VÀ
Nếu 101y  29  101 2x y  14  x y  29 mod 101 . Í
Như vậy cả hai trường hợp ta có x y mod 101 . Nhận xét được chứng minh. CH P
Xét 102 đa thức như sau P x , P P x ,..., P P ...P x... . ẠT
Theo nguyên lý Dirichlet phải tồn tại hai số m, n 1; 2;; 10 
2 ,m n sao cho
PP...Px...  PP...Px...mod101 . m n
Từ nhận xét trên ta suy ra P P...Px...  xmod101 với mọi số nguyên x. mn
Câu 68. Cho tập S  p , p ,, p P x 1 2
k  là tập hợp k số nguyên tố phân biệt và   là đa
thức với hệ số nguyên sao cho với mọi số nguyên dương n đều tồn tại pi trong S sao cho p P n p P n n i
 . Chứng minh rằng tồn tại i sao cho   * ,  i . Lời giải
Ta chứng minh phản chứng bằng cách sử dụng định lí thặng dư Trung Hoa.
66 | Tạp chí và tư liệu toán học Đa thức và số học
Giả sử không tồn tại i sao cho p P n * ,ni .
Suy ra với mọi i  1, k , luôn tồn tại a p P a i sao cho ii . x a p i mod i
heo định lí thặng dư rung Hoa luôn tồn tại x sao cho  . i   1; k
Px  Pa p
i  mod i  Do đó ta có  . i   1; k Hay ta có p P x i k i
 ,  1; . Điều này mâu thuẫn với giả thiết.
Câu 69. Cho đa thức P x 3 2
x  153x  111x  38 .
a) Chứng minh rằng trong đoạn 2000 0;3  
 tồn tại ít nhất một số nguyên dương a sao cho Pa 2000 3 . ÁN   P a O b) Hỏi trong đoạn 2000 0; 3 
 có tất cả bao nhiêu số nguyên dương a sao cho   chia hết cho 2000 3 . IC TP Lời giải
a xét phương trình đồng dư P x   2000 0 mod 3 . YM
a để ý rằng P x    3 2
0 mod 3  x  153x  111x  38  0mod 3 . OL C 3
x  38  0mod3  x  1mod3 Ụ . 1999 x y   y   PH Đặt 3 10 3 1 .
Vậy P x  P y     3 2 y y y   2000 3 1 27 52 22 3 mod 3  . NH I y  3tQy 3 2
y y y    1997 52 22 3 0 mod 3  1998 , 1  t  3   1 CH , suy ra  . y  3t   1
Nếu y  3t  1 thì phương trình 3 2
y  52y  22y  3 không chia hết cho 9, nên không chia hết cho 1997 3 .
Nếu y  3t thì 3 2 y y y    3 2 52 22
3 3 9t  156t  22t  1 .
Nên suy ra P x  
2000   Gt 3 2  t t t    1996 0 mod 3 9 156 22 1 0 mod 3 .
Bây giờ ta xét đa thức G t 3 2
 9t  156t  22t  1 .
Ta nhận thấy G t  0 mod 3  22t  1  0mod 3 .
rong đoạn 0; 3 phương trình 22t  1  0 mod 3 có nghiệm duy nhất t  2 , và có thêm
điều kiện là G'2  22  1mod 3 suy ra G'2  0 mod 3 nên áp dụng định lí Hensel
thì phương trình G t   1996 0 mod 3  có nghiệm duy nhất 1996 t  0; 3  0   .
Chinh phục olympic toán| 67
Bồi dưỡng học sinh giỏi Với 2000 t t    Gt     1998 , 1;3 , 0 mod 3
 khi và chỉ khi tồn tại h ,0  h  8 sao cho 1996 t t  3  h 1998 G t  0 mod 3 0
. Vậy phương trình đồng dư   
 có đúng 9 nghiệm nguyên thuộc đoạn 1998 1;3  1 
 . Từ đó suy ra trong đoạn 2000 1;3  
 có đúng 9 số a sao cho P a chia hết cho 2000 3 .
Câu 70. Tìm tất cả các đa thức f với hệ số nguyên sao cho f nf m  n m . Iran TST Lời giải
Ta dự đoán rằng đa thức thỏa mãn sẽ là k
ax . Nên ta đặt    k f x
x .gx với g 0  0 .
Ta sẽ chứng minh g x là đa thức hằng.
Từ giả thiết f nf m  n m , ta suy ra k   k
n g n m g m  n m . Nên để phản chứng g x C
không phải đa thức hằng thì ta có thể chọn k   k
n g n m g m nhưng không suy ra được n m Ọ H
Từ đó có mâu thuẫn. Vấn đề ta chọn thế nào ? ÁN Nếu chọn cả k k
n m và   k
g n m thì dẫn đến n m , việc chọn như vậy không có ý nghĩa gì.
Vậy liệu ta có thể chọn k k
n m g ngm . Ta sẽ theo dõi lập luận sau. U TOỆ
Xét đa thức h x  x, h 0  0 . Ta sẽ chứng minh với mọi k tồn tại m, p h 0 sao cho LI k
p h m . Thật vậy, giả sử hx  h x .h x ...h x h x 1   2   n   , trong đó
i   bất khả quy và TƯ h 0  h x h x i
0 . Do đó thay vì chọn   bất kì ta có thể chọn luôn   bất khả quy. VÀ Í
Khi h 'x nguyên tố cùng nhau với h x thì tho định lí Bézout ta có tồn tại hai đa thức
P,Q sao cho tồn tại số nguyên a ta có h.P h'Q a . Khi đó theo định lí Schur luôn tồn tại CH P
số nguyên tố p đủ lớn sao cho p a, p h n , p không là ước của h 'n . ẠT
Áp dụng định lí Hensel ta thấy k p h m . Đặt    k f x
x gx , k  0, g0  0 . Giả sử rằng gx khác đa thức hằng. Theo chứng minh
trên thì tồn tại số nguyên tố p sao cho p g 0 và k
p g m với m nguyên dương.
Dễ thấy p không là ước của g p nên theo định lí thặng dư Trung Hoa thì tồn tại n sao cho    mod k n m p   n  
pmod gp Khi đó k
p g n và gpgn nên dẫn đến f pf n .
Do đó p n p g n nên p g 0 là vô lí.
Do đó g x là đa thức hằng. Vậy    . k f x a x .
68 | Tạp chí và tư liệu toán học Đa thức và số học
Câu 71. Cho a, b,c, d, e, f là các số nguyên dương. Giả sử rằng S a b c d e f
ước của các số abc def ab bc ca de ef fd . Chứng minh rằng S là hợp số.
IMO Shortlist 2005 Lời giải Xét đa thức P x
x ax bx c x dx ex f  2 ( )        
Sx Qx R .
rong đó S a b c d e f ; Q ab bc ca de ef fd ; R abc def .
Vì theo giả thiết S Q ; S R nên S P x ,x  .
Ta có S P d  d ad bd c .
Nếu S là số nguyên tố thì một trong 3 số d a, d b, d c chia hết cho S . Nhưng điều đó là
vô lý vì S  maxd a, d b, d   c .
Do đó S là hợp số. ÁN O
Câu 72. Tìm tất cả các đa thức P với hệ số nguyên thỏa mãn Pnn *
|2557  213.2014,n IC TP Thailand 2014 YM Lời giải
Dễ thấy P n *  nP n * 1, và  1  ,n
là những đa thức thỏa m n điều kiện bài OL C
toán. Giả sử P là đa thức thỏa mãn bài toán và * n  : P n  2 0  0  . Ụ P n PH
Gọi p là ước nguyên tố của  0  . Ta có:   0 |2557n P n 213.2014 và P n p |2557n    p  213.2014 0  0  0 .
NH I Do đó      0 2557n p P n p P n 2557p  1 0 0  . CH Mặt khác, vì   0 2557n p P n  213.2014
p  2, 3,19, 53,71, 2557 0 nên  . Do đó |2557p p
 1. Hơn nữa, theo định lý Fermat nhỏ ta có |2557p p  2557 nên |
p 2556 . Suy ra p 2,3,  71 , vô lý.
Vậy chỉ có hai đa thức P n *  nP n * 1, và  1  ,n thỏa mãn bài toán.
Câu 73. Cho P là đa thức hệ số nguyên, có bậc n  1 và k là số nguyên dương bất kỳ. Xét đa thức    k Q x
P x với P được tác động k lần. Chứng minh rằng có nhiều nhất
n số nguyên t sao cho Qt  t .
IMO Shortlist 2006 Lời giải
Bổ đề. Nếu t là số nguyên thỏa mãn Q t  t thì 2
P t  t .
Chinh phục olympic toán| 69
Bồi dưỡng học sinh giỏi Thật vậy. Ta có   2 
     k   k1 
  k1   k P t t P t P t P t P t P t P t .
k1    k P t
P t  Pt  t nên Pt 2
t P t  Pt  k P tk1  P t
Đặt d P t  t . Nếu d   P t 2 0
t P t  P t  t .
Nếu d  0 . Giả sử i là chỉ số nhỏ nhất mà d    i P ti1
P t ,2  i    k khi đó i1   i2    i1    i P t P t P t P t Suy ra i P ti2
P t nên 2
P t  t .
Ngược lại nếu d P t 2
t P t  Pt  k P tk1
P t thì k
P t  t kd t , điều này mâu thuẫn.
Quay lại bài toán, giả sử rằng có n  1 số nguyên t t    t t 1 2 n n1 thỏa mãn
Qt   t ,1  i n i i 1
Khi đó theo bổ đề trên, ta có 2
P t   t ,1  i n i i 1 . C Ọ
Với mọi i, j thỏa mãn 1  i j n  1 ta có t
t Pt Pt  2 P t  2    P t i j i j ij. H
P t P t t t ÁN Nên
i   j j i. Theo bất đẳng thức về giá trị tuyệt đối, ta có: tt P tP t P t
P t P t P t
 P t P t tt n1 1  n1  1  n1  n   n   n1  2   1 n1 1 U TOỆ
Do đó, tất cả các hiệu P tP t i1 
i  đều cùng dấu. LI
Giả sử tất cả các hiệu P tP t i1 
i  đều cùng dấu dương, khi đó TƯ Pt
P t t t   i n ii ii , 1 1    1 VÀ P t
t P t t   i  Í Suy ra  n iii i , 1 1  1   .
P x x P t t n  1
t t    t t CH Do đó, đa thức     1 1 có   nghiệm phân biệt 1 2 n n1 , điều P
này là vô lý vì P x  x  P t t 1 
1  là một đa thức bậc n. ẠT
ương tự cho trường hợp tất cả các hiệu P tP t i1 
i đều cùng dấu âm.
Vậy ta có điều phải chứng minh.
Câu 74. Cho A là tập vô hạn các số nguyên dương. ìm tất cả các số nguyên dương n
thỏa mãn với mọi a là phần tử của A thì 2 n 1! 2! n!
1  a a  ...  a 1  a a  ...  a Serbia MO 2010 Lời giải
Xét một số n thỏa mãn yêu cầu bài toán.
a đặt: P x 2 n
  x x   x Qx 1! 2! n! 1 ... ,
 1  x x  ... x
Theo kết quả trên, từ giả thiết P aQa với vô hạn a ta suy ra P xQ x.
70 | Tạp chí và tư liệu toán học Đa thức và số học Ta thấy rằng n1 x
 1mod Px Do vậy, gọi t i n  
i là số dư trong phép chia ! cho 1 với i n thì Qx 0 t1
x x  ... tn
x Sxmod Px Vì các t n P x ,S x n
i đều không vượt quá
1 và cả     đều là tổng của 1 đơn thức nên để
PxQx thì Px  Sx
Như vậy, ta cần có 0,1!,..., n
! lập thành một hệ thặng dư đầy đủ theo mod n  1
rước tiên, trong n  1 số trên chỉ có một số chia hết cho n  1 là 0 nên ta suy ra n ! không
là bội của n  1, điều này chỉ xảy ra khi n  1 là số nguyên tố.
Với n  2 thì n!  1
  nmod n  1 theo định lý Wilson
Suy ra n  1!  1mod n  1 nên hệ 0,1!,..., n
! không lập thành một hệ thặng dư đầy đủ mod n  1. ÁN
Và như vậy ta phải có n  2, kiểm tra trực tiếp ta thấy n  1 và n  2 đều thỏa mãn. O
Vậy tất cả các giá trị của n thỏa mãn yêu cầu bài toán là n  1 và n  2. IC T P
Câu 75. Cho P,Q là hai đa thức hệ số nguyên không âm, khác đa thức hằng. Xét dãy số YM Pnx  2016  Q n n n
 ,  1. Chứng minh rằng tồn tại vô hạn số nguyên tố p thỏa mãn: OL
ứng với mỗi p , tồn tại số nguyên dương m sao cho p xm . C Ụ Ukraine 2016 PH Lời giải Pn Ta có x  2016  Q n Q n R n n   PnP1  2016  2016    P1 PnP1  2016  2016  2016    NH I rong đó P1 R Q  2016
là một đa thức hệ số nguyên, khác đa thức hằng và R 0  0 . CH
Theo định lý Schur, tồn tại vô hạn số nguyên tố p, p  maxR0 , 20  16 thỏa mãn ứng với
mỗi p , tồn tại số nguyên dương n sao cho p R n . Nếu p n thì |
p Rn  R0 nên |
p R0 , vô lý nên p,n  1.
Mặt khác  p, p  1  1 nên theo định lý thặng dư Trung Hoa, tồn tại số nguyên m sao cho
m nmod p ;m  1mod p  1 Vì |
p Rn và m nmod p nên | p Rm .
Hơn nữa m  1mod p  1 nên P m  P 1modp  1 do đó theo định lý Fermat nhỏ ta PmP1 có |2016 p  2016 . Suy ra p xm .
Vậy ta có điều phải chứng minh!
Chinh phục olympic toán| 71
Bồi dưỡng học sinh giỏi
Câu 76. Tìm tất cả các đa thức P hệ số nguyên thỏa mãn   2p P p
p , với mọi số nguyên tố p . Lời giải
Dễ thấy nếu P là đa thức hằng thì P  1 thỏa m n điều kiện bài toán.
Xét P không phải là đa thức hằng. Theo định lý Schur, tồn tại vô hạn số nguyên tố p thỏa
mãn: Ứng với mỗi p tồn tại số nguyên n sao cho p P n .
Nếu p n thì p P n  P p nên   2p p P p
p , vô lý nên p,n  1.
heo định lý Dirichlet về số nguyên tố, ta có thể chọn số nguyên k sao cho q n kp là số nguyên tố. Khi đó |
p P q  P n nên |  |2q 2n   kp p P q q
 n kp. Kết hợp định lý
Fermat nhỏ suy ra |2nk p
n . Đặt h ordp 2. n k n k C
Nếu có các số nguyên k , k p n p n
k k mod h 1 2 thỏa mãn 1 |2   và 2 |2   thì 2 1   . Ọ
Do đó, ta chỉ có thể chọn k theo một lớp thặng dư nào đó đối với modul h . H
Nhưng, theo định lý Dirichlet, ta có thể chọn k sao cho số nguyên tố q n kp nguyên ÁN
tố cùng nhau với h. Khi đó, vì |
h p  1 nên n k,h  1 , nghĩa là ta có h cách chọn lớp U TO
thặng dư đối với modul h cho k. Vô lý . Ệ
Vậy chỉ có P  1 là những đa thức thỏa mãn bài toán. LI TƯ
Câu 77. Cho P x ,Q x là các đa thức hệ số nguyên khác đa thức hằng. Giả sử rằng đa VÀ
thức P x.Q x  2009 có ít nhất 25 nghiệm nguyên phân biệt. Chứng minh rằng bậc Í
của mỗi đa thức P x ,Q x đều không nhỏ hơn 3. CH P Belarus 2009 ẠT Lời giải
Giả sử T  a ,1  i  25,i i
 là tập hợp gồm 25 nghiệm nguyên của đa thức
Px.Qx  2009 .
Khi đó P a Qa   2009, 1   i i i 25. Suy ra P a Q a i i  2009;  i  2009, 1    25 . Vì 2
2009  7  41 nên 2009 có tất cả 12 ước số nguyên.
Do đó, mỗi số P a , P a ,, P a 1   2 
 25 sẽ bằng với 1 trong 12 ước số nguyên của 2009, và
theo nguyên lý Dirichlet, phải có ít nhất 3 trong số 25 số P a , P a ,, P a 1   2   25 có giá trị
bằng nhau. Không mất tổng quát, giả sử P a P a P a m 1   2   3 .
72 | Tạp chí và tư liệu toán học Đa thức và số học
Khi đó, đa thức P x  m có ít nhất 3 nghiệm phân biệt nên có bậc không nhỏ hơn 3, và do
đó, đa thức P x có bậc không nhỏ hơn 3.
ương tự, đa thức Q x cũng có bậc không nhỏ hơn 3. a có điều phải chứng minh.
Câu 78. Gọi d n là ước nguyên tố nhỏ nhất của số nguyên n, với n   1  ,0,  1 và ta kí hiệu d  1
   d0 ,d1  0. Tìm tất cả các đa thức Px với hệ số nguyên thỏa mãn
P n dn  n dPn Turkey MO 2014 Lời giải
rước tiên, ta có một đánh giá chung cho hai vế của đẳng thức ở đề bài.
a đ biết d n là ước nguyên tố nhỏ nhất của n thì d n  n hoặc d n  . n ÁN
Do vậy ta dự đoán, nếu P x có bậc lớn hơn 1 thì chọn n đủ lớn sẽ có O
P n dn  n dPn IC T
Thật vậy, giả sử P x có bậc không nhỏ hơn 2. P
Chọn n q nguyên tố thì d n  q thay vào phương trình ban đầu ta được YM
P2q  q dPq  q dPq  q Pq OL C P2qq Ụ Suy ra   PqPq 1 PH
degPx
Cho q tiến ra vô cùng, thì rõ ràng vế trái tiến về 2
, còn vế phải tiến đến 1 nên điều NH I này là vô lý. CH
Dễ thấy, P x là hằng số thì không thỏa mãn yêu cầu bài toán.
Cuối cùng, ta xét P x có bậc bằng 1 thì ta viết P x  bx c; b,c  , b  0.
Thay n q là số nguyên tố một lần nữa, thì ta được:
2bq c q d bq c
Ta thấy rằng q d bq c  q nên chọn q đủ lớn thì ta suy ra phải có b là số dương, hay b  1.
Mặt khác ta lại có 2b  1 q c d bq c  bq c nên b  1.
Từ đấy ta phải có b  1, thay lại vào phương trình đề bài thì ta được
n d n  c n d n c
Suy ra d n cd nc, n       .
Từ đây ta suy ra với mọi n đủ lớn nếu n c là số nguyên tố thì n cũng là số nguyên tố và
ngược lại. Ta chứng minh chỉ có c  0 thỏa mãn tính chất này.
Chinh phục olympic toán| 73
Bồi dưỡng học sinh giỏi
Thật vậy, giả sử c  0 thì ta chọn được A  2  c
Xét r là số nguyên tố nhỏ nhất lớn hơn A! 2
Khi đó r A c  2 do tất cả các số từ A  2 đến A c  2 đều là hợp số.
Ta cần có r c r c đều là số nguyên tố, điều này không thể xảy ra vì một trong hai số
này là r c và là hợp số theo cách chọn r.
Vậy chỉ có c  0 thỏa m n, khi đó thì P x  x, thử lại thấy thỏa mãn yêu cầu bài toán.
Vậy tất cả các đa thức thỏa m n điều kiện bài toán là P x  . x
Câu 79. Tìm tất cả các số nguyên dương k thỏa mãn tồn tại đa thức f x với các hệ số
đều nguyên, có bậc lớn hơn 1 sao cho với mọi số nguyên tố p và mọi số tự nhiên a, b
p ab k thì p f af b  k . C Lời giải
Với k  1 thì không khó để thấy rằng ta chọn được   n
f x x với n  1 là đa thức thỏa H mãn. ÁN
Ta chứng minh các giá trị còn lại của k không thỏa mãn yêu cầu bài toán.
Giả sử có k  1 thỏa mãn yêu cầu bài toán. U TO n n Ệ
Xét đa thức f x bậc n bất kì, giả sử f x 1  a x a     x a x a n n ... 1 1 0 LI  k
Ta lập đa thức g xn 0 n 1 n1  x f
a k x a k x  ... n    a k 0 1 n TƯ  x
Với mỗi số nguyên dương a và số nguyên tố p mà a, p  1 thì ta chọn được b VÀ Í
sao cho ab k mod p. CH
Khi đó thì ta thấy   n
g a a f bmod p P Ạ a, p  1 T
Như vậy, ta có với mọi số nguyên tố p mà   thì     n      n g a f a
a f b f a ka mod p
Vì tập các số nguyên tố là vô hạn nên ta có thể cố định chọn p đủ lớn để  max     n p
g a f a ka  3n, a   1,3n
Khi đó thì ta suy ra     n
g a f a ka  0, a   1,3n
Mặt khác, đa thức     n
f x g x kx chỉ có bậc 2n mà lại nhận đến 3n giá trị phân biệt là
nghiệm nên đồng nhất với đa thức không.
Kết hợp với f x có bậc n ta suy ra g x phải là hằng số.
Từ công thức của g x ta suy ra g xna k n .
Ta cần có g 1 f 1  k nên g 1 k tuy nhiên điều này không thể xảy ra khi n  2 hay vô lý.
74 | Tạp chí và tư liệu toán học Đa thức và số học
Vậy k  1 là số nguyên dương duy nhất thỏa mãn yêu cầu bài toán.
Câu 80. Cho P là đa thức hệ số nguyên thỏa mãn P 0  0 và P 0 , P 1 ,...  1 .
Chứng minh rằng có vô hạn số n sao cho P n  P 0 , P n  1  P 1 ,    n . USA TST 2010 Lời giải
Từ điều kiện P 0  0,P 0 , P 1 ,...  1 suy ra P khác đa thức hằng.
Không mất tính tổng quát, giả sử P'1  0 .
Ta chứng minh rằng nếu p là một số nguyên tố bất kỳ sao cho p không là ước của P '1 thì  k
n p thỏa điều kiện bài toán. Vì k|  k
p P p i  Pi với mọi i nên k   k   0,  k p P p P
P p  1  P1,   . ÁN
Mặt khác theo nhận xét trong chứng minh bổ đề Hensel, ta có O Pk p
1 P1 P 1 k p k 1 ' mod     p  IC T
P '1 không chia hết cho p nên  k
P p  1  P1 không chia hết cho k1 p . P
Cuối cùng ta chỉ ra rằng, không có số nguyên tố p q là ước số của YM
k 0,  k P p P
P p  1  P1, OL C
Giả sử ngược lại với mọi i, ta có |  k
q P p i  Pi. Ụ Mặt khác ta cũng có |
q P q i  Pi suy ra   k
P i ap bq  Pimodq với mọi số PH
nguyên a, b . Vì  k
p ,q  1 nên tồn tại các số nguyên a, b sao cho k
ap bq  1 . NH I Do đó |
q P i  1  Pi . CH
P 0  0 nên q P i với mọi i , mâu thuẫn với giả thiết.
Vậy k    k   0 ,  k p P p P
P p  1  P1 , 
 . a có điều phải chứng minh!
Câu 81. Cho p là số nguyên tố và P x là các đa thức bậc d hệ số nguyên thỏa mãn
P0  0, P1  1
 Với mọi số nguyên dương n thì số dư trong phép chia Pn cho p là 0 hoặc 1 .
Chứng minh rằng d p  1 .
Italian Proposal to 1997 IMO Lời giải
Giả sử d p  2 . Áp dụng công thức nội suy Lagrange với  p  1 mốc nội suy
x i,i  0, p i 2 ta được
Chinh phục olympic toán| 75
Bồi dưỡng học sinh giỏi p2 p2 p2
     x    j P x P i
Pp  1  Pi1pi i Cp1 ij i  0 0 j i0 ji i
Do p nguyên tố nên i C  
p i p p 1 mod , 0, 2 1     p2 p1
Pp  1  Pimod p  Pi  0mod pi0 i0 p1
Nhưng từ 2 điều kiện ở giả thiết ta lại có  Pi  k mod p với k 1,2,, p   1 . i0
Điều này mâu thuẫn, suy ra d p  1 .
Câu 82. Chứng minh rằng không tồn tại đa thức P hệ số thực deg P n  1 sao cho
P m là số nguyên tố với mọi số nguyên dương m . C Lời giải
Giả sử tồn tại đa thức P thỏa m n điều kiện đề bài. H
rước hết, ta chứng minh n!P x  x. ÁN n n x j
Thật vậy, theo công thức nội suy Lagrange, ta có: P x   Pi  . i0 j0 i j U TO jini i n LI P i C n 1
Nên n!P x  x x  1   x n       x.  i0 x i
Chọn 2 số nguyên tố p, q, p, q n sao cho P a  p P b * ,
q, a,b  . VÀ Í
Theo định lý thặng dư Trung Hoa thì tồn tại * c  sao cho c
  amodq n!Pc  n!Pa  0mod p CH   
n!P c  0 mod pq P c   bmodq  n!P
c  n!Pb  0modq     ẠT
Điều này vô lý vì P c là số nguyên tố và n!, pq  1 .
Vậy ta có điều phải chứng minh.
Câu 83. Cho n  , n  3 và đa thức f xn n1  x a    ax a x a x n 1 1 0   thỏa mãn 0 chẵn, a a k n k
nk chẵn với mọi 1, 1 .
Giả sử thêm rằng tồn tại hai đa thức g x , h x  x thỏa mãn deg g  deg h , mọi hệ
số của h x đều lẻ và f x  g x.h x , x  .
Chứng minh rằng f x có ít nhất một nghiệm nguyên. Romani TST 2007 Lời giải
Giả sử deg g j,deg h k, 1  j k, j k n
76 | Tạp chí và tư liệu toán học Đa thức và số học g xj
b b x  b x , k
h x c c x  c x 0 1 j   0 1 k
Ta có f x   j k
b b x  b x
c c x  c x 0 1 j  0 1 k
. Đồng nhất hệ số, ta có
b b  b     a b b a j j và 0 1 1 1 1 j k1
Giả sử j  1  0 khi đó theo giả thiết a
b b  1 mod 2  a j 1 k1 chẵn nên 0 j  . Mà a c j b   g x
0 chẵn nên 0 chẵn (mâu thuẫn giả thiết). Do đó 1 mà j 1 nên   có
nghiệm nguyên. Suy ra f x cũng có nghiệm nguyên.
Vậy ta có điều phải chứng minh.
Câu 84. Cho đa thức f x monic, hệ số nguyên, bất khả quy và f 0 không phải là số
chính phương. Chứng minh rằng     2 g x
f x  cũng là đa thức bất khả quy. Romani TST 2003 ÁN O Lời giải
Giả sử ta có phân tích g x  f  2
x   px.qx với px ,q x là 2 đa thức monic có hệ số
IC TP nguyên và bậc không nhỏ hơn 1.      YM
Gọi  là một nghiệm ( thực hoặc phức ) của f x thì p
.q  f   0. Không mất OL
tổng quát, giả sử p     0 . C Ụ k k Đặt p xi
 a x ,a ia   i 0 i i thì . PH i0 i0
Do đó, tồn tại các đa thức hệ số nguyên t, u thỏa mãn t     u  0 .
NH I Do f là đa thức bất khả quy và degu deg f nên theo định lý Bézout, tồn tại số nguyên CH
m và hai đa thức hệ số nguyên s,r sao cho sxux  r xf x  m, x   .
su s
 u 2 
Suy ra s  u  m nên       . 2 m m Đặt  ,  ,,  1 2
n là các nghiệm của đa thức f thì
s 2 t  2 s 2 t  2 s t n 2  n 2 1 1 2 2     1 2 n 2n m
là bình phương của một số hữu tỉ.
Mặt khác f 0     f 0 1 2
n là số nguyên nên
  là số chính phương, trái giả thiết.
Vậy từ đó suy ra điều phải chứng minh!
Chinh phục olympic toán| 77
Bồi dưỡng học sinh giỏi
Câu 85. Cho đa thức hệ số nguyên f xn n1  a x a    x a x a n n 1 1 0 thỏa mãn điều kiện
a a a  a a f x 0 1 2 n và 0 là số nguyên tố thì   bất khả quy.
Tiêu chuẩn Perron Lời giải
Giả sử f x có nghiệm z thỏa z  1 thì n n1
a a z a     z a z a a 0 n n 1 1 1 n , mâu
thuẫn. Do đó tất cả các nghiệm của f đều có module lớn hơn 1.
Giả sử f x  P x.Q x với P,Q là hai đa thức hệ số nguyên có bậc không nhỏ hơn 1.
a f 0  P 0 Q 0 a P 0  1 Q 0  1 0  
    mà 0 là số nguyên tố nên   hoặc   .
Không mất tính tổng quát ta giả sử P 0  1 .
Gọi b là hệ số cao nhất của P z , z ,, z 1 2
k là tất cả các nghiệm của P . 1
Khi đó z z z   k 1 1 2
, mâu thuẫn vì tất cả các nghiệm của P cũng là nghiệm của f . C b
Vậy đa thức f x bất khả quy. H ÁN
Câu 86. Tìm tất cả các đa thức P x ,Q x hệ số nguyên thỏa mãn với dãy số xn  xác U TO
định bởi x  2014, x      P x x Q x n n n , n n , 0 2 1  2  2 2  2 1
thì mỗi số nguyên dương m là Ệ LI
ước của một số hạng khác 0 nào đó của x . n
Việt Nam TST 2014 Lời giải VÀ Í
Ta sẽ chứng minh rằng nếu các đa thức P x ,Qx thỏa mãn đề bài thì chúng đều có bậc CH bằng 1. Thật vậy: P
Xét trường hợp một trong hai đa thức P x ,Q x là đa thức hằng. ẠT
1 Nếu Px  a, x
  thì xn  có dạng:
x  2014, x a, x Q a , x a,... 0 1 2   3
Và dễ thấy mọi số hạng của dãy chỉ nhận một trong ba giá trị 2014, a,Qa.
2 Nếu Qx  a, x
  thì xn  có dạng:
x  2014, x P 2014 ,x a,x P a ,... 0 1   2 3  
Và dễ thấy mọi số hạng của dãy chỉ nhận một trong bốn giá trị 2014, P 2014 , a, P a.
Cả hai điều này đều không thỏa mãn điều kiện đề bài do mỗi số nguyên dương m phải là
ước của một số hạng khác 0 nào đó của dãy số x . n
Tiếp theo, nếu một trong hai đa thức P x ,Q x có bậc lớn hơn 1.
78 | Tạp chí và tư liệu toán học Đa thức và số học
Không mất tính tổng quát, ta giả sử đó là Q x thì rõ ràng khi đó QP x cũng có bậc
lớn hơn 1. Ta thấy nếu R x là đa thức có bậc lớn hơn 1 thì với mọi k  0 lớn tùy ý, tồn
tại x có giá trị tuyệt đối đủ lớn sau cho R x  k x , điều này là dễ thấy do khi x   thì Rx ta có giới hạn lim   x x
Ta sẽ chứng minh rằng tồn tại N đủ lớn sao cho Q P x  P x  x , x   N.
Ta chỉ cần xét hai trường hợp. Nếu P x là bậc 1 thì dễ thấy tồn tại k đủ lớn sao cho
k P x  x , với mọi x  .
Suy ra tồn tại N đủ lớn sao cho:
QPx  k  1 Px  Px  k Px  Px  x với mọi x N
Nhận xét được chứng minh. ÁN x O
Theo giả thiết thì trong dãy số đã cho, phải tồn tại số hạng i lớn tùy ý và rõ ràng ta cũng
phải có j  0 sao cho xN  1 x 2 j
và 2 j có giá trị tuyệt đối lớn nhất trong 2 j số hạng đầu
IC TP tiên của dãy x .n YM
Thật vậy, ta thấy rằng, tồn tại vô số số hạng x2 j thỏa mãn điều kiện OL
x  max x , k   0,2 j 2 jk  C Ụ
Gọi T là tập hợp các chỉ số thỏa mãn. Nếu như trong các số hạng như thế, không có số PH
hạng nào thỏa mãn xN  1  2 j thì với mọi t T      NH Ta có x x N i t T i t 1 với mọi . I
Tuy nhiên, do T vô hạn nên điều giả sử ở trên là vô lý và nhận xét được chứng minh. CH Với x m x   x
2 j là số hạng thỏa mãn điều kiện trên, chọn 2 j 2 2 j thì ta thấy
m QPxx Q P xx P xx x    Q x x 2 j  2 j
  2j 2j  2j 2j và 2n 2  2n 1 2n1
Do đó, trong 2 j  1 số hạng đầu tiên của dãy, không có số hạng nào chia hết cho . m Mặt khác ta có: x        x Q P x Q P x P x P x x x m 2k 2 2k
   2k   2k 2   2k  2k 2  2k 2k 2
Và tượng tự thì x       x P x P x x x m 2k 3 2k 1
  2k 2   2k   2j 2j 2
Từ đây suy ra với k j thì x   mx x x x 2k 2 2 k và 2k 3
2k1 đều chia hết cho
, tuy nhiên 2j1 và x x k j
2 j2 đều không chia hết cho m nên k không chia hết cho m với 2 2.
Do đó, trong dãy đã cho không có số hạng nào chia hết cho . m
Điều mâu thuẫn này cho ta thấy nhận xét ban đầu là đúng và deg P x  deg Qx  1.
Chinh phục olympic toán| 79
Bồi dưỡng học sinh giỏi
Đặt P x  ax b với a  1 và Q x  cx d với a,b,c, d  và ab  0 thì ta có: x    ax b 2n 1 2n  , n   0 x     cx d 2n 2 2n 1 Suy ra x    x     cax ad bcax bc d n  2n 2 2n và 2n 3 2n 1 với mọi 0.
Cả hai dãy này đều có công thức truy hồi dạng y   k ac h   ky h n 1 n với , . n    n k 1
Giả sử k  1 thì công thức tổng quát của dãy này là y k y h n 0   với mọi . n k  1 
Rõ ràng nếu k  1 thì dãy số tương ứng không thỏa mãn, ta xét k  1.
 Nếu h  0 thì ta có n y k y , n
0 rõ ràng không thỏa mãn điều kiện. n     k 1
Nếu h  0 thì do  k,
  1 với mọi n nên giả sử t là số mũ lớn nhất mà t k h k  1 
thì các số nguyên dương có dạng s
k với s t đều không là ước của bất cứ số hạng C Ọ
nào của dãy, không thỏa mãn. Từ đây, suy ra k  1 hay ac  1. H
Cuối cùng, ta chỉ cần xét hai trường hợp:
Trường hợp 1. Nếu P x  x a Q x  x b với a, b  thì bằng quy nạp, ta chứng ÁN minh được x   k a b x      a k a b k 2014 k 2014 2   và 2 1   U TO
Dễ thấy rằng nếu a b  0 thì dãy này không thỏa mãn. Ệ LI
Nếu như a b  0 thì gọi S,T , R lần lượt là tập hợp các ước nguyên của a b, 2014 và 2014  . a
Ta xét các trường hợp sau: VÀ   Í
Với m S thì giả sử a b chia m t với t  0 do đó khi k chạy qua một hệ thặng dư
đầy đủ mod m .thì tồn tại một số hạng của dãy chia hết cho m và số lượng các số hạng CH P
như thế là vô hạn. Rõ ràng số các số hạng bằng 0 của dãy là hữu hạn, không quá hai nên Ạ
tồn tại số hạng khác 0 của dãy chia hết cho . m T
Với m S m T , R thì dãy số tương ứng không thỏa mãn.
Với m S van m T hoặc m R thì tương ứng, mọi số hạng có chỉ số chẵn hoặc mọi số
hạng có chỉ số lẻ của dãy đều chia hết cho m và dễ thấy, tồn tại số hạng khác 0 của dãy chia chết cho .
m Do đó, các số a,b phải thỏa mãn S \T  S \R   hay mỗi ước của
a b phải là ước của 2014 hoặc là ước của 2014  . a
Trường hợp 2. Nếu P x  x a Q x  x b thì cũng có lập luận tương tự vì các số
hạng của dãy tương ứng khi đó là x   k a b x       a k a b k 2014 . k 2014 2   và 2 1  
Điều kiện của a, b a b  0 và mỗi ước của a b phải là ước của 2014 hoặc là ước của a  2014.
Vậy tất cả các đa thức P x ,Q x cần tìm là:
80 | Tạp chí và tư liệu toán học Đa thức và số học
Px  x a,Qx  x b trong đó a,b  thỏa mãn a b  0 và mỗi ước của a b
phải là ước của 2014 hoặc là ước của a  2014.
Px  x a,Qx  x b trong đó a,b  thỏa mãn a b  0 và mỗi ước của
a b phải là ước của 2014 hoặc là ước của a  2014.
Câu 87. Chứng minh rằng không tồn tại đa thức Pxn n1  a x a      x a x a x n n n ... 1 1 0   bậc 1
sao cho P 0 , P 1 ,... đều là số nguyên tố. Lời giải
Giả sử tồn tại đa thức thỏa mãn yêu cầu bài toán.
Khi đó, a P 0 0
  là số nguyên tố.
Mặt khác, ta có a P ka , k   0, .  0  0  ÁN O
Nhưng vì P ka
P ka a , k   0.
0  là số nguyên tố nên  0  0
Điều này suy ra đa thức Q x  P a x a 0  có vô hạn nghiệm. IC T 0 P
Dẫn đến Q x  0 hay P a x a , P x 0 
0 mâu thuẫn với việc đa thức
  có bậc ít nhất là 1. YM
Vậy giả sử phản chứng là sai nên ta suy ra không tồn tại đa thức thỏa mãn yêu cầu bài OL toán. C Ụ
Câu 88. Cho đa thức P xn n1  x a     a a   ax a x a P x x n ... , 1 1 0     với chẵn và PH 0 n k k
chẵn, với mọi k  1, n  1. Giả sử P x  Q xR x , với Q x , R x là các đa thức hệ số NH I
nguyên khác hằng, deg Qx  deg Rx và tất cả các hệ số của R x đều lẻ. Chứng CH
minh rằng, đa thức P x có nghiệm nguyên. Lời giải
Định nghĩa: P xn n1  x a        x a x a a a mod 2 , i 1, . n n ... 1 1 0 với i i  
Khi đó thì ta có P x  Q x.R x
Đặt deg Qx  r,deg Rx  s,r s và   r
Q x x  ...  b x b . 1 0
Khi đó nếu r  1 thì n n1 x a x
  a x a x b x    b x b x x       x n ...  r r 1 r ...  s s 1 ... 1 1 1 0 1 1 0  a       b b  ... b b
Đồng nhất hệ số của r 1 x  và s 1
x  thì ta được r 1 r r 1 1 0  a       b b  ... b 1 s 1 r r 1 1 Vì a   a b  1 Q 0 R 0 P 0 r 1 s1 nên 0 hay
  là số lẻ, vì   là số lẻ nên dẫn đến   là số lẻ, mà P0  a r P x
0 là số chẵn, điều này dẫn đến vô lý. Vậy
1 chứng tỏ   có nghiệm nguyên.
Chinh phục olympic toán| 81
Bồi dưỡng học sinh giỏi n
Câu 89. Chứng minh rằng với mọi số nguyên dương n thì đa thức P x  x x2 2  1
là đa thức bất khả quy trên .
Romanian Team Selection Test 1998 Lời giải
Với n  0 dễ dàng kiểm tra là đúng. Do đó ta giả sử n  1. Ta liên kết mỗi đa thức Gxn n1  a x a      x a x a x n n ... 1 1 0  
Với một đa thức G xn n1  a x a      x a x a x n n ... 1 1 0 2  
Với các hệ số lấy trong trường 2 modulo 2. n2 n 1 n 2  2 2 2 2 2 2n     Mà do  2
x x     2 x x    2 x x     2 1 1 1 ... x x  1       n
Nên suy ra P x  x x  2 2 1 C
Bây giờ giả sử P x khả quy, tức là có thể phân tích được P x  G xH x với Ọ H
G x , H x x. ÁN
Từ đây ta suy ra P x  G x.H x, lại vì 2
x x  1 là đa thức bất khả quy trên 2 nên suy n pp ra              2 2 2 1 , 1 ,1   2n G x x x H x x x p  1 U TOỆ  2 p LI Gx  
x x1 2Ux Điều này dẫn đến  2n p TƯ H x  
 2x x1 2VxU x ,V x VÀ Với
    là các đa thức hệ số nguyên. Gọi  là một nghiệm của phương trình Í 2 x x  1. CH
Thay x bởi  trong đẳng thức: P Ạ 2n p 2n 2  2   2 p  T
Px  x x  1  x x  1  2U x x x  1  2V x      
Thì ta nhận được  U  V   U V  1 2 2 .2  2
Nhưng ta nhận thấy rằng U x.V x là đa thức hệ số nguyên và 2     1 nên
U V  là một số phức có dạng a b, a,b  .
Do đó đẳng thức U V  1  không thể xảy ra. 2
Vậy điều giả sử là sai và từ đó ta có điều phải chứng minh.
82 | Tạp chí và tư liệu toán học Đa thức và số học
Câu 90. Giả sử n là số tự nhiên lớn hơn hoặc bằng 2 và P xn n1  x a     x a x n ... 1 1 1
là đa thức hệ số nguyên dương. Giả sử a a k n k
nk với mọi 1, 1. Chứng minh rằng y P  x
tồn tại vô hạn cặp số nguyên dương x, y sao cho:  * . x P y  Lời giải
Trước tiên, ta nhận thấy rằng 1, P 1 là một cặp số thỏa mãn *.
Giả sử có một số hữu hạn các cặp số nguyên dương x, y thỏa mãn yêu cầu bài toán, ta
chọn cặp x, y với x y y có giá trị lớn nhất.  Py 
Ta chứng minh cặp  y,  cũng thỏa mãn * x   PyPy
Do x, y thỏa mãn * nên có   hiển nhiên Py ÁN x x O  Py 
Ta cần chứng minh y P   1 IC T x   P
Do y P x  x, y  1 suy ra tồn tại z sao cho xz  1mod y. YM Py  P y  OL
Theo tính chất a b P a  Pb thì ta có  zPy   P
  PzPy 2 C z x   Ụ Py 
zPyPyxzPy  PH Nhưng do x x xz  1 mod y y P y xzP y x P y xzP y x P y NH Vì   nên    . Ngoài ra     do   I
Py  xzPy CH
Mà x, y  1 nên ta có y xPy 
Từ đó theo 2 thì ta được P
  PzPy  Pzmody ,do Py  1mod yx   n  1  n  1  Vì a a x P
P x y x P      y P z k
nk nên ta suy ra      x   x   Py   Py  Hay y P
 . Vậy cặp  y,
 thỏa mãn yêu cầu bài toán. x   x   P y
Ngoài ra thì ta có P yn 2  
y  1  y yx   y x
Mâu thuẫn với cách chọn y là lớn nhất.
Vậy điều giả sử là sai và từ đó ta có điều phải chứng minh.
Chinh phục olympic toán| 83
Bồi dưỡng học sinh giỏi
Câu 91. Chứng minh rằng, với mỗi số nguyên dương n tồn tại đa thức P x  x bậc
n sao cho P 0 , P1 ,..., Pn phân biệt và tất cả các số đó đều có dạng 2.2019k 3, k    . Lời giải
Bài toán chứng minh tồn tại đa thức, do đó phải chỉ ra đa thức thỏa mãn. Hãy để ý đến
yêu cầu P 0 , P 1 ,..., P n nhận giá trị đặc biệt, do đó bài toán này mang tư tưởng của
nội suy Lagrange. Tuy nhiên không hoàn toàn là nội suy Lagrange, vì các giá trị của chúng
chỉ có dạng chứ không có giá trị cụ thể. Do đó ta sẽ lấy đa thức đơn giản nhất của nội suy  x  x  x
P x  a a    a    ... a   , a  , i   1,n 0 1 2 1   2 n i  n
Lưu ý rằng đa thức P x trên nhận giá trị nguyên, nhưng hệ số của chúng là hữu tỉ. Rõ k k
ràng muốn kết quả của chúng có dạng 2.2019  3, đặt a  2019 thì dạng 2a  3. Do đó C Ọ
việc đầu tiên là cho các số P i có dạng k
a đã. Dẫn đến các hệ số a , a ,..., a 0 1 n là hằng số H
được không? Nếu chúng là hằng số thì    .2x P x k
lai không được dạng mong muốn. ÁN
Nhưng ý trên cho ta suy nghĩ, để các hệ số ai cùng tham gia vào khai triển Newton. Do đó i
ta sẽ chọn a   k a i 1 . U TOỆ
Khi đó, với x  0, n thì ta được: LI     Px 0 1 x n x n ka  1 k
a  1   ... ka  1 k
    a  1  1 kn   a TƯ         1 n      VÀ
Đến đây thì ta gần thu được kết quả mong muốn. Vì khi đó ta chỉ cần chọn đa thức Í
Qx  2Px  3 thì sẽ có được đa thức thỏa mãn bài toán. Tuy nhiên đã đúng chưa? Hãy CH
lưu ý rằng đa thức P x hệ số hữu tỉ nên đa thức Q x cũng có hệ số hữu tỉ. vậy ta phải P ẠT
cải tiến để đa thức Q x có hệ số nguyên. Lưu ý rằng đa thức P x hệ số hữu tỉ, với các
hệ số có mẫu lớn nhất là n .
1 Do đó ta sẽ phải nhân vào, nhưng không làm thay đổi một
lượng. Lưu ý các hệ số của P x đều có nhân thêm  k
a  1. Vậy phải chọn k như thế
nào? Chọn làm sao để  k
a  1 chia hết cho n!. Việc này quá to tát, không thể được? Tuy
nhiên ta sẽ chọn được k để  k
a  1 chia hết cho một phần tử nào đó của n!. Liên quan
đến lũy thừa ta nghĩ ngay đến định lý Euler. Phân tích n!  n .n n 1 2 trong đó 1 là ước
nguyên tố lớn nhất của n ! mà nguyên tố cùng nhau với a, khi đó tất nhiên n2 và a cũng
có chung ước nguyên tố.  1 n  Khi đó thì ta có a
 1modn1 
Do đó ta chọn k  n k n a  1. 1  thì 1
84 | Tạp chí và tư liệu toán học Đa thức và số học Tiếp đến ta phân tích 1 2
n p .p ... i p 2 1 2 i
Đặt s  max , ,..., n
p , p ,..., p n . s 1 2 i  thì
2 và a có chung ước nguyên tố 1 2 i nên 2
Từ đây dẫn đến ! s k n a a  1
Đến đây thì bài toán được sang tỏ, bằng cách lấy    2 s Q x
a P x  3 n x i
Với P x    k
a  1 là đa thức chúng ta cần tìm. i0  i
Câu 92. Chứng minh rằng tồn tại tập vô hạn các điểm ..., P , P , P , P , P , P , P ,... 3 2 1 0 1 2 3 
trong mặt phẳng thỏa mãn tính chất: Với ba số nguyên a, b,c phân biệt thì các điểm P P P
a b c
a , b , c thẳng hàng khi và chỉ khi 2014.
USA MO 2014 Problem 3 ÁN Lời giải O
Trước hết, ta có thể đặt a a  671, b b  671,c c  671 1 1 1
ta có thể đưa điều kiện bài toán
về thành a b c  1.
IC TP Một điểm P f i g i i
 ,   gồm hai thành phần hoành độ và tung độ. Để chứng minh tồn tại YM
dãy vô hạn, ý tưởng tự nhiên là đi tìm biểu diễn tường minh cho hai hàm f i và g i. OL
Và tìm sự biểu diễn hợp lý nhất, đơn giản nhất chính là tìm trong lớp hàm đa thức. C Ụ
Trước tiên, ta sẽ tìm điều kiện để P P P
a , b , c thẳng hang. P f a g a P f b g b P f c g c a  ,
, b   ,  , c   ,   PH Ta có    
P P f b f a g b g aa b     ,    
NH I Khi đó thì ta được 
P P f c f a g c g aa c     ,     CH
f c  f agc  ga
Khi đó ba điểm P P P
a , b , c thẳng hàng khi và chỉ khi
f b  f agb  ga
Dẫn đến f bg c  f bg c  f ag c   f ag a
f cgb  f cga  f agb  f ag a Đến đây ta đặt:
F a,b,c  f ag b  f bg c  f cg a   f ag c  f cg b  f bg a
Khi đó F a,b,c là đa thức ba biến và F a, a,c  F a,b,b  F a,b, a dẫn đến
F a,b,c  a bb cc aG a,b,c
Mặt khác chúng ta muốn ba điểm P P P
a b c
a , b , c thẳng hang khi và chỉ khi 1 hay ta
dịch qua ngôn ngữ của đa thức là F a, b,c  0 khi và chỉ khi a b c  1.
Điều này cho ta thêm một nhân tử của F a,b,c khi đó thì
F a,b,c  a b c  1a bb cc aG a,b,c
Chinh phục olympic toán| 85
Bồi dưỡng học sinh giỏi
Để đa thức này đơn giản nhất thì ta chọn G a,b,c   1, khi đó thì
F a,b,c  a b c  1a bb cc a
Khai triển đa thức này, dẫn ta đến tìm đa thức f x và g x sao cho
        3 3 2 2       3 2    3 2 f a g b f b g a ab a b a b ab a b b b a a
Đồng nhất thì ta chọn f x  x g x 3 2 ,
x x , khi đó thì các điểm Pi của đề bài ban đầu có 3 2
tọa độ là P i i   i i
 671, 671  671 
Từ đây ta dễ dàng suy ra rằng, tồn tại tập vô hạn điểm thỏa mãn đề bài.
Câu 93. Cho x x  ...  x n n  1 2 n là , 3 số thực thỏa mãn:
x x x x x x  ...  x x 2 1 3 2 4 3 n n1 P x
x , x ,..., xn.
Giả sử đa thức   có n nghiệm thực là các giá trị 1 2 Chứng minh rằng giá C Ọ
trị lớn nhất của P x đạt được tại một điểm x x x n , n . 0  1  H
Russia All – Russian Olympiad 2010 ÁN Lời giải
Vì đa thức P x có n nghiệm là x , x ,..., x
P x a x x
x x ... x x 1 2 n nên    1   2   n  U TO
Do đó bài toán quy về tìm giá trị lớn nhất của A x x x x ... x x x   x x n , , 1 2  1 n Ệ LI
Giả sử phản chứng A đạt max tại x x i n   x i , 0
 1 i với 2, 1. Khi đó ta tịnh tiến giá trị TƯ này về   x    x x n 1 i 0 thì    x     vì x x x x x i , i . i 0 do 0  1  VÀ n 1 0 Í    x x         x x x x x x x n n 1 i 0 n i 0 n
n1 điều này đúng do giả thiết thì CH
x x x x    x x P i 0 i i 1 n n1 Ạ        T Ta có x x x k i k k , 1, 2. 0
x x x x
x x    x x x   k i k i k 0  0   i n1  Ta có x x        x    x x n i .  x x x 0 n 1 n 1 0 i vì 1 0
Tương tự thì ta có x x       x x x 0 n 2 n 2 0 i1 Vì   x            x x x x x x x x x x n 2 n 1 n 2 i 0 i 1 0 n 1 n 2 i 1
i đúng theo giả thiết.
Tiếp tục như vậy thì ta có x x       x x x n n i , ... 0 3 3 0 3
Tổng quát luôn thì ta được   x x         x k i n k n i k 0, , 1 1 0
Cuối cùng thì x     x     x x x x n  i n i * 1   0   0 1   
Thật vậy * khai triển ra thì được:
86 | Tạp chí và tư liệu toán học Đa thức và số học * 2 2
x   x x          x x x x x x x x n n i 1 i 1 n 0 n i 1 0 0 i1
x   x    x x    x n   2 2 0 0 i 1  0 
x    x x     do x n i 0 0 1 0  x x      x x x x n n 1 i i 0 i1  x x    x x True n n i i 1 1  
Kết hợp tất cả các điều trên thì ta được:
  x ...   x          . x  ... x . x  ... x 1   i 2   n 1   i   i 1   n
   x ...   x          x  ... x x x 1   i 2   n 1   i   i 1   n
 x x ... x x      x x x x x x x x i n ... 0 1   0 2   1 0 
i 0  0 i 1 n 0 
 x x ... x x      x x x x x x x x i . n ... i . i ... 0 1   0 2   0 1   0   0 1   0 n
Điều này mâu thuẫn với giả sử A đạt giá trị lớn nhất tại x0
Vậy điều giả sử là sai hay từ đó ta suy ra được điều phải chứng minh. ÁN O
Câu 94. Cho P xn n1  a x a     x a x a n n ... 1 1
0 là đa thức hệ số nguyên thỏa mãn điều IC T
kiện P r   P s  0 trong đó r,s là các số nguyên thỏa mãn điều kiện 0  r  . s Chứng P
minh rằng tồn tại k 0,1, 2,..., 
n sao cho a  s k . YM Lời giải OL C
Do P s  0 nên P x  x sQ x ,Q x x,deg Q x  n  1 Ụ Đặt   t
Q x x Rx , với R0  0,t  , R x x,deg R x   n t 1 với biểu thức PH
tường minh của R x là R xnt1  b      x b x b n t ... 1 1 0 NH t I
Khi đó thì 0  P r   r sr R r   R r   0 CH
Chứng tỏ R x có nghiệm nguyên dương r khi đó bắt buộc các hệ số của R x phải đổi dấu.
Trường hợp 1. Nếu b  0 t nt1
P x x s x b      x ... b x b 0 thì khi đó      n t 1 1 0 
Đến đây thì a  sb  s a t , 0
tức t chính là hệ số cần tìm.
Trường hợp 2. Nếu b  0 R x 0 do các hệ số của
  phải đổi dấu nên sẽ tồn tại
i 0,1,...,n t  
2 sao cho: b   b i 0 i1
Ta viết lại như sau: P x  x st x nt1 i1 b         x ... i b x b x ... b x b n t 1 i 1 i 1 0 
Khi đó thì ta được a     a   b sb s t i 1 i i 1
hay ti1 chính là hệ số cần tìm
Vậy trong mọi trường hợp đều tồn tại k 0,1, 2,..., 
n sao cho a  s k .
Chinh phục olympic toán| 87
Bồi dưỡng học sinh giỏi
Câu 95. Cho P x ,Q x là các đa thức hệ số nguyên. Đặt a n n n ! . Chứng minh rằng Pa Pnn  nếu   n  thì  n
 và Qn  0 . Q a Qn , n  ,
Canadian Mathematical Olympiad 2010 Lời giải PxR x
Chia đa thức P x cho Q x thì ta được  
QxAx   Qx
Trong đó: Ax , R x là các đa thức hệ số hữu tỉ và Rx  0 hoặc
deg Rx  deg Qx. B x
Quy đồng tất cả các hệ số của đa thức Ax thì ta có thể viết Ax    , b
Với Bx là đa thức hệ số nguyên, còn b là bội chung nhỏ nhất của tất cả các mẫu số của C
các hệ số trong Ax ,b  0. Ọ H
Nếu R x không đồng nhất 0 thì, với chú ý nếu k nguyên thì hoặc Ak  0 hoặc ÁN Ak 1  . b U TO Rk 1 0   Ệ
Nhưng với k đủ lớn thì
Qkb LI P an  TƯ
Dẫn đến với n đủ lớn thì
không thể là số nguyên nên ta phải có R x  0. Qan  VÀ
PxBx Í Khi đó thì 
biểu diễn này với những giá trị x Q x  0. Qxb CH P
Bây giờ với n0 là số nguyên cho trước, khi đó tồn tại vô hạn số nguyên dương k sao cho: ẠT a n b k mod 
Có thể lấy các giá trị k tn b n ,t     , 0 0
với t nguyên dương đủ lớn thì k là số nguyên dương. Bak  Khi đó thì
là số nguyên hay b Bak b Pn
Từ đó suy ra b Bn dẫn đến là một số nguyên. Qn
Vậy từ đây ta có điều phải chứng minh.
88 | Tạp chí và tư liệu toán học Đa thức và số học
Câu 96. Cho P x là đa thức bậc n  5 với hệ số nguyên và Pxn n1  a x a     x a x a n n ... 1 1 0
Giả sử P x có n nghiệm nguyên phân biệt là 0,  ,..., n. 2
Tìm tất cả các số nguyên k
sao cho thỏa mãn P P k  0.
French Team Selection Test 2005 Lời giải
Nếu số nguyên k P P k  0 thì chứng tỏ P k là một nghiệm của P x.Nhưng các
nghiệm nguyên của P x đã cho là 0,  ,..., 
P k  0, ,...,n . n . 2 Do đó phải có    2  Ta phân
tích P k dưới dạng:
Px  a x x   x   n  ... 2   n     
Nếu P k 0 chứng tỏ k là một nghiệm của P k khi đó k 0, ,..., n . 2  ÁN
 Nếu Pk   a k k   k     n ... n * 2 khi đó thì  2    2   . O
Từ * thì k  0, k   ,..., k  
k    k    2 n ta cũng có i j với mọi i j IC T          P
Mặt khác vì k  5 nên k k k k n . . . ... 2 2 3 n
k . k   . k   . k   . k    4  1.1.2. 1 . 2 YM 2 3 4 5
k    k    k    k   và k có thể bằng với một trong số k   với i  1, 5 OL 2 3 4 5 i C   k Ụ 2 Từ đó   4 *
k . k    2
mặt khác từ   thì ta được 2 2    k   PH  2 2 Mặt khác k   k     k  2 2 và 2
2 đẳng thức nếu có xảy ra thì 2 tuy nhiên lại mâu
NH I thuẫn với * do đó ta phải có 2 k   ,2 k   2 2 2 CH 2 2  Nhưng từ đây ta suy ra 2
k . k      .     4 2 2 2 do 4 4 2 
Nhưng từ đây theo * thì a k   k   k     n   ... n  2 1 3 4
k k  2 
Nhưng cũng theo * thì a k  
k   ... k  
k   . k   . k   n 3   4   n  3 4 5  2  1.1.2  1. 1  . 2   1. 1  .2  1.1. 2 
Hai kết quả trên là mâu thuẫn với nhau.
Chứng tỏ không tồn tại số nguyên k để P k   .
2 Hoàn toàn tượng tự, ta cũng chứng
minh được không tồn tại số nguyên k để P k   , i   2, . n i
Từ đó suy ra các số nguyên k cần tìm là k 0,  ,..., n . 2 
Chinh phục olympic toán| 89
Bồi dưỡng học sinh giỏi
Câu 97. Tìm số nguyên dương n nhỏ nhất sao cho tồn tại đa thức P x bậc n có hệ số
nguyên thỏa mãn: P 0  0, P 1  1 và với mọi *   thì
P2P1P là bội của p với p là số nguyên tố
Doãn Quang Tiến Lời giải
Ta sẽ chứng minh n p  1 thỏa mãn yêu cầu bài toán.
Ta sẽ giả sử phản chứng rằng 1  n p  2 và tồn tại đa thức P x thỏa mãn bài toán.
Từ đây, ta sử dụng công thức nội suy Lagrange thì ta được: p2 p2       x i P x P k   k0
i0,ik k i p2 p2 p2 p  1  i k1
Từ đây ta suy ra P p  1  Pk    1
  Pkk Cp 1 1    k0
i0,ik k i k0 C k k
Bây giờ, ta sẽ đi chứng minh một kết quả sau C       p k p p 1 mod , 0, 2 1     H
Ta sẽ chứng minh một cách nhanh chóng bằng phương pháp quy nạp như sau: 0 ÁN Với k  0 thì 0 C      p p 1 1 1 mod 1   
 điều này là hiển nhiên. k
Giả sử mệnh đề đúng với k tức là ta có k C    p p 1 mod , 1   
 ta sẽ chứng minh mệnh đề U TOỆ
cũng đúng với k  1. LI
Mà ta có kết quả cơ bản của hệ số nhị thức như sau: k1 k1 k C    C C p 1 p
p1 nên từ đó ta suy ra: TƯ k k k CCC              p p p p
0  1k  1. 1k  1k 1 1 1 mod 1 1   VÀ Í
Vậy từ đó theo nguyên lý quy nạp toán học thì kết quả trên được chứng minh.
Từ đây, kết hợp với kết quả của 1 thì ta được: CH P p2 p2 k1 k1 k k
P p  1  1 PkC      P k p 1 1 1      T k0 k0 p2 p2 p1
 12k1 Pk   Pkmod p  Pk  0mod p 2 k0 k0 k0 p1 p1 p1
Nhưng mà ta có  Pk  P0  P1 Pk  1 Pkk0 k2 k2
do giả thiết cho P 0  0, P 1  1.
Và từ giả thiết bài toán là P   2P   1 P  là bội của p nên ta suy ra được:
P k  0,1,2 mod p , k   2, p  1 p1 p1
Do đó ta suy ra được Pk 1  Pk  1, p  1,2p  3  0mod pk0 k2
Ta thấy điều này là hoàn toàn mâu thuẫn với 2
90 | Tạp chí và tư liệu toán học Đa thức và số học
Vậy giả sử phản chứng là sai nên ta phải có n p  1.
Với n p  1 ta xét P xp 1 x  
thỏa mãn yêu cầu bài toán.
Do đó giá trị nhỏ nhất của n p  1 và ta hoàn tất bài toán.
Câu 98. Chứng minh rằng với mọi m
tồn tại đa thức P x
m   có hệ số hữu tỉ thỏa mãn với mọi * n  thì 2m1 2m1 2m1 1  2  ...  nP n n m   1
Đề chọn đội tuyển VMO Đà Nẵng 2017 Lời giải x
Với m  0 thì ta chọn được đa thức P x  0   2 2  x
Với m  1 thì ta chọn được đa thức P x  1   4  ÁN
Giả sử khẳng định đúng đến m  1 thì ta xét đa thức: O
x  x 2 2 x  1  2 2 x  2 ... 2 2 x m  2m1 2m1 2m3  xa     x a x ... a x m 1 m 2 0 2m1 2m3 2m1 IC T
 x  a      x a x ... a x x m 1 m 2 0 P
Thay x lần lượt bởi 1, 2,..., n thì ta được: YM n
im 1   a P n n   i i i   n 1 2m 1 * OL i1 i0 i1 C              Ụ
Mà ta có xx x 1x 1...x mx m x mx m 1...x m 1x m 1 PH
Lại có x  
 xx m  1  xx m  1 2m 2  
NH I Bây giờ ta đặt: dxmxm1xxmxm1...xm1 CH
dx m  1  x m  1x m...x m  x m  1x  x 1  d
 x m  d x m  1 2m 2   n 1
Do đó thì ta được i 
dn m  dm    i1 2m 2 1 
dn m 1 
nmnm 1...nm 1 2m  2 2m  2
Mà ta có n in i   2 2
1  n i n i nn  1  i i  1 , i   0,m n 1 m
Từ đây suy ra i  n
 n  1  i i  1 Qn n   m   1  . i1 2m 2 i0 Khi đó Q *
m là đa thức có hệ số hữu tỉ và từ   ta chọn:
P x Q x a     P x
a P x a P x m   m   m m m m ... 1 1   2 2   0 0  
Và từ đây ta có điều phải chứng minh.
Chinh phục olympic toán| 91
Bồi dưỡng học sinh giỏi
Câu 99. Cho P x ,Q x , R x là ba đa thức hệ số thực thỏa mãn:
P Qx  PRx  c, x
  với c const
Chứng minh rằng: P x là hằng số hoặc Qx  R x là hằng số.
Đề chọn đội tuyển VMO Hà Nam 2017 Lời giải
Bỏ qua trường hợp tầm thường P x là hằng số. Bây giờ ta xét Pxn n1  a x a       x a x a a n n n ... , n 0, 1 1 1 0 Qxm m1  b x b     x b x b m m ... 1 1 0 Rxk k1  c x c     x c x c k k ... 1 1 0
Giả sử ta có m k thì từ:
P Qx  PRx  c, x
  với c const  * C Ọ
Đồng nhất hệ số bậc cao nhất của * thì ta được: n a b  0. n m H Do đó hoặc b
deg R x   deg Qx  0 m k m 0 suy ra   hoặc . ÁN
Khi m k thì a n n
b c   b cn m m  0 m
m với n là số lẻ. Khi đó thì ta được: U TOỆ Qxm m1  b x b      x b x b b m m ... , m 0 1 1 0 LI Rxm m1  b x c      x c x c b m m ... , m 0 1 1 0 TƯ
Bây giờ xét đa thức H x  a n Q xn
R x a Q x R x S x n
  n       ,trong đó VÀ n1 n2 n3 2 n2 n1 Í
Sx  Q x Q xRx  Q xR x ...QxR
x R xmn 1 CH Xét hệ số của x
 trong Sx thì ta được: P n2 n   1 n 1 n 2 n1 Ạ b
b b ... bb  b  nb  0 T
Do đó deg Sx  mn  1.
Ta sẽ giả sử phản chứng rằng deg Qx  Rx  1.
Khi đó thì ta có deg H x  mn  1  1 n1
Ta xét đa thức T x  P Qx  P Rx  H x  a i Q xiR x i   i0
Nhận thấy rằng: deg T x  mn  1 rõ ràng điều này là vô lý.
Từ đó ta phải có deg Qx  Rx  0 hay Q x  R x là hằng số.
Vậy từ đây ta suy ra điều phải chứng minh.
92 | Tạp chí và tư liệu toán học Đa thức và số học
Câu 100. Chứng minh rằng tồn tại các đa thức hệ số nguyên S ,S , 1 2 tương tứng với
các biến x , x ,, y , y , n  1 2 1 2
thỏa mãn với mọi số nguyên 1 n n n   d d d
dS  dx y d d d  * | d n | d n  
Với hàm tổng chạy qua các ước nguyên dương d của n .
Chú ý. Lưu ý rằng ta chỉ xét đến các đa thức trong trường x. Ví dụ, xét hàm
S x y
S x y x y n  1 1 1 và 2 2 2 1 1 trong trường hợp 2 ta được 2
S  2S   2 2
x y  2  x y * 1 2 1 1 
 2 2  là hàm   thỏa mãn yêu cầu bài toán.
2018 Brazil 4th TST Day 2 Lời giải
Ta chứng minh rằng Sn có hệ số nguyên bằng phương pháp quy nạp
Với n  1 ta được hiển nhiên đúng ÁN O
Giả sử điều ta cần chứng minh vẫn đúng đến n  1 , tức các đa thức S ,S ,,S 1 2 n1 đều là đa thức có hệ số nguyên
IC TP Gọi p là số nguyên tố thỏa mãn v nk 0 p
, từ đây ta có thể nhận thấy rằng: n n n YM   k d d d
p dx y    d S d d . d    OL | d n | d n,d n C
Đặt n pm , suy ra với mỗi giá trị d không phải là ước số của m sẽ bị triệt tiêu trong tổng Ụ
trên khi lấy modulo k
p , do đó đủ để ta thấy được điều sau: PH n n n   k d d d
p dx y   dS NH d d d I | d m   |dm p p p p CH
Đặt T x x y y   S x x y y ii  , , , , , , , , , , , 1 2 1 2  i  1 2 1 2 
Áp dụng giả thiết bài toán cho T , p x , p y i i
i , tổng trước đó trở thành: pm pm pm m pm     d d d d d
dx y dS  dT S d d d d d  | d m   |dm | d m   Ta sẽ chứng minh k
p có thể chia cho mỗi phần trong tổng này. m 1 Thật vậy, đặt l
p q với p q , dễ dàng thấy được l 1 l l p q p q p T S    d d d Theo giả thiết quy nạp: pq
A Sd là đa thức hệ số nguyên, theo tính chất của tự đồng cấu
Frobenius (Frobenius Endomorphism), ta có được q
T A Bp d
với hàm B có hệ số nguyên, l l   l lp ip
vậy ta có được điều sau l 1 pABpp
A   Bpp i A   1 lipi  Lại có l 1
p  chia hết cho mỗi phần trong tổng này vì
Chinh phục olympic toán| 93
Bồi dưỡng học sinh giỏi l l l   p  
p p   i i 1
v p    v p  
  i l v i l   i v i p p p   1 p   1  ,  i    i i 1      
Điều này đúng với i  1 , ta có điều phải chứng minh.
Nhận xét. Đây là một bài toán khó với việc sử dụng lý thuyết về hàm tự đồng cấu rất ít
gặp trong các bài toán dự thi Olympic, tự đồng cấu Forbenius sẽ được tìm hiểu rõ hơn
trong chương trình đại học. C Ọ H ÁN U TOỆLI TƯ VÀ Í CH P ẠT
94 | Tạp chí và tư liệu toán học TÀI LIỆU THAM KHẢO
Dưới đây là các tài liệu mà ebook này có tham khảo và đồng thời có cả những tài liệu mà
bọn mình đề xuất cho bạn đọc
[1]. A comprehensive course in number theory – Alan Baker – Cambridge University Press (2012).
[2]. Problem – Solving and Selected Topics in Number Theory_ In the Spirit of the
Mathematical Olympiads – Michael Th. Rassias-Springer – Verlag New York (2011).
[3]. Tính chất số học trong các bài toán về đa thức – Phạm Viết Huy – THPT Chuyên Lê Khiết – Quảng Ngãi.
[4] Chuyên đề đa thức và số học – Nguyễn Thành Nhân – Chuyên Hùng Vương – Bình Dương
[5]. Number Theory A Historical Approach – John J.Watkins.
[6]. Number theory and polynomials (London Mathematical Society Lecture Note Series) –
James McKee, Chris Smyth – CUP (2008).
[7]. An Introduction to Theory of Functional Equations – Marek Kuczma, Attila Gilányi and Inequalities.
[8]. The IMO Compendium. A Collection of Problems Suggested for The International
Mathematical Olympiads: 1959 – 2009 – Djukic D., Vladimir Jankovic, Ivan Matic, Nikola Petrovic – Springer (2011).
[9]. Problem – Solving and Selected Topics in Number Theory – Michael Th. Rassias.
[10]. Number Theory Structures, Examples and Problems – Titu Andreescu, Dorin Andrica.
[11]. Elementary Methods in Number Theory – Nathanson M.B.
[12]. Elementary Number Theory Primes Congruences and Secrets A Computational Approach – William Stein.
[13]. http://maths.vn/tag/so-hoc/?fbclid=IwAR34ppP14Xm45l9dK8H_R6YKpjKPdSeCY- AT1eRjLwQATzvluVo4raqrVH4. [14]. diendantoanhoc.net.
[15]. artofproblemsolving.com.
[16]. Đột phá đỉnh cao bồi dưỡng học sinh giỏi chuyên đề số học – Văn Phú Quốc.
[17]. A Computational Introduction To Number Theory And Algebra – Victor Shoups.
[18]. Tuyển chọn các bài toán trong kì thi chọn đội tuyển của các tỉnh, thành phố năm học
2016 – 2017 – Toán học cho mọi người.
[19]. Định hướng bồi dưỡng học sinh năng khiếu toán – Nhà xuất bản Giáo dục Việt Nam
[20]. 104 Number Theory Problems: From the Training of the USA IMO Team – Titu
Andreescu, Dorin Andrica, Zuming Feng.
[21]. Tài liệu ôn đội tuyển VMO 2015 – Thầy Trần Minh Hiền – Chuyên Quang Trung – Bình Phước.
TẠP CHÍ VÀ TƯ LIỆU TOÁN HỌC CHINH PHỤC OLYMPIC TOÁN
TẠP CHÍ VÀ TƯ LIỆU TOÁN HỌC
Thôn 6 – Thạch Hòa – Thạch Thất – Hà Nội
Điện thoại: 0343763310; Email: tuangenk@gmail.com
Fanpage: https://www.facebook.com/OlympiadMathematical/
CHỊU TRÁCH NHIỆM NỘI DUNG DOÃN QUANG TIẾN NGUYỄN MINH TUẤN TÔN NGỌC MINH QUÂN HUỲNH KIM LINH BIÊN TẬP NGUYỄN MINH TUẤN
TRÌNH BÀY BẢN THẢO NGUYỄN MINH TUẤN ĐA THỨC VÀ SỐ HỌC
Đề nghị quý bạn đọc tôn trọng bản quyền của tác giả, không sao chép bản phụ.
Mọi ý kiến thắc mắc đóng góp vui lòng gửi về địa chỉ đã cung cấp ở trên.
Phiên bản sách điện tử được phát hành vào ngày 2/9/2019. CHINH PHỤC OLYMPIC TOÁN
MỌI Ý KIẾN THẮC MẮC XIN VUI LÒNG GỬI VỀ ĐỊA CHỈ DOÃN QUANG TIẾN 0817431404
doanquangtien1442001@gmail.com
https://www.facebook.com/OlympiadMathematical/
Đại học KHTN – Đại học Quốc Gia TP.HCM LIMITED EDITION